3) Горение на воздухе (отличие от аммиака)

advertisement
Аннотация
Основной учебной целью предлагаемого пособия является повторение и
обобщение знаний всего курса химии. Одновременно продолжается развитие умений
самостоятельно работать, систематически заниматься решением задач, работать с тестами
различных типов. Химия – один из самых сложных общеобразовательных предметов.
Успешно овладеть даже базовым школьным курсом химии невозможно, если у ученика
недостаточно развит мыслительный процесс. Особенно востребованы на уроках химии
умения анализировать учебный материал, сравнивать, обобщать, а также способность к
абстрагированию и в этом большую помощь оказывает применение опорных конспектов и
алгоритмов. Для успешной работы на ЕГЭ и на других видах контроля в аналогичном
формате, ученики должны знать основные виды заданий, ориентироваться в их структуре,
понимать, в какой форме нужно давать ответ. Большую помощь в подготовке к экзамену
ученикам и педагогам окажет предлагаемое пособие.
Опорные конспекты и алгоритмы выполнения заданий ЕГЭ по
химии (части А, В)
А1. Строение электронных оболочек атомов элементов первых четырех
периодов: s-, p- и d-элементы. Электронная конфигурация атома. Основное и
возбужденное состояние атомов.
1.
Число электронов, протонов и нейтронов в атоме.
Число электронов равно порядковому номеру и числу протонов, число нейтронов
равно разности между массовым числом и порядковым номером.
2.
Физический смысл порядкового номера, номера периода и номера группы.
Порядковый номер равен числу протонов и электронов, заряду ядра; номер периода
равен числу заполняемых электронных слоёв; номер А - группы равен числу электронов
на внешнем слое (валентных электронов).
3.
Максимальное число электронов на уровнях.
Максимальное число электронов на уровнях определяется по формуле N= 2· n2.
1 уровень – 2, 2 уровень – 8, 3 уровень - 18, 4 уровень – 32.
4.
s-, p- и d- элементы.
s-элементы расположены в I и II А-группах; p- элементы в III –VIII А –группах; dэлементы в I- VIII В группах.
5.
Порядок заполнения электронами Е уровней и орбиталей в атомах.
1s 2s 2p 3s 3p 4s 3d 4p 5s 4d 5p 6s 4f 5d 6p 7s 5f 6d 7p
6.
«Провал» электрона — переход электрона с внешнего энергетического
уровня на более низкий, что объясняется большей энергетической устойчивостью
образующихся при этом электронных конфигураций. Подобное явление наблюдается у
меди и хрома.
2 2 6 2 6 5 1
2 2
6 2 6
10 1
24Cr 1s 2s 2p 3s 3p 3d 4s
29Cu 1s 2s 2p 3s 3p 3d 4s
7.
Особенности заполнения электронных оболочек у элементов А и В групп.
У элементов А-групп валентные электроны заполняют последний слой, а у
элементов В-групп- внешний и частично предвнешний электронный слой.
8.
Степени окисления элементов в высших оксидах и ЛВС.
Группы
I
II
III
IV
V
VI
VII
VIII
С.О. в высшем оксиде = + №
гр
+1
+2
+3
+4
+5
+6
+7
Кочулева Людмила Рамановна, учитель химии МОБУ «Средняя общеобразовательная школа №73» г.
Оренбурга. Опорные конспекты и алгоритмы для подготовки к ЕГЭ.
+8
Высший оксид
С.О. в ЛВС = № гр - 8
R2 О
-
RО
-
R2О3
-
RО2
-4
R2О5
-3
RО3
-2
R2О7
-1
RО4
-
-
-
-
Н4R
Н3R
Н2R
НR
-
ЛВС
9.
Строение электронных оболочек ионов.
У катиона – меньше электронов на величину заряда, у анионов - больше на
величину заряда.
Напимер: Na0 - 11 электронов, Na+ - 10 электронов; P0 – 15 электронов, P3- - 18
электронов.
10.
Основное и возбужденное состояние атомов.
Основное состояние – состояние атома, в котором его энергия минимальна.
Возбужденные состояния атомов образуются из основного состояния при переходе
одного или нескольких электронов (например, под действием излучения или при
нагревании) с занятых орбиталей на свободные (или занятые лишь одним электроном).
Пример: Al - …3s2 3p1, *Al - 3s13p2; Р …3s2 3p3, Р* - 3s13p3 3d1.
11. Изотопы.
Изотопы - разновидности атомов одного ХЭ, имеющие одинаковое число
электронов и протонов, но разную массу атома (разное число нейтронов).
Пример:
Элементарные
Изотопы
40
42
частицы
Ca
Ca
é
20
20
p+
20
20
0
n
20
22
Алгоритмы построения электронных формул и структур элементов
1.
Алгоритм построения электронной структуры
1). Сначала надо построить таблицу, в которой количество строк должно быть
равно номеру периода искомого элемента. Возьмем для примера атом цинка.
Zn находится в IV периоде.
2). Указательный палец левой руки
движется по клеткам таблицы Менделеева, от одного элемента к последующему в
соответствии с их номерами. А правая рука в это время расставляет стрелочки в таблицу.
3). При этом необходимо каждый раз определять, в каком периоде стоит этот
элемент и какой подуровень у него заполняется, т.е. это s-, p-, d- или f-элемент.
4). Потом по правилам размещения электронов на орбиталях ставить стрелочку в
соответствующую клетку. Затем палец сдвигается на новый элемент, и вся процедура
повторяется.
Итак:
1)Сначала установим палец на начало периодической системы, он укажет нам на
водород.
Это s-элемент, т.е. надо поставить стрелку вверх в клетку с координатами I и s, т.е.
Is.
2)Следующий элемент Не, это тоже s-элемент, поэтому вторую стрелку надо
поставить в соответствии с принципом Паули в клетку Is, направленную вниз.
Кочулева Людмила Рамановна, учитель химии МОБУ «Средняя общеобразовательная школа №73» г.
Оренбурга. Опорные конспекты и алгоритмы для подготовки к ЕГЭ.
↑↓
3) Следующие элементы – литий и бериллий также s-элементы, но второго периода.
↑↓
↑↓
4)Следующие три элемента B, C, N - p-элементы, в соответствие с правилом Хунда их
электроны надо разместить в р-подуровне по одному в каждой клетке.
↑↓ ↑
↑↓
↑
↑
5)Остальные три р-элемента O,F и Ne размещаются в том же подуровне.
↑↓ ↑↓ ↑↓ ↑↓
↑↓
6)Продолжая двигать палец левой руки последовательно от одного элемента к другому,
без пропусков, мы построим всю электронную структуру Zn:
↑↓
↑↓ ↑↓ ↑↓ ↑↓ ↑↓ ↑↓ ↑↓ ↑↓ ↑↓
↑↓ ↑↓ ↑↓ ↑↓
↑↓
Построим по аналогичной схеме электронную структуру атома меди:
↑↓
↑↓ ↑↓ ↑↓ ↑↓ ↑↓ ↑↓ ↑↓ ↑↓ ↓
↑↓ ↑↓ ↑↓ ↑↓
↑↓
Мы видим, что в d-подуровне 9 электронов, поэтому необходимо применить
правило
«проскока»,т.е. перенести один электрон из s-подуровня внешнего IV-го уровня в
d-подуровень.
Таким образом, мы можем построить электронную структуру любого элемента.
Надо только запомнить, что правило «проскока» работает до ниобия, после ниобия оно
Кочулева Людмила Рамановна, учитель химии МОБУ «Средняя общеобразовательная школа №73» г.
Оренбурга. Опорные конспекты и алгоритмы для подготовки к ЕГЭ.
справедливо, лишь, если в d-подуровне 9 электронов, а правило о 4-х электронах на dподуровне уже не работает.
2. Алгоритм построения электронной формулы элемента
1)В электронной формуле не надо показывать место каждого электрона, в ней
указывается уровни, подуровни и количество электронов на каждом подуровне.
2).При построении полной электронной формулы также удобно использовать
алгоритм предложенный выше (назовем его правилом указательного пальца левой
руки). Скользя пальцем по таблице элементов от одного элемента к другому,
последовательно записываются заполняемые подуровни.
Напишем электронную формулу цинка: 1s22s22p63s23p64s23d10.
3). Можно написать и сокращенную электронную формулу, для этого находим
перед цинком последний инертный газ в таблице Менделеева, которым заканчивается
период (в данном случае это аргон).
4).Записываем символ инертного газа в квадратные скобки и заполняем по правилу
указательного пальца электронную формулу после аргона. [Ar] 4s23d10
Полная электронная формула меди будет:
1s22s22p63s23p64s13d10,сокращенная [Ar] 4s13d10.
Алгоритм составления электронных формул атомов (на примере атома
йода)
№
операци
и
Операция
Результат
1
Определите координаты
атома в таблице элементов.
Период 5-й, группа VIIA
2
Составьте валентную
электронную формулу.
3
Допишите символы
внутренних электронов в
последовательности
заполнения ими
подуровней.
4
Учитывая уменьшение
энергии полностью
заполненных d- и fподуровней, запишите
полную электронную
формулу.
5
Отметьте валентные
электроны.
5s25p5
1s22s22p63s23p64s23d104p65s24d105p5
1s22s22p63s23p63d104s24p64d105s25p5
Кочулева Людмила Рамановна, учитель химии МОБУ «Средняя общеобразовательная школа №73» г.
Оренбурга. Опорные конспекты и алгоритмы для подготовки к ЕГЭ.
6
Выделите электронную
конфигурацию
предшествующего атома
благородного газа.
7
Запишите сокращенную
электронную формулу,
объединив в квадратных
скобках
все невалентные электрон
ы.
[Kr, 4d10]5s25p5
3.Алгоритм описания химических свойств элементов
1). Указывают положение элемента в Периодической системе.
2). Составляют полную и краткую электронную формулу. На основании этого
делают вывод о том, к какому классу элементов относится данный элемент (s-, p-, d- или fэлементам).
3). Характеризуют металлические свойства элемента.
4).Указывают форму гидрида этого элемента ( в гидридах степень окисления
неметаллов минимальна и равна номеру группы минус 8). Делают вывод о газообразности
гидрида ( гидриды неметаллов газообразны).
а). Гидриды не образуются с благородными газами, с платиновыми металлами
(исключение – Pd), Ag, Au, Cd, Hg, In, Tl.
5). Записывают формулу высшего оксида данного элемента и делают вывод о его
кислотно-основных и агрегатных свойствах (Газообразны только оксиды серы, углерода и
азота).
а). Пишут формулу кислоты или основания для данного оксида, дают им названия.
б). Делают вывод о силе кислоты или основания этого оксида, а также об их
окислительно-восстановительных свойствах.
в). Указывают названия солей, образованных этим оксидом.
6). Если есть оксиды, в которых данный элемент имеет другие степени окисления,
оксиды характеризуются по плану 5-го пункта.
Пример 1. Описать свойства ванадия.
1)V- элемент с номером 23 находится в IV периоде в группе VБ.
2)Электронная формула 1s22s22p63s23p64s23d3 или сокращенно [Ar] 4s23d3.
3)Следовательно, ванадий d-элемент, проявляющий металлические свойства.
4)Ванадий образует негазообразный гидрид, который не обладает кислотными
свойствами.
5)Высшая степень окисления у V +5 (при этом он отдает 2 электрона с s-подуровня
и 3 электрона с d-подуровня). Оксид, соответствующий высшей степени окисления –
V2O5. Этот оксид обладает кислотными свойствами.
а) Ванадий в высшей степени окисления способен образовывать метаванадиевую
кислоту HVO3.
в) Соли этой кислоты – ванадаты. Эти соли и кислоты – сильные окислители.
Кочулева Людмила Рамановна, учитель химии МОБУ «Средняя общеобразовательная школа №73» г.
Оренбурга. Опорные конспекты и алгоритмы для подготовки к ЕГЭ.
6) Ванадий может отдать 2 электрона с внешнего s-подуровня, проявляя степень
окисления +2. При этом он образует оксид VO, проявляющий основные свойства, ему
соответствует гидроксид ванадия (II) – V(OH)2, имеющий свойства слабого основания.
Для решение задания А1 нужно знать как находить
количество протонов, электронов в атоме и ионе, знать как распределяются электроны по
электронным уровням и подуровням, уметь записывать электронную конфигурацию атома
и иона, находить количество неспаренных электронов в атоме и ионе, знать как связана
электронная конфигурация и валентность химического элемента
Тест №1
1. Количество электронов в атоме равно
1)числу протонов
2)числу нейтронов
3)числу энергетических уровней
4)относительной атомной массе
2 . Ион, в составе которого 16 протонов и 18 электронов, имеет заряд
1) +4
2) -2
3) +2
4) -4
3. Внешний энергетический уровень атома элемента, образующего высший оксид состава
ЭОз, имеет формулу
1) ns2np1
2) ns2nр2
3) nз2nр3
4) ns2nр4
4. Конфигурация внешнего электронного слоя атома серы в невозбужденном состоянии
1) 4s2 2) 3s23р6
3) 3s23р4
4) 4s24р4
5. Электронную конфигурацию 1s22s22p63s23p64s1 в основном состоянии имеет
атом
1) лития 2) натрия 3) калия 4) кальция
6. Восьмиэлектронную внешнюю оболочку имеет ион
1) Р3+
2) S23) С15+
4) Fe2+
7. Двухэлектронную внешнюю оболочку имеет ион
1) S6+
2) S23) Вг5+
4) Sn4+
8. Число электронов в ионе железа Fe2+ равно
1) 54
2) 28
3) 58
4) 24
9. Электронная конфигурация Is22s22p63s23p6 соответствует иону
1) Sn2+
2) S23) Cr3+
4) Fe2+
10. В основном состоянии три неспаренных электрона имеет атом
1) кремния 2) фосфора 3) серы 4) хлора
11. Элемент с электронной конфигурацией внешнего уровня ... 3s23p3 образует
водородное соединение состава
1) ЭН4
2) ЭН
3) ЭН3
4) ЭН2
12. Электронная конфигурация Is22s22p63s23p6 соответствует иону
1) Сl2) N33) Br4) О213. Электронная конфигурация Is22s22p6 соответствует иону
1) А13+
2) Fe3+
3) Zn2+
4) Cr3+
14. Одинаковую электронную конфигурацию внешнего уровня имеют Са2+ и
1) К+
2) Аr
3) Ва
4) F15. Атом металла, высший оксид которого Ме2О3, имеет электронную формулу
внешнего энергетического уровня
1) ns2пр1
2) ns2пр2
3) ns2np3
4) ns2nps
Кочулева Людмила Рамановна, учитель химии МОБУ «Средняя общеобразовательная школа №73» г.
Оренбурга. Опорные конспекты и алгоритмы для подготовки к ЕГЭ.
16. Элемент, которому соответствует высший оксид состава R2O7 имеет
электронную конфигурацию внешнего уровня:
1) ns2np3
2)ns2np5
3) ns2np1
4) ns2np2
17. Высший оксид состава R2O7 образует химический элемент, в атоме которого
заполнение электронами энергетических уровней соответствует ряду чисел:
1) 2, 8, 1
2) 2, 8, 7
3) 2, 8, 8, 1
4) 2, 5
18. У атома серы число электронов на внешнем энергетическом уровне и заряд ядра
равны соответственно
1)4 и + 16 2)6 и + 32
3)6 и + 16 4)4 и + 32
19. Число валентных электронов у марганца равно
1) 1
2) 3
3) 5
4) 7
20. Одинаковое электронное строение имеют частицы
1) Na0 и Na+
2) Na0 и K0
3) Na+ и F4) Cr2+ и Сr3+
21. Высший оксид состава ЭО3 образует элемент с электронной конфигурацией внешнего
электронного слоя
1) ns2np1
2) ns2np3
3) ns2np4
4) ns2np6
22. Число энергетических слоев и число электронов во внешнем энергетическом слое
атомов мышьяка равны соответственно
1)4, 6 2)2, 5 3)3, 7 4)4, 5
23 Иону Al3+ отвечает электронная конфигурация:
1) 1s22s22p6;
2) 1s22s22p63s1;
3) 1s22s22p63s23p1 4) Is22s22p63s23p64s1
24. Иону Zn2+ отвечает электронная конфигурация:
1) 1s22s22p63s23p63d84s2
2) 1s22s22p63s23p63d104s24p6
3 ) 1s22s22p63s23p63d10
4) Is22s22p63s23p64s1
25. Химическому элементу соответствует летучее водородное соединение состава RH3.
Электронная конфигурация внешнего уровня этого элемента
1)3s23p1
2)3s23p2
3)3s23p3
4)3s23p5
26. Атомы серы и кислорода имеют
1)одинаковое число электронных слоев
2)одинаковое число электронов внешнего электронного слоя
3)одинаковое число протонов в ядре
4)одинаковые радиусы
27. Электронная конфигурация атома фтора
1)1s22s22p5 2)1s22s22p4
3)1s22s22p6
4)1s22s22p3
28. Сколько неспаренных электронов имеет атом углерода в состоянии sp3-гибридизации?
1) 1
2) 2
3) 3
4) 4
29. У атома хлора на третьем электронном уровне имеется одна s-орбиталь, три pорбитали и пять d-орбиталей. Максимальная валентность хлора равна
1)четырем 2)семи
3)восьми
4)девяти
30. Элемент, электронная конфигурация атома которого 1s22s22p63s23p2 образует
водородное соединение
1) СН4
2) SiH4
3) H2O
4) H2S
31. Какую электронную конфигурацию внешнего уровня имеют атомы IVA группы?
1) ns2np5
2) ns2np2
3) ns2np4
2) ns2np6
32. Одинаковое число валентных электронов имеют атомы калия и
1) углерода
2) магния
3) фосфора
4) натрия
Кочулева Людмила Рамановна, учитель химии МОБУ «Средняя общеобразовательная школа №73» г.
Оренбурга. Опорные конспекты и алгоритмы для подготовки к ЕГЭ.
Ответы: 1-1, 2-2,3-4,4-3,5-3,6-2,7-3,8-4,9-2, 10-2, 11-3, 12-1, 13-1, 14-1, 15-1, 16-2, 17-2, 183, 19-4, 20-3, 21-3, 22-4, 23-1, 24-3, 25-4, 26-2, 27-1, 28-4, 29-2, 30-2, 31-2, 32-4.
А2. Закономерности изменения химических свойств элементов и
соединений по периодам и группам.
их
1.
Физический смысл порядкового номера, номера периода и номера группы.
Порядковый номер равен числу протонов и электронов, заряду ядра; номер периода
равен числу заполняемых электронных слоёв; номер А - группы равен числу электронов
на внешнем слое (валентных электронов).
2.
Горизонтальная периодичность.
В периоде слева направо:
- заряды атомных ядер увеличиваются;
- число электронов на внешнем электронном слое увеличивается;
- число электронных слоёв не изменяется;
- радиус атомов уменьшается;
- ЭО увеличивается;
- металлические свойства уменьшаются, неметаллические свойства увеличиваются;
- основные свойства соединений (оксидов, гидроксидов) уменьшаются, сменяются
амфотерными, кислотные свойства (оксидов, кислот) увеличиваются.
3.
Вертикальная периодичность.
В А - группе сверху вниз:
-заряды атомных ядер увеличиваются;
- число электронов на внешнем электронном слое не изменяется;
- число электронных слоёв увеличивается;
- радиус атомов увеличивается;
- ЭО уменьшается;
- металлические свойства увеличиваются, неметаллические свойства уменьшаются;
- основные свойства соединений (оксидов, гидроксидов) увеличиваются, кислотные
свойства соединений (оксидов, кислот) уменьшаются.
Тест №2
1. В ряду
Na -->Mg -->Al -->Si
1) увеличивается число энергетических уровней в атомах
2) усиливаются металлические свойства элементов
3) уменьшается высшая степень окисления элементов
4) ослабевают металлические свойства элементов
2. У элементов подгруппы углерода с увеличением атомного номера уменьшается
1) атомный радиус
2) заряд ядра атома
3) число валентных электронов в атомах
4) электроотрицательность
3. В ряду элементов
азот - кислород - фтор возрастает
1) валентность по водороду
2) число энергетических уровней
3) число внешних электронов
4) число неспаренных электронов
4. В ряду химических элементов бор - углерод - азот возрастает
1) способность атома отдавать электроны
Кочулева Людмила Рамановна, учитель химии МОБУ «Средняя общеобразовательная школа №73» г.
Оренбурга. Опорные конспекты и алгоритмы для подготовки к ЕГЭ.
2) высшая степень окисления
3) низшая степень окисления
4) радиус атома
5. Какой элемент имеет более выраженные неметаллические свойства, чем кремний?
1) углерод
2) германий
3) алюминий
4) бор
6. С ростом заряда ядра атомов кислотные свойства оксидов в ряду
N2O5 --> P2O5 --> As2O5 --> Sb2O5
1) ослабевают
2) усиливаются
3) не изменяются
4) изменяются периодически
7. В порядке возрастания неметаллических свойств элементы расположены в ряду:
1) O,N,C,B 2) Cl,S,P,Si
3) C,Si,Ge,Sn
4) B,C,O,F
8. В порядке усиления металлических свойств элементы расположены в ряду:
1) А1,Са,К
2) Ca.Ga.Fe
3) K,Al,Mg
4) Li,Be,Mg
9. В каком ряду элементы расположены в порядке возрастания их атомного радиуса?
1) Si,P, S.C1
2) O,S,Se,Te
3) At,I,Br,Cl
4) Mg,Al,Si, P
10. Какой элемент образует газообразное водородное соединен
соответствующее общей формуле RH2?
1) бор
2) калий
3) сера
4) хром
11. В главных подгруппах периодической системы восстановительная способность
атомов химических элементов растет с
1) уменьшением радиуса атомов
2) увеличением числа энергетических уровней в атомах
3) уменьшением числа протонов в ядрах атомов
4) увеличением числа валентных электронов
12. В какой группе периодической системы находится элемент Э, входящий в состав
кислоты НЭО4?
1) IV
2) V
3) VI
4) VII
13. В ряду оксидов SiO2 - Р2О5 - SO2 - Cl2O7 кислотные свойства
1) возрастают
2) убывают
3) не изменяются
4) сначала уменьшаются, потом увеличиваются
14. В каком ряду простые вещества расположены в порядке усиления
металлических свойств?
1) Mg, Ca, Ва
2) Na, Mg, A1
3) K,Ca,Fe
4) Sc, Ca, Mg
15. По периоду слева направо уменьшается(-ются)
1) атомный радиус элементов
2) число валентных электронов в атомах
3) электроотрицательность элементов
4) кислотные свойства гидроксидов
16. В порядке увеличения электроотрицательности химические элементы
расположены в раду:
1) С, N, О
2) Si.Al.Mg
3) Mg,Ca, Ва
4) Р, S, Si
17. Химический элемент расположен в IV периоде, IA группе. Распределению электронов
в атоме этого элемента соответствует ряд чисел:
1) 2,8,8,2
2) 2, 8, 18, 1
3) 2, 8, 8, 1
4) 2,8, 18,2
18. Электроотрицательность химических элементов с возрастанием заряда ядра атома
1) увеличивается и в периодах, и в группах
2) уменьшается и в периодах, и в группах
3) увеличивается в периодах, а в группах уменьшается
Кочулева Людмила Рамановна, учитель химии МОБУ «Средняя общеобразовательная школа №73» г.
Оренбурга. Опорные конспекты и алгоритмы для подготовки к ЕГЭ.
4) уменьшается в периодах, а в группах увеличивается
19. В каком ряду химические элементы расположены в порядке возрастания их атомного
радиуса?
1) Rb,K,Na,Li
2) Na,Mg,Al, S
3) О, S, Se, Те
4) C,N, О, F
20. Среди элементов третьего периода наименьший атомный радиус имеет
1) натрий 2) алюминий 3) фосфор 4) сера
21. В главных подгруппах периодической системы восстановительная способность
атомов химических элементов растет с
1) уменьшением радиуса атомов
2) увеличением числа энергетических уровней в атомах
3) уменьшением числа протонов в ядрах атомов
4) увеличением числа валентных электронов
22. По периоду слева направо уменьшается
1) число валентных электронов в атомах
2) атомный радиус элементов
3) электроотрицательность элементов
4) кислотность гидроксидов элементов
23. Наиболее сильное основание образует
1) цезий
2) натрий
3} литий
4) цинк
24. Оксид с наиболее выраженными кислотными свойствами образует
1) кремний
2) фосфор
3) сера
4) хлор
25. Наиболее сильное основание образует
1) магний
2) стронций
3) барий
4) кадмий
26. Кислотный характер наиболее выражен у высшего оксида, образованного элементом:
1) Sn
2) А1
3) С
4) S
27. Кислотный характер наиболее выражен у высшего оксида, образованного
1) бериллием
2) бором
3) фосфором
4) кремнием
28. Сила бескислородных кислот неметаллов VIIА группы соответственно возрастанию
заряда ядра атомов элементов
1)увеличивается 2)уменьшается 3)не изменяется 4)изменяется периодически
29. Одинаковое значение валентности в водородном соединении и высшем оксиде имеет
элемент
1)хлор
2)германий
3)мышьяк
4)селен
30. Кислотные свойства оксидов в ряду SiO2 --> P2O5 -->SО3
1) ослабевают
2) усиливаются
3) не изменяются
4) изменяются периодически
31. Газообразные водородные соединения состава ЭН3 образуют
1) Be, Ca, Sr
2) P, As, Sb
3) Ga, Al, B
4) Te, S, Sc
32. В ряду элементов
Cl → S→ P→ Si
1) уменьшается число электронных слоев в атомах
2) увеличивается число внешних электронов в а томах
3) возрастают радиус атомов
4) усиливаются неметаллические свойства
33. Неметаллические свойства наиболее выражены у
1) серы
2) кислорода 3) кремния
4) фосфора
34. Наибольший радиус имеет атом
Кочулева Людмила Рамановна, учитель химии МОБУ «Средняя общеобразовательная школа №73» г.
Оренбурга. Опорные конспекты и алгоритмы для подготовки к ЕГЭ.
1) олова
2} кремния
35. В ряду химических элементов
Na -->Mg --> Al --> Si
1)
2)
3)
4)
3) свинца
4} углерода
увеличивается число валентных электронов в атомах
уменьшается число электронных слоев а атомах
уменьшается число протонов в ядрах атомов
увеличиваются радиусы атомов
36.Наибольший радиус имеет атом
1) брома
2) мышьяка
3) бария
4) олова
37.Электронную конфигурацию 1s22s22р63.s2Зр63d1 имеет ион
1) Са2+
2) А13+
3) K+
4) Sc2+
38. Какую электронную конфигурацию имеет атом наиболее активного металла?
1)1s22s22p1
2)1s22s22p63s1 3)1s22s2 4) 1s22s22p63s23p1
Ответы 1-4, 2-4, 3-3, 4-2, 5-1, 6-1, 7-4, 8-1, 9-2, 10-3, 11-2, 12-4, 13-1, 14-1, 15-1, 16-1, 17-3,
18-3, 19-3, 20-4, 21-2, 22-2, 23-1, 24-4, 25-3, 26-4, 27-3, 28-1, 29-2, 30-2, 31-2, 32-3, 33-2, 343, 35-1, 36-3, 37-4, 38-2.
А3. Общая характеристика металлов главных подгрупп I – III групп в связи с
их положением в периодической системе химических элементов Д. И. Менделеева и
особенностями строения их атомов. Характеристика переходных элементов – меди,
цинка, хрома, железа по их положению в периодической системе химических
элементов Д. И. Менделеева и особенностям строения их атомов.
Общая
характеристика неметаллов главных подгрупп IV – VII групп в связи с их
положением в периодической системе химических элементов Д.И. Менделеева и
особенностями строения их атомов.
ХЭ
IА
Ве
II А
Аl
Сu
Zn
Cr
Fe
Mn
C
СО
+1
+2
+2
+3
+1; +2
+2
+2
+3
+6
+2
+3
+2
+4
+6
+7
-4
-1
+2
характер соединений
основные
амфотерный
основные
амфотерный
основный
амфотерный
основный
амфотерный
кислотный
основный
амфотерный
амфотерный
основный
кислотный
кислотный
несолеобразующий
соединения
оксиды, щелочи
BeO, Be(OH)2 = H2BeO2
оксиды, основания
Al2O3, Al(OH)3 = HAlO2
оксиды, основания
ZnO, Zn(OH)2 = H2ZnO2
CrO, Cr(OH)2
Cr2O3, Cr(OH)3 = HСrO2
CrO3, H2CrO4
FeO, Fe(OH)2
Fe2O3, Fe(OH)3= HFeO2
MnO, Mn(OH)2
MnO2, Mn(OH)3
MnO3, H2MnO4
Mn2O7, HMnO4
СH4, карбид алюминия Аl4С3
карбиды (Na2С2, СаС2)
СО
Кочулева Людмила Рамановна, учитель химии МОБУ «Средняя общеобразовательная школа №73» г.
Оренбурга. Опорные конспекты и алгоритмы для подготовки к ЕГЭ.
Si
N
P
O
S
F
остальные
галогены
(на примере Cl)
+4
кислотный
СО2, Н2СО3, карбонаты
-4
+2
+4
-3
+1
+2
+3
+4
+5
-3
+3
+5
-2
-1
+2
-2
+2
+4
+6
-1
-1
+1
+3
+5
+7
несолеобразующий
кислотный
несолеобразующий
несолеобразующий
кислотный
кислотный
кислотный
кислотный
кислотный
кислотный
кислотный
кислотный
кислотный
кислотный
кислотный
силициды М
SiО
SiО2, Н2SiО3, силикаты
NН3, соли аммония, нитриды М
N2О
NО
N2О3, НNО2, нитриты
NО2, N2О5, НNО3, нитраты
PН3, фосфиды М
P2О3
P 2О5, Н3РО4, фосфаты, НРО3
оксиды
пероксиды
ОF2
Н2S, сульфиды Ме
SCl2
SО2, Н2SО3, сульфиты
SО3, Н2SО4, сульфаты
НF, фториды
ЛВС, галогениды
Cl2O, НClO
Cl2O3
Cl2O5
Cl2O7, НClO4, перхлораты
Характер соединений:
ХЭ
металл
основный
С.О.+1,+2
неметалл
-
амфотерный
С.О.+2, +3, +4
амф. Ме – Ве, Аl, Zn,
Cr, Fe, Mn
-
несолеобразующий
-
кислотный
С.О.+5, +6, +7
С.О.+1,+2 (искл.
Cl2O )
С.О.+3,+4,+5,+6,+7
Тест №3
1. В порядке
ряду:
1) K,Al,Cr,Sn
увеличения
восстановительной
2) Sn,Cr,Al,Zn
3) Sn,Ca,Al,K
способности
металлы расположены в
4) Au,Al,Ca,Li
2. Щелочные металлы
1)являются сильными восстановителями
2) проявляют как окислительные, так и восстановительные свойства
3) легко образуют отрицательно заряженные ионы
Кочулева Людмила Рамановна, учитель химии МОБУ «Средняя общеобразовательная школа №73» г.
Оренбурга. Опорные конспекты и алгоритмы для подготовки к ЕГЭ.
4) легко присоединяют электроны в химических реакциях
3. В ряду элементов:
натрий -->магний -->алюминий
возрастает их
1) атомный радиус
2) восстановительная способность
3) химическая активность
4) электроотрицательность
4. У магния металлические свойства выражены
1) слабее, чем у бериллия
2) сильнее, чем у алюминия
3) сильнее, чем у кальция
4) сильнее, чем у натрия
5. В порядке уменьшения восстановительных свойств металлы расположены в ряду:
1) Al,Zn,Fe
2) Al,Na,K
3) Fе,Zn,Mg
4) Fe,Zn,Al
6. Наибольший радиус имеет атом
1) лития
2) натрия
3) кальция
4) калия
7. У элементов II А группы сверху вниз
1) уменьшаются радиусы атомов
2) увеличивается число валентных электронов в атоме
3) увеличиваются радиусы атомов
4) уменьшается число валентных электронов в атоме
8. Сила оснований возрастает в ряду:
1) Ве(ОН)2, Mg(OH)2, Ca(OH)2
2) Ва(ОН)2, Са(ОН)2, Ве(ОН)2
3) Са(ОН)2, Mg(OH)2, Ве(ОН)2
4) Sr(OH)2, Ca(OH)2, Mg(OH)2
9. У элементов I А группы сверху вниз
1) усиливаются окислительные свойства
2) ослабевают восстановительные свойства
3) увеличиваются радиусы атомов
4) уменьшаются радиусы атомов
10. Валентные электроны наиболее легко отдают атомы
1) алюминия
2) натрия
3) бериллия
4) магния
11. Восстановительные свойства наиболее выражены у
1) алюминия 2) магния
3) натрия
4) калия
12. Для растворения как меди, так и железа, следует использовать
1) концентрированную фосфорную кислоту
2) разбавленную азотную кислоту
Кочулева Людмила Рамановна, учитель химии МОБУ «Средняя общеобразовательная школа №73» г.
Оренбурга. Опорные конспекты и алгоритмы для подготовки к ЕГЭ.
3) разбавленную соляную кислоту
4) раствор гидроксида калия
13. К основным гидроксидам относится каждое из двух веществ:
1) Fe(OH)3 и Си(ОН)2
3) Fe(OH)2 и Ca(OH)2
2) Fe(OH)3 и Сг(ОН)2
4) Fe(OH)3 и Сг(ОН)3
14. При нагревании меди с концентрированной серной кислотой образуется
1) оксид серы (IV) 2) водород
3) оксид серы (VI) 4) сероводород
15. Медь может вступать во взаимодействие с водным раствором
1) гидроксида натрия
2) хлорида кальция
3) нитрата цинка
4) азотной кислоты
16. Основные свойства веществ ослабевают в ряду:
1) NaОН --> КОН -->RbOH
2) А1(ОН)3 -->Mg(OH)2 --> NaOH
3) Са(ОН)2 --> Mg(OH)2 -->Be(OH)2
4) В(ОН)3 -->Ве(ОН)2 --> LiOH
17. Верны ли следующие суждения?
А. И хром, и железо образуют устойчивые оксиды в степени окисления +3.
Б. Оксид хрома (III) является амфотерным.
1) верно только А
2) верно только Б
3) верны оба суждения
4) оба суждения неверны
18. Верны ли следующие суждения?
А. Только s-элементы содержит IA группа.
Б. Все элементы IA группы взаимодействуют с водой при комнатной температуре.
1) верно только А
2) верно только Б
3) верны оба суждения 4) оба суждения неверны
19. Оксид хрома (VI) является
1) основным
2) кислотным
3) амфотерным
4) несолеобразующим
20. При обычных условиях практически осуществима реакция между железом и
1) серой (тв)
2) серной кислотой (конц.)
3) нитратом меди (II) (р-р)
4) нитратом цинка (р-р)
21. Только при нагревании с водой реагируют
1) К и Hg
2) Zn и Fe
3) Cs и Ag
22. Только основные свойства проявляет
1) Сr2O3
2) Сr(ОН)2
3) СrO3
4) Sr и Сu
4) Сr(ОН)3
23. Сильные окислительные свойства характерны для
Кочулева Людмила Рамановна, учитель химии МОБУ «Средняя общеобразовательная школа №73» г.
Оренбурга. Опорные конспекты и алгоритмы для подготовки к ЕГЭ.
1) оксида меди (I)
2) оксида железа (II)
3) оксида хрома (III)
4) оксида хрома (VI)
24. Верны ли следующие суждения об оксидах железа?
А. Степень окисления железа в высшем оксиде равна + 3.
Б. Высший оксид железа относится к основным оксидам.
1) верно только А
2) верно только Б
3) верны оба суждения
4) оба суждения неверны
25. В ряду оксидов
CrO - Сr2О3 - СrОз
происходит
1) уменьшение степени окисления хрома
2) усиление восстановительных свойств
3) увеличение массовой доли хрома
4) усиление кислотных свойств
26. Оцените справедливость суждений о металлах:
А. Чем сильнее атом удерживает валентные электроны, тем ярче
выражены металлические свойства элемента.
Б. Чем сильнее выражены металлические свойства элемента, тем
более основный характер имеет его гидроксид.
1) верно только А
2) верно только Б 3) верны оба суждения
4) оба суждения неверны
27. Оцените справедливость суждений о металлах:
А. Для атомов металлов характерно малое число валентных
электронов и слабое их притяжение к ядру.
Б. Чем выше степень окисления металла в его гидроксиде, тем
более основными свойствами обладает гидроксид.
1) верно только А 2) верно только Б 3) верны оба суждения
4) оба суждения неверны
28. Оцените справедливость суждений о металлах:
А. Атомы металла могут образовывать только ионные связи.
Б. Оксиды и гидроксиды металлов всегда имеют основный
характер.
1) верно только А 2) верно только Б
3) верны оба суждения 4) оба суждения неверны
29. Верны ли следующие суждения о неметаллах?
А. В периодической системе химических элементов Д.И. Менделеева все неметаллы
располагаются в главных подгруппах.
Б. Все неметаллы являются р-элементами.
1) верно только А 2) верно только Б
3) верны оба суждения 4) оба суждения неверны
30. При обычных условиях из двухатомных молекул состоят
1) гелий и аргон
2) азот и неон 3) сера и фосфор 4) водород и кислород
31. Верны ли следующие суждения о неметаллах?
А. Все неметаллы являются химически активными веществами.
Б. Неметаллы обладают только окислительными свойствами.
1) верно только А 2) верно только Б
Кочулева Людмила Рамановна, учитель химии МОБУ «Средняя общеобразовательная школа №73» г.
Оренбурга. Опорные конспекты и алгоритмы для подготовки к ЕГЭ.
3) верны оба суждения 4) оба суждения неверны
32. Верны ли следующие суждения о неметаллах?
А. Неметаллы образуют с щелочными металлами соединения преимущественно с
ионной связью.
Б. Между собой неметаллы образуют соединения с ковалентной связью.
1) верно только А 2) верно только Б
3) верны оба суждения 4) оба суждения неверны
33. У атомов химических элементов, расположенных в ряду:
P-S-C1, увеличивается
1) радиус
2) окислительная способность
3) восстановительная способность
4) число неспаренных электронов
34. Соединения состава NaHЭO3 и NaHЭO4 может образовать
1) углерод
2) сера
3) хлор
4) фосфор
35. Наиболее сильными кислотными свойствами обладает
1) НС1О4
2) H2SO3
3) Н3РО4
4) H2SiО3
36 Соединения состава КЭО2 и КЭО3 образует элемент
1) азот
2) фосфор
3) сера
4) марганец
37. Водород проявляет окислительные свойства при реакции с
1) натрием
2) хлором
3) азотом
4) кислородом
38. Способность атомов химических элементов принимать электроны усиливается в
ряду:
1)F -->O -->N
2) N -->F -->О
3) N -->O -->F
4) O -->N -->F
39. Степени окисления хлора, брома и йода в высших оксидах и водородных соединениях
соответственно равны:
1)+1и-1
2)+7и-1
3)+7и-7
4)+5и-1
40. Сера проявляет как окислительные, так и восстановительные свойства при
взаимодействии с
1) водородом и железом
2) углеродом и цинком
3) хлором и фтором
4) натрием и кислородом
41. В ряду:
Si -->Р --> S --> С1
электроотрицательность элементов
1) увеличивается 2) уменьшается
3) не изменяется 4) сначала уменьшается, потом увеличивается
42. В ряду элементов мышьяк -->селен --> бром возрастает
1) атомный радиус
2) число неспаренных электронов в атоме
3) число электронных слоев в атоме
4) электроотрицательность
43. Водородное соединение состава Н2Э2 образует
1) углерод 2) кремний 3) бор 4) азот
44. Верны ли следующие суждения о галогенах?
А. Наиболее электроотрицательным среди галогенов является иод.
Б. Хлор вытесняется бромом из хлорида алюминия.
1) верно только А
2) верно только Б
3) верны оба суждения 4) оба суждения неверны
5. Кислород не реагирует с
1) водой и оксидом кальция
Кочулева Людмила Рамановна, учитель химии МОБУ «Средняя общеобразовательная школа №73» г.
Оренбурга. Опорные конспекты и алгоритмы для подготовки к ЕГЭ.
2) железом и оксидом фосфора (V)
3) водородом и оксидом фосфора (III)
4) сероводородом и оксидом углерода (IV)
46. Высшему гидроксиду элемента VIIA группы соответствует формула
1) Н2ЭО3
2) Н2ЭО4
3) НЭО3
4) НЭО4
47. Верны ли следующие суждения о галогенах?
А. Фтор в соединениях проявляет как положительную, так и
отрицательную степень окисления.
Б. При нормальных условиях бром и иод являются жидкостями.
1) верно только А
2) верно только Б 3) верны оба суждения
4) оба суждения неверны
48. Водород проявляет окислительные свойства при взаимодействии с
1) натрием
2) хлором
3) азотом
4) кислородом
49. Окислительные свойства фосфор проявляет при взаимодействии с
1) кислородом 2) магнием 3) хлором 4) серой
50. Верны ли следующие суждения о свойствах серы и хлора?
А. Максимальная валентность серы и хлора в соединениях равна номеру группы.
Б. В водородных соединениях серы и хлора связь ковалентная полярная.
1) верно только А 2) верно только Б
3) верны оба суждения 4) оба суждения неверны
51. Фосфор проявляет окислительные свойства при реакции с
1) кальцием
2) серой
3} хлором
4) кислородом
52. При взаимодействии высшего оксида хлора с водой образуется кислота
1) НС1O
2) НС1O2
3) НСlO3
4} HClO4
53. Характерными степенями окисления хлора в его соединениях являются:
1) -1, +1, +3, +5, +7
2) - 2, +4, +6, +8
3) -3, +3, +5
4) -1, +2, +5
54. Оцените справедливость суждений о неметаллах:
А. Атомы неметалла могут участвовать в образовании как ионных,
так и ковалентных связей.
Б. Гидроксиды неметаллов имеют кислотный характер.
1) верно только А
2) верно только Б
3) верны оба суждения 4) оба суждения неверны
55. Оцените справедливость суждений о неметаллах:
А. Чем больше заряд ядра атома, тем сильнее выражены его
неметаллические свойства.
Б. Чем сильнее выражены неметаллические свойства элемента, тем
более кислотный характер имеет его оксид.
1) верно только А
2) верно только Б
3) верны оба суждения
4) оба суждения неверны
56. Оцените справедливость суждений о неметаллах:
А. В периоде с увеличением зарядов атомных ядер происходит
усиление неметаллических свойств элементов.
Кочулева Людмила Рамановна, учитель химии МОБУ «Средняя общеобразовательная школа №73» г.
Оренбурга. Опорные конспекты и алгоритмы для подготовки к ЕГЭ.
Б. В главной подгруппе с увеличением зарядов атомных ядер
происходит ослабление кислотных свойств гидроксидов.
1) верно только А
2) верно только Б
3) верны оба суждения
4) оба суждения неверны
57. Кислотные свойства наиболее выражены у высшего гидроксида
1) азота 2) фосфора 3) мышьяка 4) сурьмы
58. Только восстановительные свойства азот проявляет в соединении
1) N2 2) NНз 3) NО2 4) НNОз
59. Верны ли следующие суждения о свойствах соединений элемента,
электронная конфигурация атома которого 1s22s22p6 3s2 Зр1 ?
А. Этот элемент образует гидроксид с ярко выраженными кислотными свойствами.
Б. Степень окисления этого элемента в высшем гидроксиде равна (+ 4).
1) верно только А 2) верно только Б 3) верны оба суждения
4) оба суждения неверны
60. Верны ли следующие суждения о соединениях натрия и бериллия?
А. Оксид натрия проявляет основные свойства
Б. Гидроксид бериллия проявляет амфотерные свойства
Ответы: 1-4, 2-1, 3-4, 4-2, 5-1, 6-4, 7-3, 8-1, 9-3, 10-2, 11-3, 12-2, 13-3, 14-1, 15-4, 16-3, 17-3,
18-3, 19-2, 20-3, 21-2, 22-2, 23-4, 24-1, 25-4, 26-2, 27-1, 28-4, 29-1, 30-4, 31-4, 32-3, 33-2, 342, 35-1, 36-1, 37-1, 38-3, 39-2, 40-4, 41-1, 42-4, 43-1, 44-4, 45-1, 46-4, 47-4, 48-1, 49-2, 50-3,
51-1, 52-4, 53-1, 54-3, 55-2, 56-3, 57-1, 58-2, 59-4
А4. Ковалентная химическая связь, ее разновидности и механизмы
образования. Характеристики ковалентной связи (полярность и энергия связи).
Ионная связь. Металлическая связь. Водородная связь.
1)
Химическая связь – это силы взаимодействия между атомами или группами
атомов, приводящие к образованию молекул, ионов, свободных радикалов, а также
ионных, атомных и металлических кристаллических решеток.
2)
Ковалентная связь – это связь, возникающая между атомами за счёт
образования общих электронных пар.
3)
Ковалентная связь классифицируется:
а) по механизмам образования:
o
обменный (за счёт общих электронных пар);
o
донорно-акцепторный (атом- донор обладает свободной электронной парой
и передаёт её в общее пользование с другим атомом - акцептором, у которого имеется
свободная орбиталь). Примеры: ион аммония NH4+, угарный газ СО.
б) по полярности - ковалентная полярная и неполярная.
Ковалентная неполярная связь возникает между атомами одинаковых элементов –
неметаллов. Примеры - O2, H2, N2.
Ковалентная полярная связь возникает между атомами разных элементов –
неметаллов.
Примеры – H2O, НСl, NH3, SO3.
в) по кратности – одинарные, двойные и тройные связи.
Кочулева Людмила Рамановна, учитель химии МОБУ «Средняя общеобразовательная школа №73» г.
Оренбурга. Опорные конспекты и алгоритмы для подготовки к ЕГЭ.
4)
Полярность связи – степень смещения общей электронной пары к более ЭО
атому.
5)
Длина связи – расстояние между центрами связанных атомов.
Чем больше радиусы атомы, тем длиннее связь.
6)
Энергия связи – энергия, которая необходима для разрыва связи.
Чем меньше атом, тем короче связь, тем она прочнее, тем больше требуется
энергии для её разрыва. Чем больше атом, тем больше длина связи и меньше Е связи.
Химическая связь тем прочнее, чем меньше её длина.
7)
Ионная связь – это связь, возникающая между ионами.
Примеры: оксиды, галогениды щелочных и щелочноземельных металлов, все соли
(в том числе соли аммония), все щёлочи.
Чем больше разница ЭО атомов, тем связь более ионная.
8)
Металлическая связь - связь в металлах и сплавах, которая осуществляется
совокупностью валентных электронов между атом - ионами металлов.
9)
Водородная связь – связь между атомами водорода одной молекулы (или её
части) и атомами наиболее ЭО элементов (фтор, кислород, азот) другой молекулы (или её
части).
Межмолекулярная водородная связь- вода, спирты, карбоновые кислоты,
целлюлоза, амины, аммиак, фтороводород.
Внутримолекулярная водородная связь - белки, нуклеиновые кислоты.
Тест№4
1. В аммиаке и хлориде бария химическая связь соответственно
1) ионная и ковалентная полярная
2) ковалентная полярная и ионная
3) ковалентная неполярная и металлическая
4) ковалентная неполярная и ионная
2. Вещества только с ионной связью приведены в ряду:
1) F2, ССl4, КС1
2) NaBr,Na2O,KI
3) SO2.P4.CaF2
4) H2S,Br2,K2S
3. Соединение с ионной связью образуется при взаимодействии
1) СН4 и О2
2) SO3 и Н2О
3) С2Н6 и HNO3
Кочулева Людмила Рамановна, учитель химии МОБУ «Средняя общеобразовательная школа №73» г.
Оренбурга. Опорные конспекты и алгоритмы для подготовки к ЕГЭ.
4) NH3 и HCI
4. В каком ряду все вещества имеют ковалентную полярную связь?
1) HCl,NaCl.Cl2
2) O2.H2O.CO2
3) H2O.NH3.CH4
4) NaBr.HBr.CO
5. В каком ряду записаны формулы веществ только с ковалентной полярной
связью?
1) С12, NO2, НС1
2) HBr,NO,Br2
3) H2S.H2O.Se
4) HI,H2O,PH3
6. Ковалентная неполярная связь характерна для
1) С12
2) SO3
3) СО
4) SiO2
7. Веществом с ковалентной полярной связью является
1) С12
2) NaBr
3) H2S
4) MgCl2
8. Веществом с ковалентной связью является
1) СаС12
2) MgS
3) H2S
4) NaBr
9. Вещество с ковалентной неполярной связью имеет формулу
1) NH3
2) Сu
3) H2S
4) I2
10. Веществами с неполярной ковалентной связью являются
1) вода и алмаз
2) водород и хлор
3) медь и азот
4) бром и метан
Кочулева Людмила Рамановна, учитель химии МОБУ «Средняя общеобразовательная школа №73» г.
Оренбурга. Опорные конспекты и алгоритмы для подготовки к ЕГЭ.
11. Между атомами с одинаковой относительной электроотрицательностью образуется
химическая связь
1) ионная
2) ковалентная полярная
3) ковалентная неполярная
4) водородная
12. Ковалентная полярная связь характерна для
1) KC1
2) НВг
3) Р4
4) СаСl2
13. Химический элемент, в атоме которого электроны по слоям распределены так: 2, 8, 8, 2
образует с водородом химическую связь
1)ковалентную полярную
2) ковалентную неполярную
3) ионную
4) металлическую
14. В молекуле какого вещества длина связи между атомами углерода наибольшая?
1} ацетилена
2) этана
3) этена
4) бензола
15. Тремя общими электронными парами образована ковалентная связь в молекуле
1) азота
2) сероводорода
3) метана
4) хлора
16. Водородные связи образуются между молекулами
Кочулева Людмила Рамановна, учитель химии МОБУ «Средняя общеобразовательная школа №73» г.
Оренбурга. Опорные конспекты и алгоритмы для подготовки к ЕГЭ.
1) диметилового эфира
2) метанола
3) этилена
4) этилацетата
17. Полярность связи наиболее выражена в молекуле
1) HI
2) НС1
3) HF
4) НВг
18. Веществами с неполярной ковалентной связью являются
1) вода и алмаз
2) водород и хлор
3) медь и азот
4) бром и метан
19. Водородная связь не характерна для вещества
1) Н2О
2) СН4
3) NH3
4) СНзОН
20. Ковалентная полярная связь характерна для каждого из двух веществ, формулы которых
1) KI и Н2О
2) СО2 и К2О
3) H2S и Na2S
4) CS2 и РС15
21. Наименее прочная химическая связь в молекуле
1) фтора
2) хлора
3} брома
4} иода
Кочулева Людмила Рамановна, учитель химии МОБУ «Средняя общеобразовательная школа №73» г.
Оренбурга. Опорные конспекты и алгоритмы для подготовки к ЕГЭ.
22. В молекуле какого вещества длина химической связи наибольшая?
1) фтора
2) хлора
3) брома
4) иода
23. Ковалентные связи имеет каждое из веществ, указанных в ряду:
1) C4H10, NO2, NaCl
2} СО, CuO, CH3Cl
3} BaS,C6H6,H2
4} C6H5NO2, F2, CC14
24. Ковалентную связь имеет каждое из веществ, указанных в ряду:
1) СаО,С3Н6, S8
2) Fe.NaNO3, CO
3) N2, CuCO3, K2S
4) C6H5N02, SО2, CHC13
25. Ковалентную связь имеет каждое из веществ, указанных в ряду:
1) С3Н4, NO, Na2O
2) СО, СН3С1, PBr3
3) Р2Оз, NaHSO4, Сu
4) C6H5NO2, NaF, СС14
26. Ковалентные связи имеет каждое из веществ, указанных в ряду:
1) C3Ha,NO2, NaF
2) КС1, CH3Cl, C6H12О6
3) P2O5, NaHSO4, Ba
Кочулева Людмила Рамановна, учитель химии МОБУ «Средняя общеобразовательная школа №73» г.
Оренбурга. Опорные конспекты и алгоритмы для подготовки к ЕГЭ.
4) C2H5NH2, P4, CH3OH
27. Полярность связи наиболее выражена в молекулах
1) сероводорода
2) хлора
3) фосфина
4) хлороводорода
28. В молекуле какого вещества химические связи наиболее прочные?
1)СF4
2)CCl4
3)CBr4
4)CI4
29. Среди веществ NH4Cl, CsCl, NaNO3, PH3, HNO3 - число соединений с ионной связью равно
1)
1
2)
2
3)
3
4)
4
Кочулева Людмила Рамановна, учитель химии МОБУ «Средняя общеобразовательная школа №73» г.
Оренбурга. Опорные конспекты и алгоритмы для подготовки к ЕГЭ.
30. Среди веществ (NH4)2SO4, Na2SO4, CaI2, I2, CO2 - число соединений с ковалентной связью
равно
1)
1
2)
2
3)
3
4)
4
Ответы: 1-2, 2-2, 3-4, 4-3, 5-4, 6-1, 7-3, 8-3, 9-4, 10-2, 11-3, 12-2, 13-3, 14-2, 15-1, 16-2, 17-3, 18-2, 192, 20-4, 21-4, 22-4, 23-4, 24-4, 25-2, 26-4, 27-4, 28-1, 29-3, 30-4
А5. Электроотрицательность. Степень окисления и валентность ХЭ.
1.
ЭО - это способность атома в соединении притягивать к себе электроны.
ЭО в таблице Менделеева увеличивается слева направо и снизу вверх.
2.
Степень окисления – условный заряд атома в молекуле, рассчитанный
исходя из предположения, что в молекуле существуют только ионные связи.
3.
Правила определения степени окисления элемента в соединении:
o
С.О. свободных атомов и простых веществ равна нулю.
o
Сумма степеней окисления всех атомов в сложном веществе равна нулю.
o
Металлы имеют только положительную С.О.
o
С.О. атомов щелочных металлов +1.
o
С.О. атомов щелочноземельных металлов +2.
o
С.О. атомов бора, алюминия +3.
o
С.О. атомов водорода +1 (в гидридах щелочных и щелочноземельных
металлов –1).
o
С.О. атомов кислорода –2 (исключения: в пероксидах –1, в OF2 +2).
o
С.О. атомов фтора всегда - 1.
o
Степень окисления одноатомного иона совпадает с зарядом иона.
o
Высшая (максимальная) С.О. элемента равна номеру группы.
o
Низшая (минимальная) С.О. элемента определяется по формуле: 8 - номер
группы.
Кочулева Людмила Рамановна, учитель химии МОБУ «Средняя общеобразовательная школа №73» г.
Оренбурга. Опорные конспекты и алгоритмы для подготовки к ЕГЭ.
4.
Валентность атома – это его способность образовывать определенное число
химических связей с другими атомами. Например, число черточек, отходящих от символа
элемента в структурных формулах, равно валентности этого элемента. Валентность не
имеет знака.
Валентные электроны располагаются на внешнем слое у элементов А - групп, на
внешнем слое и d – подуровне предпоследнего слоя у элементов В - групп.
Углерод С 1s2 | 2s2 p2 – валентные электроны
Железо Fe 1s2 | 2s2 p6 | 3s2 p6 d6 | 4s2 – валентные электроны
Валентные возможности атомов химических элементов определяются:
а) числом неспаренных электронов
б) наличием свободных орбиталей
в) наличием неподеленных пар электронов (связь по донорно-акцепторному
механизму – СО, ион аммония).
Число неспаренных валентных электронов в нормальном состоянии определяет
минимальную валентность, в возбужденном состоянии – промежуточные и максимальную
(высшую) валентность:
↑С
↑
↑
↓Валентность
↑С*
- II (минимальная)
↑
↑
↑
Валентность – IV (высшая)
Валентности некоторых элементов (традиционно записываются римскими
5.
цифрами).
постоянные
ХЭ
валентность
H, Na, K, Ag, F
I
Be, Mg, Ca, Ba, O, Zn
II
Al
III
6.
Формула
N2
NF3
ХЭ
Cl, Br, I
Cu, Hg
Fe
S
Mn
Cr
N
P
C, Si
переменные
валентность
I (III, V, VII)
II, I
II, III
II, IV, VI
II, IV, VII
III, VI
I-V
III, V
IV (II)
Примеры определения валентности и С.О. атомов в соединениях:
Валентности
N III
N III, F I
С.О.
0
N +3, F -1
С2 Н2
С2 Н4
С IV, Н I
С IV, Н I
С -1, Н +1
С -2, Н +1
С2 Н6
С IV, Н I
С -3, Н +1
Структурная формула вещества
N N
F
F- N- F
Н-С С-Н
Н Н
Н–С=С-Н
Н Н
Н-С–С–Н
Н Н
Кочулева Людмила Рамановна, учитель химии МОБУ «Средняя общеобразовательная школа №73» г.
Оренбурга. Опорные конспекты и алгоритмы для подготовки к ЕГЭ.
С2 Н5ОН
С IV, Н I,
О II
Первый С -1,
второй С - 3,
Н +1, О - 2
NH3
N III, Н I
N -3, Н +1
H2O2
OF2
*СО
Н I, О II
О II, F I
С III, О III
Н +1, О –1
О +2, F –1
С +2, О –2
Н Н
Н-С–С–О-Н
Н Н
Н
Н- N- Н
H—O—O—H
F—O—F
Атом
С
передал
в
общее
пользование два электрона, а более
электроотрицательный
атом
О
оттянул к себе два электрона:
При таком «раскладе сил» у С не
будет
заветной
восьмерки
электронов на внешнем уровне –
четыре своих и два общих с атомом
кислорода. Атому О придется
передать в общее пользование одну
свою свободную электронную пару,
т.е. выступить в роли донора.
Акцептором будет атом С.
1. Электроотрицательность атома – это
1) отрицательный заряд атома в молекуле
2) способность атома переходить в возбужденное состояние
3) способность атома, участвующего в химической связи, смещать к себе электронную пару,
участвующую в образовании химической связи
4) потенциал ионизации атома
2. Элементы расположены в порядке возрастания электроотрицательности в ряду
1) O, H, Br, Te
2) C, I, B, P
3) Sn, Se, Br, F
Кочулева Людмила Рамановна, учитель химии МОБУ «Средняя общеобразовательная школа №73» г.
Оренбурга. Опорные конспекты и алгоритмы для подготовки к ЕГЭ.
4) H, Br, C, B
3. Степень окисления атома – это
1) условный заряд, вычисленный из предположения, что все полярные ковалентные связи
являются ионными
2) число отданных в ходе химической реакции электронов
3) отрицательный заряд, сосредоточенный на какой-либо части молекулы
4) заряд иона в нерастворимом веществе
4. Степень окисления элемента в простом веществе равна
1) нулю
2) числу электронов во внешнем электронном слое
3) числу неспаренных электронов
4) номеру группы
5. Высшую степень окисления марганец проявляет в соединении
1) КМnО4
2) МnО2
3) К2MnО4
4) MnSO4
6. Наибольшую степень окисления марганец проявляет в соединении
1) МпС12
2) МnО
3) К2МnО4
4) МnСO3
7. Наибольшую степень окисления марганец имеет в соединении
1) MnSO4 2) МnО2
3) К2МnО4
4) Мn2Оз
8. Степень окисления - 3 фосфор проявляет в соединении
1) РН3
2) Р2Оз
3) NaH2PO4
4) Н3РО4
9. Наименьшую степень окисления сера проявляет в соединении
Кочулева Людмила Рамановна, учитель химии МОБУ «Средняя общеобразовательная школа №73» г.
Оренбурга. Опорные конспекты и алгоритмы для подготовки к ЕГЭ.
1) Na2S
2) Na2SO3
3) Na2SO4
4) SO3
10. Степень окисления - 3 фосфор проявляет в соединении
1) РН3
2) Р2Оз
3) NaH2PO4
4) НзРО4
11. Наименьшую степень окисления сера проявляет в соединении
I ) Na2S
2) Na2SO3
3) Na2SO4
4) SO3
12. Одинаковую степень окисления азот проявляет в веществах, указанных в РЯДУ:
1) N2O5, HNO3, NaNO3
2) NО2, HNO3, KNO3
3) NO, NO2, N2O3
4) HNO3,HNO2,NO2
13. В порядке увеличения электроотрицательности элементы расположены в ряду:
1) O-N-C-B
2) Si-Ge-Sn-Pb
3) Li-Na-K-Rb
4) Sb-P-S-Cl
14. Степень окисления азота увеличивается в ряду веществ:
1) NH3,NO,HNO3
2) NO,NO2,NH3
3) NH3,HNO3,NO2
4) KNO3, KNO2, NO2
15. Электроотрицательность химических элементов увеличивается в ряду:
Кочулева Людмила Рамановна, учитель химии МОБУ «Средняя общеобразовательная школа №73» г.
Оренбурга. Опорные конспекты и алгоритмы для подготовки к ЕГЭ.
1) Be,Mg,Ca
2) F,Cl,Br
3) P,S,C1
4) Cl.S.P
16. В порядке возрастания относительной электроотрицательности элементы расположены в
ряду:
1) Р, S,CI 2) N,P,As
3) O,N,C
4) Cl, Br, I
17. Из перечисленных элементов наиболее электроотрицательным является
1) азот
2) кислород
3) хлор
4) фтор
18. Степень окисления хлора в Са(С1О)2 равна
1)+1
2) +3
3) +5
4) +7
19. Степень окисления хлора в Ва(СlOз)2 равна
1) + 1
2) + 3
3) +5
4) + 7
20. Минимальную степень окисления хлор проявляет в соединении
1) NH4Cl
2) Сl2
3) Ca(OCl)2
4} NaCIO
21. Степень окисления + 3 азот проявляет в каждом из двух соединений:
1) HNO2 и NH3
2) NH4C1 и N203
3) NaNO2 и NF3
4) HNO3 и N2
Кочулева Людмила Рамановна, учитель химии МОБУ «Средняя общеобразовательная школа №73» г.
Оренбурга. Опорные конспекты и алгоритмы для подготовки к ЕГЭ.
22. В каком соединении степень окисления серы равна +4?
1) H2SO4
2) FeS
3) H2SO3
4) SO3
23. Наиболее электроотрицательным элементом является
1) кремний
2) свинец
3) олово
4) углерод
24. Азот проявляет степень окисления +3 в каждом соединении, указанном в ряду:
1) N203, HNO2, NH3
2) NH4C1, N20, NF3
3) HNO2,N2H4,N2
4) NaNO2, NF3, N2O3
25. Наиболее электроотрицательным элементом является
I) кремний
2) азот
3) фосфор
4) селен
26. В порядке возрастания электроотрицательности элементы расположены в ряду
1) H-S-C1-O-F
2) F-0-C1-S-H
3) H-CI-S-O-F
4) H-S-C1-F-O
27. Хлор проявляет положительную степень окисления в соединении с
Кочулева Людмила Рамановна, учитель химии МОБУ «Средняя общеобразовательная школа №73» г.
Оренбурга. Опорные конспекты и алгоритмы для подготовки к ЕГЭ.
1) серой
2) водородом
3) кислородом
4) железом
28. Степень окисления + 3 азот проявляет в соединении
1) NН4С1
2) NaNO3
3) N2O4
4) KNO2
29. Степень окисления + 3 хром имеет в соединении
1) СrО
2) Сr2О3
3) СrО3
4) Н2СrО4
30. Степень окисления азота в сульфате аммония равна
1) - 3
2) - 1
3) + 1
4) + 3
Ответы: 1-3, 2-3, 3-1, 4-1, 5-1, 6-3, 7-3, 8-1, 9-1, 10-1, 11-1, 12-1, 13-4, 14-1, 15-3, 16-1, 17-4, 18-1, 193, 20-1, 21-3, 22-3, 23-4, 24-4, 25-2, 26-1, 27-3, 28-4, 29-2, 30-1
А6. Вещества молекулярного и немолекулярного строения. Тип
кристаллической решётки. Зависимость свойств веществ от особенностей их
кристаллической решетки.
1)
Вещества молекулярного и немолекулярного строения.
Вещества молекулярного строения имеют молекулярную КР, немолекулярного
строения – атомную, ионную или металлическую КР.
2)
Тип кристаллической решётки. Зависимость свойств веществ от
особенностей их кристаллической решетки.
тип КР
ионная
частицы
узлах
простые
сложные
ионы
атомная
атомы
в типы ХС,
прочность
и ионная
прочная
ковалентная
очень
физические
свойства
веществ
твёрдые,
тугоплавкие,
высокие t плавления и
кипения, многие растворимы
в воде, растворы и расплавы
проводят электрический ток
большая
твёрдость,
примеры веществ
соли (в том числе соли
аммония- NH4Cl и др.),
основания (щёлочи),
некоторые
оксиды,
гидриды
алмаз, графит,
бор,
Кочулева Людмила Рамановна, учитель химии МОБУ «Средняя общеобразовательная школа №73» г.
Оренбурга. Опорные конспекты и алгоритмы для подготовки к ЕГЭ.
неполярная
прочная
молеку
лярная
молекулы
силы
слабая
полярные и межмолеку
неполярные лярного
взаимодейст
вия,
в том числе
водородные
металли
ческая
атом-ионы
металличес
кая
хрупкость, непластичность,
высокие t плавления и
кипения, в воде практически
нерастворимы
невысокая
прочность,
легкоплавкость, летучесть,
в основном растворимы в
воде, водные растворы в
основном
не
проводят
электрический ток
германий,
кремний,
карбид кремния SiС,
SiO2 (кварц, песок,
горный хрусталь)
лёд, «сухой лёд» СО2,
твёрдые HCl, H2S;
одно-, двух-, трёх-,
четырёх-,
восьмиатомные
молекулы
–
благородные газы, O2,
O3, P4, S8; многие
орган. соединения
металлический
блеск, металлы и сплавы
ковкость,
пластичность,
электро- и теплопроводность
разной
прочности
1. Молекулярное строение имеет
1) С12
2) СаО
3) ZnCl2
4) NaBr
2. Кристаллическая решетка хлорида кальция
1) металлическая
2) молекулярная
3) ионная
4) атомная
3. Кристаллическая решетка твердого оксида углерода (IV)
1) ионная
2) молекулярная
3) металлическая
4) атомная
4. Молекулярную кристаллическую решетку имеет
1) CaF2
2) СО2
3) SiO2
4) A1F3
Кочулева Людмила Рамановна, учитель химии МОБУ «Средняя общеобразовательная школа №73» г.
Оренбурга. Опорные конспекты и алгоритмы для подготовки к ЕГЭ.
5. Немолекулярное строение имеет
1) Н2О
2) H2SO4
3) SiО2
4) СО2
6. Молекулярное строение имеет
1) алмаз
2) азот
3) кремний
4) поваренная соль
7. Немолекулярное строение имеет
1) азот
2) графит
3) аммиак
4) кислород
8. Наибольшую температуру плавления имеет
1) водород
2) кислород
3) оксид углерода (IV)
4) оксид кремния (IV)
9. Ионное строение имеет
1) оксид бора
2) оксид углерода (IV)
3) оксид серы (VI)
4) оксид магния
10. Вещества с металлической кристаллической решеткой
Кочулева Людмила Рамановна, учитель химии МОБУ «Средняя общеобразовательная школа №73» г.
Оренбурга. Опорные конспекты и алгоритмы для подготовки к ЕГЭ.
1) хрупкие, легкоплавкие
2) проводят электрический ток, пластичные
3) обладают низкой тепло- и электропроводностью
4) обладают хорошими оптическими свойствами
11. Немолекулярное строение имеет каждое из двух веществ:
1) СО2иСl2 2) Fe и NaCl
3) СО и Mg
4) Na2CO3 и I2 (тв)
12. Вещества твердые, прочные, с высокой температурой плавления, расплавы которых проводят
электрический ток, имеют кристаллическую решетку
1) металлическую
2) молекулярную
3) атомную
4) ионную
13. Молекулярную кристаллическую решетку имеет
1) кремний
2) оксид углерода (IV)
3) оксид кремния
4) нитрат аммония
14. Молекулярная кристаллическая решетка характерна для каждого из веществ,
расположенных в ряду:
1) хлорид калия, азот, метан
2) иод, диоксид углерода, гелий
3) алюминий, бром, алмаз
Кочулева Людмила Рамановна, учитель химии МОБУ «Средняя общеобразовательная школа №73» г.
Оренбурга. Опорные конспекты и алгоритмы для подготовки к ЕГЭ.
4) водород, сульфат магния, оксид железа (Ш)
15. Ионную
ряду:
кристаллическую решетку имеет каждое из
веществ, расположенных в
1) натрий, хлорид натрия, гидрид натрия
2) кальций, оксид кальция, карбонат кальция
3) бромид натрия, сульфат калия, хлорид железа (II)
4) фосфат магния, хлорид калия, оксид фосфора (V)
16. Молекулярное строение имеет
1) СO2
2) КВг
3) MgSО4
4) SiO2
17. Ионы являются структурными частицами
1) кислорода
2) воды
3) оксида углерода (IV)
4) хлорида натрия
18. Металлическую кристаллическую решетку имеет
1) малахит
2) бронза
3) кремнезем
4) графит
19.
Кристаллическая решетка брома
1) молекулярная
Кочулева Людмила Рамановна, учитель химии МОБУ «Средняя общеобразовательная школа №73» г.
Оренбурга. Опорные конспекты и алгоритмы для подготовки к ЕГЭ.
2) металлическая
3) ионная
4) атомная
20. Верны ли следующие суждения о зависимости свойств веществ от особенностей их
кристаллической решетки?
А. Расплавы веществ с ионной кристаллической решеткой проводят электрический ток.
Б. Алмаз и графит имеют атомную кристаллическую решетку.
1) верно только А
2) верно только Б
3) верны оба суждения
4) оба суждения неверны
21. Немолекулярное строение имеет
1)
H2O
2)
NH3
3)
SiO2
4)
CO2
22. Ионы являются структурной единицей для каждого из двух веществ:
1) СН4 и I2
2) SO, и Н2О
3) Сl2 и NH3
4) LiF и KCl
23. Молекулярное строение имеет каждое из двух веществ:
1) NН4С1 и CH3NH3
Кочулева Людмила Рамановна, учитель химии МОБУ «Средняя общеобразовательная школа №73» г.
Оренбурга. Опорные конспекты и алгоритмы для подготовки к ЕГЭ.
2) Na2CO3 и HNO3
3) C2H5OH и СН4
4} H2S и CH3COONa
24. Молекулярную кристаллическую решетку имеет
1) фторид кальция
2) бромид алюминия
3) сероводород
4) хлорид меди (П)
25. Молекулярное строение имеет
1) С2Н5ОН
2) А1
3) Fe2(SO4)3
4) КСЮ3
26. Вещества только немолекулярного строения приведены в ряду
1) S8, O2(г), лед
2) Fe, NaCl (тв), алмаз
3) СО2 (г), N2 (г), А1
4) графит, Na2CO3 (тв), I2
27. Утверждение о том, что структурной частицей данного вещества является молекула,
справедливо только для
1)
алмаза
2)
поваренной соли
3)
кремния
Кочулева Людмила Рамановна, учитель химии МОБУ «Средняя общеобразовательная школа №73» г.
Оренбурга. Опорные конспекты и алгоритмы для подготовки к ЕГЭ.
4)
азота
28. Кристаллическая решетка хлорида кальция
1)
ионная
2)
молекулярная
3)
металлическая
4)
атомная
29. Кристаллическая решетка твердого оксида углерода (IV)
1)
ионная
2)
молекулярная
3)
металлическая
4)
атомная
Кочулева Людмила Рамановна, учитель химии МОБУ «Средняя общеобразовательная школа №73» г.
Оренбурга. Опорные конспекты и алгоритмы для подготовки к ЕГЭ.
30. Молекулярную кристаллическую решетку имеет
1)
CaF2
2)
SO2
3)
SiO2
4)
AlF3
Ответы: 1-1, 2-3, 3-2, 4-2, 5-3, 6-2, 7-2, 8-4, 9-4, 10-2, 11-2, 12-1, 13-2, 14-2, 15-3, 16-1, 17-4, 18-2, 191, 20-3, 21-3, 22-4, 23-3, 24-3, 25-1, 26-2, 27-4, 28-1, 29-2, 30-2
А7.
В1.
Классификация
неорганических
веществ.
Номенклатура
неорганических веществ (тривиальная и международная).
Классификация органических веществ. Номенклатура органических веществ
(тривиальная и международная).
1)
Классификация неорганических веществ.
Кочулева Людмила Рамановна, учитель химии МОБУ «Средняя общеобразовательная школа №73» г.
Оренбурга. Опорные конспекты и алгоритмы для подготовки к ЕГЭ.
Оксиды – соединения элементов с кислородом, степень окисления кислорода в
оксидах всегда равна -2.
Оснóвные оксиды образуют типичные металлы со С.О. +1,+2 (Li2O, MgO,
СаО,CuO и др.).
Кислотные оксиды образуют неметаллы со С.О. более +2 и металлы со С.О. от +5
до +7 (SO2, SeO2, Р2O5, As2O3, СO2,SiO2 , CrO3 и Mn2O7). Исключение: у оксидов NO2 и
ClO2 нет соответствующих кислотных гидроксидов, но их считают кислотными.
Амфотерные оксиды образованы амфотерными металлами со С.О. +2,+3,+4 (BeO,
Cr2O3, ZnO, Al2O3, GeO2, SnO2 и РЬО).
Несолеобразующие оксиды – оксиды неметаллов со С.О.+1,+2 (СО, NO, N2O,
SiO).
Основания (осно́вные гидрокси́ды) — сложные вещества, которые состоят из иона
металла (или иона аммония) и гидроксогруппы (-OH).
Кочулева Людмила Рамановна, учитель химии МОБУ «Средняя общеобразовательная школа №73» г.
Оренбурга. Опорные конспекты и алгоритмы для подготовки к ЕГЭ.
Кислотные гидроксиды (кислоты) - сложные вещества, которые состоят из
атомов водорода и кислотного остатка.
Амфотерные гидроксиды образованы элементами с амфотерными свойствами.
Соли – сложные вещества, образованные атомами металлов, соединёнными с
кислотными остатками.
Средние (нормальные) соли — все атомы водорода в молекулах кислоты
замещены на атомы металла.
Кислые соли — атомы водорода в кислоте замещены атомами металла частично.
Они получаются при нейтрализации основания избытком кислоты. Чтобы правильно
назвать кислую соль, необходимо к названию нормальной соли прибавить приставку
гидро- или дигидро- в зависимости от числа атомов водорода, входящих в состав кислой
соли.
Например, KHCO3 – гидрокарбонат калия, КH2PO4 – дигидроортофосфат калия
Нужно помнить, что кислые соли могут образовывать двух и более основные
кислоты, как кислородсодержащие, так и бескислородные кислоты.
Осно́вные соли — гидроксогруппы основания (OH−) частично замещены
кислотными остатками. Чтобы назвать основную соль, необходимо к названию
нормальной соли прибавить приставку гидроксо- или дигидроксо- в зависимости от числа
ОН - групп, входящих в состав соли.
Например, (CuOH)2CO3 - гидроксокарбонат меди (II).
Нужно помнить, что основные соли способны образовывать лишь основания,
содержащие в своём составе две и более гидроксогрупп.
Двойные соли — в их составе присутствует два различных катиона, получаются
кристаллизацией из смешанного раствора солей с разными катионами, но одинаковыми
анионами.
Смешанные соли — в их составе присутствует два различных аниона.
Гидратные соли (кристаллогидраты) — в их состав входят молекулы
кристаллизационной воды. Пример: Na2SO4 ·10H2O.
2.Номенклатура неорганических веществ.
1. Названия соединений, состоящих из двух элементов, образуются по простому
правилу: к названию аниона (или электроотрицательной части молекулы) добавляется
суффикс –ид, например:
1) Оксиды – сложные вещества из двух элементов, один из которых кислород со
степенью окисления -2.
2) Гидриды - сложные вещества из двух элементов, один из которых водород.
3) Нитриды - сложные вещества из двух элементов, один из которых азот.
4) Фосфиды - сложные вещества из двух элементов, один из которых фосфор.
5) Карбиды - сложные вещества из двух элементов, один из которых углерод.
Если вы усвоили этот принцип, вы легко поймете, что такое сульфиды, хлориды и
т.д. 2. Обратите внимание, что во всех этих соединениях элемент, название которого
имеет суффикс –ид, находится в низшей степени окисления, которая рассчитывается так:
номер группы минус восемь.
Например, в нитридах азот всегда имеет степень окисления «-3».
3. В современной номенклатуре неорганических веществ число атомов каждого
элемента в молекуле указывается греческим числительным, например, N2O4 – тетраоксид
диазота, NO2 – диоксид (моно) азота (приставка моно- может не употребляться).
4. В названиях бинарных соединений неионной природы сначала указывается
более электроотрицательный элемент (элемент, стоящий в периодической системе правее,
а в группах выше), в формуле они записываются на втором месте.
Кочулева Людмила Рамановна, учитель химии МОБУ «Средняя общеобразовательная школа №73» г.
Оренбурга. Опорные конспекты и алгоритмы для подготовки к ЕГЭ.
Например,ClF – монофторид хлора, OF2 – дифторид кислорода.
5. Названия одинаковых по заряду частиц пишутся через черточку,
например, MgIBr – бромид-иодид магния, BiClO – оксид-хлорид висмута, SO2Cl2 –
диоксид-дихлорид серы.
6.Название химических соединений начинается с последней группы формулы,
например, сульфид калия – K2S, гидросульфид натрия – NaHS.
7.В соответствие с номенклатурой неорганических соединений, рекомендованной
ИЮПАК, в начале формулы ставится положительно заряженная группа, а в конце –
имеющая отрицательный заряд. В русском языке в названии сначала указывается анион, а
затем в родительном падеже катион,
например, хлорид натрия (NaCl), сульфат калия (K2SO4) и т.д.
8. Чтобы правильно написать формулу надо помнить, что молекулы нейтральны,
т.е. суммарный заряд элементов, входящих в них, должен быть равен нулю.
Например, построим нитрида кальция: Ca2+ N3- → Ca3N2, вы видите, здесь
сработало правило креста: тройка стала индексом кальция, а двойка индексом азота.
Название этого соединения – нитрид кальция или динитрид трикальция.
9.Приставка перокси- означает, что соединении имеется пероксогруппа (-О-О-),
например, пероксид натрия Na2O2.
10. Греческие числительные, применяемые в названиях таковы: «геми» - 0,5,
«моно» - 1, «сескви» - 1,5, «ди» - 2, «три» - 3, «тетра» - 4, «пента» - 5, «гекса» - 6 и т.д.
11. Группы в квадратных скобках пишутся одним словом.
Например, тетраиодидмеркурат (II) калия K2[HgI4]. Обратите внимание на то, что
сначала указывается число анионов-лигандов, потом название аниона, и только после
этого название центрального иона.
12. В тех случаях, когда у элемента в соединениях возможны несколько степеней
окисления, после названия этого элемента указывают его степень окисления римскими
цифрами в круглых скобках,
например, FeCl3 – хлорид железа(III) или трихлорид моножелеза, Cr2O3 – оксид
хрома (III) или триоксид дихрома.
13. Цифра может не указываться, если у элемента степень окисления во всех
соединениях постоянна,
например AlCl3 – хлорид алюминия или трихлорид алюминия.
Как строить названия кислот и солей легко понять из ниже приведенной
таблицы.
14.Изменение степени окисления элемента, образующего кислоту, приводит к
изменениям суффикса кислоты и соли, образованной ею.
15. Корнем в названии кислоты и ее соли является корень латинского названия
элемента. В таблице указаны приставки и суффиксы кислот и их солей в зависимости от
степени окисления элемента, образовавшего их.
В ниже приведенной таблице степень окисления элемента уменьшается сверху
вниз.
Номенклатура кислот и кислотных остатков
№ группы
Суффикс
кислоты
II
- ная
-вая
H2RO2
III
ортоH3RO3
мета-
IV
ортоH4RO4
мета-
V
ортоH3RO4
мета-
VI
H2RO4
VII
HRO4
Суффикс
кислотного
остатка
пер- (VII)
-ат
Кочулева Людмила Рамановна, учитель химии МОБУ «Средняя общеобразовательная школа №73» г.
Оренбурга. Опорные конспекты и алгоритмы для подготовки к ЕГЭ.
HRO2
H2RO3
HRO3
-оватая
-истая
ортоH3RO3
метаHRO2
-оватистая
ортоH3RO2
метаHRO
H3R
-водородная
H2RO3
H2R
HRO3
-ат
HRO2
-ит
HRO
гипо-ит
HR
-ид
Примечания:
1.
Элементы второго периода не имеют орто-кислот, приставка мета- в их
названиях не применяется. Пример: HNO3 – азотная кислота.
2.
Марганцеватая кислота имеет формулу H2MnO4 и образует соли –
манганаты (например: Na2MnO4 – манганат натрия (VI)).
Азотноватистая кислота H2N2O2 имеет соли гипонитриты, например,
Na2N2O2 –
гипонитрит натрия.
3.
Приставка гидро- указывает сколько атомов Н имеется в кислой соли.
Например, KHCO3 – гидрокарбонат калия, KH2PO4 – дигидрофосфат калия.
4.
Приставка гидроксо- указывает сколько групп OH- имеется в соли.
Например, BaOHCl – гидроксохлорид бария, K[Al(OH)4] – тетрагидроксоалюминат калия.
В основаниях и в основных солях возможно применение отдельного слова – гидроксид.
Например, гидроксид калия KOH или гидроксид-хлорид кальция CaClOH.
5.
Приставки ди-, би-, пиро- показывают, что надо удвоить формулу кислоты,
отнять от нее одну молекулу воды и заменить атомы водорода на соответствующий
катион:
*2
H 2O
K
Пирофосфат калия: H3PO4 
H6P2O8 
K4P2O7
 H4P2O7 
*2
 H 2O
K
Дихромат калия: H2CrO4  H4Cr2O8  H2Cr2O7  K2Cr2O7
а)
Приставка тетра- указывает, что формулу кислоты надо не удвоить, а
учетверить, остальной алгоритм остается без изменения. Например, тетраборат калия
H 2O
*4
K
HBO2 
H4B4O8 
K2B4O7
 H2B4O7 
б)
Исключения из этого правила: бикарбонат – HCO3-, бисульфит – HSO3-,
бисульфат – HSO4-, H2PO4- - однозамещенный фосфат, HPO42- - двузамещенный фосфат
(это связано с традицией устаревшей номенклатуры).
6.
Приставка тио- указывает сколько атомов кислорода замешено на атом
серы в кислородной кислоте, например, тиофосфат кальция – Ca3(PSO3)2.
7.
Приставка циано- указывает сколько групп CN- имеется в комплексной
соли, например, гексацианоферрат (III) калия – K3[Fe(CN)6]. В некомплексных солях эта
частица имеет название – цианид.
Кочулева Людмила Рамановна, учитель химии МОБУ «Средняя общеобразовательная школа №73» г.
Оренбурга. Опорные конспекты и алгоритмы для подготовки к ЕГЭ.
8.
Слово гидрат указывает, сколько молекул воды имеется в молекуле
кристаллогидрата. Например, пентагидрат сульфата меди (II) CuSO4 * 5H2O (пятиводный
сульфат меди (II)).
9.
Названия некоторых многоатомных ионов, не вошедших в эту таблицу: O22- пероксид, O2- - надпероксид, O3- - озонид, CN- - цианид, CN22- - цианамид, N3- - азид,
NH2- - амид, NCS- - тиоцианид (роданид).
Алгоритм написания формулы по названию
1.Провести морфологический разбор слов, означающих анион (первое слово
названия) и катион (второе слово названия в родительном падеже).
2.Корень слова аниона отождествляется с элементом, место которого отыскивается
в периодической системе.
3.Находится форма кислоты в выше приведенной таблице.
4.Кислота преобразуется в соль, по вышеприведенным правилам.
Примеры заданий
1. Написать формулу гидроортоалюмината цинка.

1.Выделим части первого слова: гидроорто алюмин ат . Корень – алюмин,
следовательно, мы должны найти в таблице Д.И.Менделеева алюминий, посмотреть в
таблицу номенклатуры в графу III группы, в первой строке, которая соответствует
суффиксу солей – ат.
2. Форма ортокислоты будет H3AlO3.
3.Приставка гидро указывает, что в молекуле остался только один протон, а
остальные два замещены цинком, следовательно, получим – ZnHAlO3.
2. Написать формулу тиоцинката алюминия.

1. Разберем части первого слова: тио цинк ат , корень – цинк, цинк находится во II
группе таблицы Менделеева, суффикс – ат,
2. Следовательно, форма кислоты будет H2ZnO2.
3. Замещаем водород на алюминий. Получаем AlZnO2.
4. Мы знаем, что алюминий в своих соединениях всегда имеет степень окисления
3+, ион цинката должен иметь заряд 2-, т.е. необходимо записать Al3+ ZnO22-.
5. Воспользуемся правилом креста и получим Al2(ZnO2)3.
6. Заменим кислород на серу: Al2(ZnSO)3.
Алгоритмы построения названия по формуле соединения
1.Выделить катион и кислотный остаток в соединении.
2.В таблице номенклатуры отыскать формулу данного кислотного остатка, назвать
его по выше приведенным правилам.
3.Построить название соединения так, чтобы название аниона было на первом
месте, а название катиона на втором в родительном падеже.
Пример
Назвать соединение Fe(OH)2Cl.
1. Это соединение состоит из ионов Fe3+ и (OH)2Cl3-.
Кочулева Людмила Рамановна, учитель химии МОБУ «Средняя общеобразовательная школа №73» г.
Оренбурга. Опорные конспекты и алгоритмы для подготовки к ЕГЭ.
2. Мы видим, что кислотный остаток содержит две группы OH, следовательно
соединение должно иметь приставку дигидроксо-;
3. Сl не соединен с кислородом, следовательно, он имеет суффикс –ид (нижняя
строка таблицы номенклатуры).
4. В результате получаем название: дигидроксохлорид железа (III)
или
дигидроксид-хлорид железа (III).
Тривиальные названия часто употребляемых неорганических веществ:
Название
Белый фосфор
Боксит
Бурый газ
Гипс (жженый)
Глауберова соль
Доломит
Едкий натр
Едкое кали
Известковая вода (молоко)
Известняк, мел, мрамор
Каустическая сода
Кварц, кварцевый песок
Ляпис
Нашатырный спирт
Олеум
Пирит,
серный
колчедан,
железный
колчедан
Поташ
Сернистый ангидрид, сернистый газ
Химическая формула
P4
Al2O3
2O
NO2
CaSO4
2
2O)
Na2SO4
O
2
CaCO3
3
NaOH
KOH
Ca(OH)2 + H2O
CaCO3
NaOH
SiO2
AgNO3
NH3
2O
SO3 + H2SO4
FeS2
K2CO3
SO2
2) Классификация органических соединений по ФГ.
СХНУ (нет ФГ)
О-содержащие
N-содержащие Галогенопроизводные
алканы
спирты (одно- и
нитросоединения
многоатомные)
алкены
фенолы
амины
алкины
альдегиды
АМК
циклоалканы
кетоны
белки
алкадиены
карбоновые кислоты
арены
эфиры (простые и
сложные)
жиры
мыла
углеводы
Кочулева Людмила Рамановна, учитель химии МОБУ «Средняя общеобразовательная школа №73» г.
Оренбурга. Опорные конспекты и алгоритмы для подготовки к ЕГЭ.
3) Классификация органических соединений по строению углеродного скелета
4)
Номенклатура органических веществ (тривиальная и международная)
Название по тривиальной
номенклатуре
Хлороформ
Дивинил
Изопрен
Название по международной
номенклатуре
трихлорметан
бутадиен - 1,3
2- метилбутадиен - 1,3
Формула
СНСl3
СН2=СН- СН=СН2
СН2= С- СН=СН2
Ксилол
Стирол
Муравьиный альдегид
(формальдегид)
Уксусный альдегид
(ацетальдегид)
Метиловый (древесный) спирт
Этиловый (винный) спирт
Этиленгликоль
диметилбензол
винилбензол
метаналь
СН3
С6Н4(СН3)2
С6Н5- СН=СН2
НСНО
этаналь
СН3СНО
метанол
этанол
этандиол-1,2
СН3- ОН
СН3- СН2 - ОН
СН2- СН2
Глицерин
пропантриол-1, 2, 3
ОН ОН
СН2-СН- СН2
Муравьиная кислота
Уксусная кислота
Пропионовая кислота
Пальмитиновая кислота
метановая
этановая
пропановая
гексадекановая
ОН ОН ОН
НСООН
СН3- СООН
СН3- СН2 - СООН
СН3- (СН2)14- СООН
Кочулева Людмила Рамановна, учитель химии МОБУ «Средняя общеобразовательная школа №73» г.
Оренбурга. Опорные конспекты и алгоритмы для подготовки к ЕГЭ.
Стеариновая кислота
Карболовая кислота
Толуол
Анилин
октадекановая
фенол
метилбензол
фениламин
СН3- (СН2)16- СООН
С6Н5- ОН
С6Н5- СН3
С6Н5-NН2
1. Среди перечисленных веществ
А) FeO
Б) CaO
В) А12Оз
Г) К2О
Д) CO2
Е) NO
основными оксидами являются
1) АГД
2) АВЕ
3) БДЕ
4) АБГ
2. Среди перечисленных элементов
А) стронций
Б) марганец
В) кальций
Г) магний
Д) сера
Е) хром
могут образовать кислотный оксид
1) АГД
2) АВЕ
Кочулева Людмила Рамановна, учитель химии МОБУ «Средняя общеобразовательная школа №73» г.
Оренбурга. Опорные конспекты и алгоритмы для подготовки к ЕГЭ.
3) БДЕ
4) АБД
3. Среди перечисленных оксидов
А) СО2,
Б) Mn2O7,
В) SO2
Г) Na2O,
Д) Сг2Оз
Е) CrO
относятся к кислотным оксидам
1) АГД
2) АВЕ
3) БДЕ
4) АБВ
4. Среди перечисленных оксидов
А) Na2O
Б) MgO
В) Al2O3
Г) SO2
Д) CO2,
Е) SiO2
относятся к кислотным оксидам
1) АГД
2) АВЕ
Кочулева Людмила Рамановна, учитель химии МОБУ «Средняя общеобразовательная школа №73» г.
Оренбурга. Опорные конспекты и алгоритмы для подготовки к ЕГЭ.
3) БДЕ
4) ГДЕ
5. Среди перечисленных веществ
А) Н2С2О4,
Б) HCN,
В) H2S
Г) К2SO3,
Д) NaHSO4
Е) KNO3
к кислотам относятся
1) АГД
2) АВЕ
3) БДЕ
4) АБВ
6. Среди перечисленных оксидов
А) SeO3,
Б) SiO2,
В) Cl2O7
Г) CO
Д) NO
Е) N2O
солеобразующими являются
1) АГД
Кочулева Людмила Рамановна, учитель химии МОБУ «Средняя общеобразовательная школа №73» г.
Оренбурга. Опорные конспекты и алгоритмы для подготовки к ЕГЭ.
2) АВЕ
3) БДЕ
4) АБВ
7. Среди перечисленных веществ
А) NaHCO3
Б) HCOOK
В)(NH4)2SO4
Г) KHSO3
Д) Na2HPO4
Е) Na3PO4
кислыми солями являются
1) АГД
2) АВЕ
3) БДЕ
4) АБВ
8. Среди перечисленных веществ
А) H2SO4,
Б) HC1
В) HNO3
Г) Н3РО4,
Д) H2S
Е) HCI
гидроксидами неметаллов являются
1) АГД
Кочулева Людмила Рамановна, учитель химии МОБУ «Средняя общеобразовательная школа №73» г.
Оренбурга. Опорные конспекты и алгоритмы для подготовки к ЕГЭ.
2) АВЕ
3) БДЕ
4) АВГ
9. Среди перечисленных веществ
А) А12О3,
Б) MgO,
В) Na2O
Г) N2O
Д) CuO
Е) ZnO
к основным оксидам относятся
1) АГД
2) АВЕ
3) БВД
4) АВД
10. Среди перечисленных веществ
А) СгО3
Б) СаО
В) А12О3
Г) NO
Д) SiO2
Е) Mn2O7
к кислотным оксидам относятся
Кочулева Людмила Рамановна, учитель химии МОБУ «Средняя общеобразовательная школа №73» г.
Оренбурга. Опорные конспекты и алгоритмы для подготовки к ЕГЭ.
1) АГД
2) АВЕ
3) БВД
4) АДЕ
11. Среди перечисленных веществ
А) Н2СОз
Б) НС1O4
В) H2SO3
Г) HMnO4
Д) H2ZnO2
Е) H2СrO4
к двухосновным кислотам относятся
1) АГД
2) АВЕ
3) БВД
4) АДЕ
12. Среди перечисленных веществ
А) НNО3
Б) КНСОз
В) HNO2
Г) H2S
Д) Na2SO3
Е) CH3COOH
Кочулева Людмила Рамановна, учитель химии МОБУ «Средняя общеобразовательная школа №73» г.
Оренбурга. Опорные конспекты и алгоритмы для подготовки к ЕГЭ.
к кислотам относятся
1) АГД
2) АВЕ
3) БВД
4) АДЕ
13. Среди перечисленных веществ
А) N2O5
Б) NO2
В) N2O3
Г) NO
Д) СО
Е) N2O
к несолеобразующим оксидам относятся
1) АГД
2) АВЕ
3) БВД
4) ГДЕ
14. Среди перечисленных оксидов
А) оксид серы (IV)
Б) оксид алюминия
В) оксид лития
Г) оксид фосфора (V)
Кочулева Людмила Рамановна, учитель химии МОБУ «Средняя общеобразовательная школа №73» г.
Оренбурга. Опорные конспекты и алгоритмы для подготовки к ЕГЭ.
Д) оксид цинка
Е) оксид железа (III)
к амфотерным оксидам относятся
1) АГД
2) АВЕ
3) БДЕ
4) ГДЕ
15. Среди перечисленных солей
А) Mn(NO3)2
Б) Mg(H2P04)2
В) A12(SO4)3
Г) (NH4)2HP04
Д) NaHSO3
Е) (NH4)2S
к средним солям относятся
1) АГД
2) АВЕ
3) БВД
4) ГДЕ
16. Среди перечисленных веществ
А) С3Н6
Б) С2Н4
В) СН4
Г) С6Н6
Кочулева Людмила Рамановна, учитель химии МОБУ «Средняя общеобразовательная школа №73» г.
Оренбурга. Опорные конспекты и алгоритмы для подготовки к ЕГЭ.
Д) С3Н8
Е) С2Н6
к алканам относятся
1) АГД
2) АВЕ
3) БВД
4) ВДЕ
17. Среди перечисленных солей
А) Ag2CO3
Б) NaHS
В) Cu(NO3)2
Г) Fe2(SO4)3
Д) Ca(HCO3)2
Е) KH2PO4
к кислым солям относятся
1) АГД
2) АВЕ
3) БДЕ
4) ГДЕ
18. Среди перечисленных веществ
А) анилин
Б) стирол
В) глицин
Кочулева Людмила Рамановна, учитель химии МОБУ «Средняя общеобразовательная школа №73» г.
Оренбурга. Опорные конспекты и алгоритмы для подготовки к ЕГЭ.
Г) аланин
Д) фенилаланин
Е) фенол
к аминокислотам относятся
1) АГД
2) АВЕ
3) БДЕ
4) ВГД
19. Среди перечисленных веществ
А) глюкоза
Б) бутанол-1
В) глицин
Г) муравьиная кислота
Д) гидроксид цинка
Е) гидроксид хрома (III)
к амфотерным соединениям относятся
1) АГД
2) АВЕ
3) БДЕ
4) ВДЕ
20. Среди перечисленных веществ
А) Сa(OH)2
Б) Fe(OH)3
В) Ве(ОН)2
Кочулева Людмила Рамановна, учитель химии МОБУ «Средняя общеобразовательная школа №73» г.
Оренбурга. Опорные конспекты и алгоритмы для подготовки к ЕГЭ.
Г) КОН
Д) Zn(OH)2
Е) Ва(ОН)2
к амфотерным гидроксидам относятся
1) АБЕ
2) БВД
3) АГЕ
4) БДЕ
Ответы: 1-4, 2-3, 3-4, 4-4, 5-4, 6-4, 7-1, 8-4, 9-3, 10-4, 11-2, 12-2, 13-4, 14-3, 15-2, 16-4, 17-3, 18-4, 194, 20-2.
А9. Характерные химические свойства оксидов (основных, амфотерных,
кислотных)
Основные оксиды
1. Основный оксид + кислота → соль + вода
CuO + H2SO4 → CuSO4 + H2O
Примечание: кислота сильная, при нагревании, образуется растворимая соль.
2. Сильноосновный оксид (металла I, II А- групп) + вода → щелочь
CaO + H2O → Ca(OH)2
3. Сильноосновный оксид + кислотный оксид → соль
Na2O + CO2 → Na2CO3
CaO + Mn2O7 → Ca(MnO4)2
4. Основный оксид (металов В-групп) + водород → металл + вода
CuO + H2 → Cu + H2O
Примечание: при нагревании; металл менее активный, чем алюминий.
Кислотные оксиды
1. Кислотный оксид + вода → кислота
SO3 + H2O → H2SO4
Некоторые оксиды, например SiO2, с водой не реагируют, поэтому их кислоты
получают косвенным путём.
2. Кислотный оксид + основный оксид → соль
CO2 + CaO → CaCO3
3. Кислотный оксид + щелочь → соль + вода
SO2 + 2NaOH → Na2SO3 + H2O
Если кислотному оксиду соответствует многоосновная кислота, возможно
образование кислых или средних солей:
Ca(OH)2 + CO2 → CaCO3↓ + H2O
CaCO3 + CO2 + H2O → Ca(HCO3)2
4. Нелетучий оксид + соль1 → соль2 + летучий оксид
SiO2 + Na2CO3 → Na2SiO3 + CO2↑
Кочулева Людмила Рамановна, учитель химии МОБУ «Средняя общеобразовательная школа №73» г.
Оренбурга. Опорные конспекты и алгоритмы для подготовки к ЕГЭ.
Амфотерные оксиды
При взаимодействии с сильной кислотой или кислотным оксидом проявляют
основные свойства:
ZnO + 2HCl → ZnCl2 + H2O
При взаимодействии с сильным основанием или основным оксидом проявляют
кислотные свойства:
ZnO + 2KOH + H2O → K2[Zn(OH)4] (в водном растворе)
ZnO + CaO → CaZnO2 (при сплавлении)
В воде не растворяются.
1. Оксид серы (VI) взаимодействует с каждым из двух веществ:
1) вода и соляная кислота
2) кислород и оксид магния
3) вода и медь
4) оксид кальция и гидроксид натрия
2. Оксид углерода (IV) реагирует с каждым из двух веществ:
1) гидроксидом натрия и оксидом кальция
2) оксидом кальция и оксидом серы (IV)
3) кислородом и водой
4) хлоридом натрия и оксидом азота (IV)
3. Оксид серы (IV) взаимодействует с
1) СО2
2) Н2О
3) Na2SO4
4) НС1
4. Диоксид углерода в водном растворе реагирует с каждым из двух веществ:
1) Н2О, КС1
2) Ba(OH)2,SiO2
3) K2SO4,BaCO3
4) СаСОз, КОН
5. Способны взаимодействовать между собой
1) SiO2 и Н2О
2) СО2 и H2SO4
3) CO2 и Са(ОН)2
4) Na2O и Са(ОН)2
6. Ни с водой, ни с раствором гидроксида натрия не реагирует
1) SiО2
2) SO3
3) ВаО
4) NО
7. Реагирует с соляной кислотой, но не с водой, оксид
1) SiО2
2) N2O3
3) Na2О
4) Fе20з
8. Между собой взаимодействуют
1) NO и А12Оз 2) СО и ВаО 3) Р2О5 и SCl4
4) ВаО и SO2
9. Между собой взаимодействуют
1) СuО и FeO 2) СО2 и ВаО 3) Р2О5 и NO 4) СгО3 и SO3
10. Реакция возможна между
1) Н2О и А12О3 2) СО и СаО
3) Р2О3 и SO2
4) Н2О и ВаО
11. И с раствором гидроксида натрия, и с соляной кислотой реагирует оксид
1) SiО2
2) AI2O3
3) СО2
4) MgO
12. Реакция возможна между
1) ВаО и NH3
2) А12О3 и Н2О
3) Р2О5 и НС1 4) MgO и SO3
13. Оксид натрия не взаимодействует с
1) Н2О
2) СО2
3) CaO
4) А12О3
14. Оксид углерода (IV) реагирует с каждым из двух веществ:
1) водой и оксидом кальция
2) кислородом и водой
3) сульфатом калия и гидроксидом натрия
4) оксидом кремния (IV) и водородом
15. Основные свойства наиболее выражены у оксида, формула которого
Кочулева Людмила Рамановна, учитель химии МОБУ «Средняя общеобразовательная школа №73» г.
Оренбурга. Опорные конспекты и алгоритмы для подготовки к ЕГЭ.
1) Fe2O3
2) FeO
3) Cr2O3
4) СгО3
16. Какие из двух оксидов могут взаимодействовать между собой?
1) СаО и СгО 2) СаО и NO 3) К2O и СО2 4) SiO2 и SO2
17. Реакция возможна между:
1) Н2О и ВаО 2) СО и СаО 3) Р2O3 и SO2 4) Н2О и А12О3
18. Верны ли следующие суждения о свойствах оксидов алюминия и хрома (III)?
А. Эти оксиды проявляют амфотерные свойства.
Б. В результате взаимодействия этих оксидов с водой получаются гидроксиды.
1) верно только А 2) верно только Б 3) верны оба суждения 4) оба суждения неверны
19. Верны ли следующие суждения об оксидах цинка и алюминия?
А. В результате взаимодействия этих оксидов с водой получаются гидроксиды.
Б. Эти оксиды взаимодействуют как с кислотами, так и со щелочами.
1) верно толь ко А2) верно только Б3) верны оба суждения4) оба суждения неверны
20. Оксид фосфора (V) взаимодействует с каждым из двух веществ:
1) Fe и NaOH2) NaOH и H2О3) Н2О и НС1 4) Н2 и КС1
21. Оксид фосфора (V)1) не проявляет кислотно-основных свойств
2) проявляет только основные свойства
3) проявляет только кислотные свойства
4) проявляет как основные, так и кислотные свойства
22. Оксид углерода (II) взаимодействует с каждым из двух веществ:
1) О3 и СuО2) Н2О и SО23) H2SO4 и Na4) NaOH и НС1
23. Верны ли следующие суждения об оксиде азота (V)?
А. При взаимодействии со щелочами оксид образует как кислые, так и средние соли.
Б. Растворяясь в воде, оксид дает сильную одноосновную кислоту.
1) верно только А2) верно только Б3) верны оба суждения 4) оба суждения неверны
24. Оксид серы (VI) взаимодействует с каждым из двух веществ:
1)вода и соляная кислота 2)кислород и оксид магния
3)оксид кальция и гидроксид натрия 4)вода и медь
25. Окислительные свойства оксид серы (IV) проявляет в реакции
1)SO2 + NaOH = NaHSO3 2)SO2 + Br2 + 2H2O = H2SO4 + 2HBr
3)SO2 + 2H2S = 3S + 2H2O 4)2SO2 + O2 = 2SO3
26. Оксид серы (IV) взаимодействует с
1)KNО3
2)H2O 3)Na2SO4
4)НCl
27. Между собой могут взаимодействовать
1)SiO2 и H2O2)CO2 и H2SO43)CO2 и Сa(OH)24)Na2O и Сa(OH)2
28. Оксид серы (IV) взаимодействует с каждым из двух веществ:
1)H2O и KCl 2)
Ba(OH)2 и CaO 3)CaCO3 и ZnSO3 4)
Ca(OH)2 и N2
29. Оксид серы (VI) взаимодействует с каждым из двух веществ:
1)вода и соляная кислота 2) кислород и оксид магния
3)оксид кальция и гидроксид натрия 4)вода и медь
30. Оксид фосфора (V) не взаимодействует с
1) гидроксидом натрия 2) оксидом кальция
3) водой 4) кислородом
Ответы: 1-4, 2-1, 3-2, 4-4, 5-3, 6-4, 7-4, 8-4, 9-2, 10-4, 11-2, 12-4, 13-3, 14-1, 15-2, 16-3, 17-1, 18-1, 192, 20-2, 21-3, 22-1, 23-2, 24-1, 25-3, 26-2, 27-3, 28-2, 29-3, 30-4.
А10. Характерные химические свойства оснований, амфотерных
гидроксидов. Характерные химические свойства кислот.
Кочулева Людмила Рамановна, учитель химии МОБУ «Средняя общеобразовательная школа №73» г.
Оренбурга. Опорные конспекты и алгоритмы для подготовки к ЕГЭ.
Характерные химические свойства щелочей:
1)
Действие на некоторые кислотно-основные индикаторы:
o
лакмус становится синим,
o
метилоранж — жёлтым,
o
фенолфталеин приобретает цвет фуксии.
2)
При взаимодействии с кислотой образуется соль и вода:
NaOH + HCl → NaCl + H2O
Условие: реакция не идёт, если и кислота и основание слабые.
3)
Щёлочь + кислотный или амфотерный оксид → соль + вода.
2 NaOH + SiO2 → Na2SiO3 + H2O
4)
Щёлочь + соль → (новое) основание + (новая) соль.
Условие: исходные вещества должны быть в растворе, а хотя бы один из продуктов
реакции должен выпасть в осадок или мало растворяться.
Ba(OH)2 + Na2SO4 → BaSO4 + 2NaOH
Характерные химические свойства нерастворимых оснований:
1)
При взаимодействии с кислотой образуется соль и вода:
Cu(OH)2 + 2HCl → CuCl2 + 2H2O
2)
При нагревании разлагаются на воду и основной оксид:
Cu(OH)2 = CuO + H2O
2 Fe(OH)3 = Fe2O3 + 3 H2O
Характерные химические свойства амфотерных гидроксидов:
1)
Как основание взаимодействуют с кислотой:
Al(OH)3 + 3HCl = AlCl3 + 3H2O
2)
Как кислота взаимодействует со щелочами:
Al(OH)3 + NaOH (конц.)= Na[Al(OH)4](тетрагидроксоалюминат натрия)
t
Al(OH)3 + NaOH(тв.)= NaAlO2 + 2H2O
Характерные химические свойства кислот:
1)
С металлами, стоящими в ряду активности металлов до Н (кроме кислотокислителей H2SO4конц. и HNO3). Образуются соль и выделяется газ водород Н2. Тип
реакции - реакция замещения:
Mg + 2HCl → MgCl2 + H2
Условие: металл до Н, образуется растворимая соль, особые свойства конц. серной
кислоты и азотной кислоты любой концентрации, не используются активные металлы.
2)
С основными и амфотерными оксидами (последние ведут себя как
основные оксиды) при нагревании. Образуются соль и вода. Тип реакции - реакция
обмена:
ZnO + H2SO4 → ZnSO4 + H2O
3)
С основаниями и амфотерными гидроксидами (последние ведут себя как
основания).
Образуются соль и вода. Тип реакции - реакция обмена:
Fe(OH)2 + 2HBr → FeBr2 + 2H2O
Условие: образование растворимой соли.
4)
С солями, сильные кислоты вытесняют слабые из растворов их солей,
образуются новая соль и новая кислота. Тип реакции - реакция обмена, например:
K3PO4 + 3HCl→> 3KCl + H3PO4 (образуется слабая фосфорная кислота)
Кочулева Людмила Рамановна, учитель химии МОБУ «Средняя общеобразовательная школа №73» г.
Оренбурга. Опорные конспекты и алгоритмы для подготовки к ЕГЭ.
Условие: в результате реакции образуется нерастворимая соль или слабая кислота,
газ.
Кислоты изменяют окраску индикаторов (см. таблицу окраски индикаторов
в различных средах). Например, метилоранж в кислой среде меняет окраску с оранжевой
на красную. Индикатор фиксирует наличие ионов Н+ в растворе кислоты: метилоранж +
Н+ > красная окраска.
Формулы важнейших кислот
5)
Кислота
Кислотный остаток.
название
формула
название
формула
Соляная (хлороводородная)
HCl
Хлорид
Cl –
Плавиковая (фтороводородная)
HF
Фторид
F–
Бромоводородная
HBr
Бромид
Br –
Иодоводородная
HI
Иодид
I–
Хлорноватистая
HClO
Гипохлорит
ClO –
Хлорноватая
HClO3
Хлорат
ClO3 –
Хлорная
HClO4
Перхлорат
ClO4 –
Азотистая
HNO2
Нитрит
NO2 –
Азотная
HNO3
Нитрат
NO3 –
Сероводородная
H2S
Сульфид
Гидросульфид
S2 –
HS –
Сернистая
H2SO3
Сульфит
Гидросульфит
SO3 2 –
HSO3 –
Серная
H2SO4
Сульфат
Гидросульфат
SO4 2 –
HSO4 –
Угольная
H2CO3
Карбонат
Гидрокарбонат
СО3 2 –
НСО3 –
Кремниевая
H2SiO3
Силикат
SiO3 2 –
Ортофосфорная
H3PO4
Ортофосфат
Гидроортофосфат
Дигидроортофосфат
РО4 3 –
НРО4 2 –
Н2РО4 –
Муравьиная
НСООН
Формиат
НСОО –
Уксусная
СН3СООН
Ацетат
СН3СОО –
Кочулева Людмила Рамановна, учитель химии МОБУ «Средняя общеобразовательная школа №73» г.
Оренбурга. Опорные конспекты и алгоритмы для подготовки к ЕГЭ.
1. Гидроксид калия взаимодействует с каждым из двух веществ
1) NH3 и HCl 2) CO2 и CuCl2 3) H2SO4 и NaNO3 4) MgO и HNO3
2. Разбавленная хлороводородная кислота взаимодействует с каждым из двух веществ
1) медью и гидроксидом натрия2) магнием и нитратом серебра
3) железом и оксидом кремния (IV)4) свинцом и нитратом калия
3. С соляной кислотой взаимодействует
1) NaHCO3 2)Hg
3) SiO2 4) S
4. Реакция нейтрализации происходит при взаимодействии
1) Fe2O3 и HCl2) Fe(OH)3 и HCl3) FeCl3 и NaNCS4) Fe и HCl
5. Гидроксид железа (II) взаимодействует с
1) азотной кислотой2) оксидом кальция3) сульфатом меди4) аммиаком
6. Разбавленная серная кислота не взаимодействует с
1) гидроксидом кальция2) оксидом меди (II)3) цинком4) оксидом углерода (IV)
7. С гидроксидом калия реагирует каждое из двух веществ
1) AlCl3 и H2S2) CuO и Ba(OH)23) CaCO3 и NH34) K2SO4 и AlCl3
8. Гидроксид кальция реагирует с каждым из двух веществ
1) HCl и СО22) HNO3 и MgO3) HCl и KOH4) BaCl2 и NaOH
9. Гидроксид калия реагирует с
1) водой2) щелочью3) кислотой4) кислотой и щелочью
10. Гидроксид кальция не взаимодействует
1) HCl
2) ZnS
3) CO2 4) HNO3
11. Гидроксид хрома (III) реагирует с каждым из двух веществ
1) СО2 и HCl2) SiO2 и Cu(OH)23) NO и NaNO34) H2SO4 и NaOH
12. Реакция нейтрализации происходит между
1)цинком и соляной кислотой
2)серной кислотой и хлоридом бария
3)гидроксидом кальция и азотной кислотой
4)гидроксидом натрия и сульфатом меди
13 . С раствором серной кислоты взаимодействует каждое из двух веществ:
1)хлорид бария и оксид углерода (IV)2)магний и хлорид бария
3)хлорид натрия и фосфорная кислота4)медь и гидроксид калия
14. Гидроксид кальция реагирует с
1)Ba(NO3)2 2)KCl
3)
NH3·H2O 4)Na3PO4
15. Концентрированная азотная кислота в обычных условиях не взаимодействует с
1)магнием2)гидроксидом натрия3)железом4)оксидом магния
16. При сливании водных растворов уксусной кислоты и гидроксида калия образуется
1)ацетат калия и водород
2)карбонат калия и вода
3)ацетат калия и вода
4)карбид калия и углекислый газ
17. Гидроксид натрия не реагирует с
1)Al(OH)32)ZnO3)H2SO44)Ba(OH)2
18. Разбавленная серная кислота реагирует с каждым из двух веществ:
1)Na2SiO3 и HNO32)Fe2O3 и KNO33)Ag и Cu(OH)24)Fe и Al2O3
19. Как гидроксид алюминия, так и соляная кислота могут взаимодействовать с
1)CuO 2)H2SO4
3)CO2 4)NaOH
Кочулева Людмила Рамановна, учитель химии МОБУ «Средняя общеобразовательная школа №73» г.
Оренбурга. Опорные конспекты и алгоритмы для подготовки к ЕГЭ.
20. Соляная кислота не взаимодействует ни с одним из двух веществ:
1)цинком и гидроксидом натрия
2)медью и оксидом меди (II)
3)ртутью и оксидом углерода (IV)
4)магнием и аммиаком
21. С каждым из перечисленных веществ: H2S, KOH, Zn
взаимодействует
1)
Pb(NO3)2
2)
ZnSO4
3)
Na2CO3
4)
HCl
22. Разбавленная серная кислота может реагировать с каждым из двух веществ:
1)
серой и магнием
2)
оксидом железа (II) и оксидом кремния (IV)
3)
гидроксидом калия и хлоридом калия
4)
нитратом бария и гидроксидом меди (II)
Кочулева Людмила Рамановна, учитель химии МОБУ «Средняя общеобразовательная школа №73» г.
Оренбурга. Опорные конспекты и алгоритмы для подготовки к ЕГЭ.
23. С раствором серной кислоты взаимодействует каждое из двух веществ:
1)
хлорид бария и оксид углерода (IV)
2)
магний и хлорид бария
3)
хлорид натрия и фосфорная кислота
4)
медь и гидроксид калия
25. Раствор гидроксида натрия реагирует с каждым из веществ, указанных попарно
1)
хлоридом железа (III) и углекислым газом
2)
оксидом железа (II) и соляной кислотой
3)
серной кислотой и карбонатом кальция
4)
оксидом цинка и хлоридом калия
26. Гидроксид натрия не реагирует с
Кочулева Людмила Рамановна, учитель химии МОБУ «Средняя общеобразовательная школа №73» г.
Оренбурга. Опорные конспекты и алгоритмы для подготовки к ЕГЭ.
1)
Al(OH)3
2)
ZnO
3)
H2SO4
4)
Ba(OH)2
27. Разбавленная серная кислота реагирует с каждым из двух веществ:
1)
Na2SiO3 и HNO3
2)
Fe2O3 и KNO3
3)
Ag и Cu(OH)2
4)
Fe и Al2O3
28. Как гидроксид алюминия, так и соляная кислота могут взаимодействовать с
1)
CuO
2)
H2SO4
3)
CO 2
4)
NaOH
Кочулева Людмила Рамановна, учитель химии МОБУ «Средняя общеобразовательная школа №73» г.
Оренбурга. Опорные конспекты и алгоритмы для подготовки к ЕГЭ.
29. Соляная кислота не взаимодействует ни с одним из двух веществ:
1)
цинком и гидроксидом натрия
2)
медью и оксидом меди (II)
3)
ртутью и оксидом углерода (IV)
4)
магнием и аммиаком
30. Раствор гидроксида натрия не взаимодействует с
1)
СО2
2)
HСl
3)
SO2
4)
MgO
31. Гидроксид цинка взаимодействует с каждым из двух веществ
1) HCl и NaOH
2) HNO3 и NaCl
3) Mg(OH)2 и NaNO3
4) Н2S и Fe(OH)2
Кочулева Людмила Рамановна, учитель химии МОБУ «Средняя общеобразовательная школа №73» г.
Оренбурга. Опорные конспекты и алгоритмы для подготовки к ЕГЭ.
ОТВЕТЫ: 1-2, 2-2, 3-1, 4-2, 5-1, 6-4, 7-1, 8-1, 9-3, 10-2, 11-4, 12-3, 13-2, 14-4, 15-1, 16-3, 17-4, 18-4, 194, 20-3, 21-1, 22-4, 23-2, 24-3, 25-3, 26-4, 27-4, 28-4, 29-3, 30-4, 31-1.
А11. Характерные химические свойства солей: средних, кислых,
основных;комплексных (на примере соединений алюминия и цинка).
Химические свойства средних солей:
1. Обменные реакции с кислотами.
Условие: сильная кислота взаимодействует с растворимой солью, образование
осадка, газа. Реакиц между нерастворимыми карбонатами и сильными кислотами
проходят, т.к. образуется СО2.
AgNO3 + HCl = AgCl + HNO3
2. Обменные реакции со щелочами.
Условие: образование нерастворимого основания.
Fe(NO3)3 + 3NaOH = Fe(OH)3 + 3NaNO3
3. Обменные реакции с другими солями.
Условие: реагируют растворимые соли, в результате реакции образуется
нерастворимая соль.
CaCl2 + Na2SiO3 = CaSiO3 + 2NaCl
4 . Термическое разложение (карбонатов, нитратов, солей аммония).
а) Карбонаты разлагаются на оксид металла и СО2.
CaCO3 = CaO + CO2
б) разложение нитратов:
Металл левее Mg
Mg – Cu (вкл.)
правее Cu
Схема
МеNO3 = МеNO2 +
МеNO3 = МеО + NO2 + O2
МеNO3 = Ме + NO2 + O2
O2
Пример 2КNO3 = 2КNO2 + O2 2Cu(NO3)2 = 2CuO + 4NO2 + 2AgNO3= 2Ag + 2NO2 +
O2
O2
в) соли аммония разлагаются на летучую кислоту и аммиак:
NH4Cl = NH3 + HCl.
Исключение: NH4NO2 = N2+ 2H2O
5. Гидролиз.
Al2S3 + 6H2O = 2Al(OH)3 + 3H2S
FeCl3+ H2O = Fe(OH)Cl2 + HCl
Na2S + H2O = NaHS +NaOH
Химические свойства кислых солей:
1. Термическое разложение с образованием средней соли
Ca(HCO3)2 = CaCO3 + CO2 + H2O
2KНСО3 = K2СО3 + Н2О + СО2
2. Взаимодействие со щёлочью. Получение средней соли.
Кочулева Людмила Рамановна, учитель химии МОБУ «Средняя общеобразовательная школа №73» г.
Оренбурга. Опорные конспекты и алгоритмы для подготовки к ЕГЭ.
Ba(HCO3)2 + Ba(OH)2 = 2BaCO3 + 2H2O
2KHSO4 + 2NaOH = 2H2O + K2SO4 + Na2SO4
Получение кислых солей:
Кислые соли получаются при взаимодействии кислот с металлами, оксидами
металлов, гидроксидами металлов, солями, аммиаком, если кислота в избытке.
Например:
Zn + 2H2SO4 = H2 + Zn(HSO4)2,
CaO + H3PO4 = CaHPO4 + H2O,
NaOH + H2SO4 = H2O + NaHSO4,
Na2S + HCl = NaHS + NaCl,
NH3 + H3PO4 = NH4H2PO4,
2NH3 + H3PO4 = (NH4)2HPO4.
Также кислые соли получаются при взаимодействии кислотных оксидов со
щелочами, если оксид в избытке. Например:
CO2 + NaOH = NaHCO3, 2SO2 + Ca(OH)2 = Ca(HSO3)2.
Химические свойства основных солей:
1)
Термическое разложение.
[Cu(OH)]2CO3 = 2CuO + CO2 + H2O
малахит
2)
Взаимодействие с кислотой: образование средней соли.
Sn(OH)Cl + HCl = SnCl2 + H2O
Химические свойства комплексных солей:
1) При действии избытка сильной кислоты получаются две средних соли и вода:
Na[Al(OH)4] + 4HCl (изб.) = NaCl + AlCl3 + 4H2O
2) При действии сильной кислоты (в недостатке) получаются средняя соль
активного металла, амфотерный гидроксид и вода:
Na[Al(OH)4] + HCl = NaCl + Al(OH)3 + H2O
3) При действии слабой кислоты получаются кислая соль активного металла,
амфотерный гидроксид и вода: Na[Al(OH)4] + H2S = NaHS + Al(OH)3 + H2O
4) При действии углекислого или сернистого газа получаются кислая соль
активного металла и амфотерный гидроксид:
Na[Al(OH)4] + CO2 = NaHCO3 + Al(OH)3
5) При действии солей, образованных сильными кислотами и катионами Fe 3+, Al3+
и Cr3+, происходит взаимное усиление гидролиза, получаются два амфотерных гидроксида
и соль активного металла:
3Na[Al(OH)4] + FeCl3 = 3Al(OH)3 + Fe(OH)3 + 3NaCl
6) При нагревании гидроксокомплексов щелочных металлов выделяется вода:
Na[Al(OH)4] NaAlO2 + 2H2O
1. Нитрат алюминия в растворе взаимодействует с
Кочулева Людмила Рамановна, учитель химии МОБУ «Средняя общеобразовательная школа №73» г.
Оренбурга. Опорные конспекты и алгоритмы для подготовки к ЕГЭ.
1) KCl
2) Fe(NO3)2
3) MgCl2
4) Ca(OH)2
2. С водными растворами хлороводорода, гидроксида бария и хлорида меди (II) реагирует
1) CaCO3
2) K2SO3
3) Na2SO4
4) Al2(SO4)3
3. И с железом, и с гидроксидом калия и с нитратом серебра реагирует в растворе
1) MgCl2 2) Na2SO4 3) ZnBr2 4) FeCl3
4. И с гидроксидом натрия, и с соляной кислотой, и с хлоридом бария реагирует в растворе
1) (NH4)2CO3
2) Zn(OH)2
3) CO2
4) Na2SO4
5. Раствор йодида калия реагирует с каждым из веществ
1) Br2 и AgNO3 2) AgNO3 и HCl 3) Cl2 и NaOH 4) HCl и Cl2
6. Гидрокарбонат натрия реагирует с каждым из веществ
1) CaCl2 и NaOH 2) NaOH и HCl 3) HCl и О2 4) О2 и СО2
7. И с хлором, и с гидроксидом калия, и с нитратом серебра реагирует
в растворе
1) NaI 2)FeCl3 3) FeCl2 4)CuSO4
8. Карбонат кальция при обычных условиях реагирует с
1) кремниевой кислотой
Кочулева Людмила Рамановна, учитель химии МОБУ «Средняя общеобразовательная школа №73» г.
Оренбурга. Опорные конспекты и алгоритмы для подготовки к ЕГЭ.
2) углекислым газом в водном растворе
3) гидроксидом натрия
4) раствором хлорида бария
9. Очистить воду от ионов кальция, содержащихся в растворенном в ней гидрокарбонате кальция
можно при
1) кипячении
2) добавлении хлорида бария
3) добавлении соляной кислоты
4) добавлении хлорида натрия
10. Сульфид натрия в растворе не реагирует с
1) соляной кислотой
2) сероводородом
3) хлором
4) сульфатом калия
11. И с медью, и с раствором сульфида натрия реагирует
1) соляная кислота
2) раствор нитрата серебра
3) гидроксид калия
4) раствор хлорида железа (III)
12. Реагируют друг с другом
1) ртуть и раствор нитрата свинца (II)
2) бром и раствор хлорида натрия
Кочулева Людмила Рамановна, учитель химии МОБУ «Средняя общеобразовательная школа №73» г.
Оренбурга. Опорные конспекты и алгоритмы для подготовки к ЕГЭ.
3) серная кислота и сульфит натрия
4) раствор гидроксида натрия и сульфид железа (II)
13. Карбонат калия в растворе реагирует с
1) гидроксидом натрия
2) углекислым газом
3) хлоридом натрия
4) кислородом
14. Возможна реакция между
1) хлоридом аммония и гидроксидом кальция
2) сульфатом натрия и соляной кислотой
3) хлоридом меди (II) и ртутью
4) нитратом натрия и водой
15. И с гидроксидом натрия и с разбавленной серной кислотой реагирует соль
1) BaCl2
2) Cu(NO3)2
3) NaHCO3
4) Ca3(PO4)2
16. Азот выделяется при разложении
1) NH4Cl
2) (NH4)2CO3
3) NaNO3
4) NH4NO2
17. Возможна реакция в растворе между
1) нитратом ртути (II) и медью
2) хлоридом натрия и нитратом калия
3) сульфатом бария и соляной кислотой
Кочулева Людмила Рамановна, учитель химии МОБУ «Средняя общеобразовательная школа №73» г.
Оренбурга. Опорные конспекты и алгоритмы для подготовки к ЕГЭ.
4) сульфидом железа (II) и гидроксидом калия
18. И с гидроксидом натрия, и нитратом серебра, и с хлором реагирует в водном растворе
1) Fe2(SO4)3
2) NH4Cl
3) CuBr2 4) K3PO4
19. При действии раствора серной кислоты на раствор карбоната аммония выделяется газ
1) NH3
2) CO2 3) NO2 4) SO2
20. Гидроксид натрия образуется при взаимодействии в растворе
1) NaCl и H2O
2) NaNO3 и Ca(OH)2
3) Na2SO4 и Ba(OH)2
4) NaCl и Fe(OH)3
21. Раствор хлорида железа (II) реагирует с каждым из двух веществ
1) NaOH и Сu
2) HNO3 и Ag 3) Cu и HNO3 4) AgNO3 и Ba(OH)2
22. Превращение Na[Al(OH)4] --> Al(OH)3
происходит при
1) термическом разложении исходного вещества
2) действии на исходное вещество раствора щелочи
3) пропускании углекислого газа через раствор исходного вещества
4) действии на исходное вещество избытком раствора сильной кислоты
23. Хлорид аммония в растворе реагирует с
1) KOH
2) HNO3
3) KNO3
4) MgSO4
Кочулева Людмила Рамановна, учитель химии МОБУ «Средняя общеобразовательная школа №73» г.
Оренбурга. Опорные конспекты и алгоритмы для подготовки к ЕГЭ.
24. Продуктами разложения нитрата натрия являются
1) Na2O и NO2
2) Na, NO2, O2 3) NaNO2 и О2
4) Na2O, NO2, O2
25. Раствор карбоната натрия не реагирует с
1) СО2 2) HNO3
3) CaCl2 4) K2SO4
26. Оцените правильность суждений о карбонатах
А. С соляной кислотой реагируют как растворимые, так и нерастворимые карбонаты
Б. Реакции разложения карбонатов являются окислительно-восстановительными
1) верно только А
2) верно только Б
3) верны оба суждения
4) оба суждения неверны
27. Оцените верность суждений о нитратах
А. Соляная кислота вытесняет из любого нитрата азотную кислоту
Б. Реакции разложения нитратов являются окислительно-восстановительными
1) верно только А
2) верно только Б
3) верны оба суждения
4) оба суждения неверны
28. Нитрат серебра не способен
Кочулева Людмила Рамановна, учитель химии МОБУ «Средняя общеобразовательная школа №73» г.
Оренбурга. Опорные конспекты и алгоритмы для подготовки к ЕГЭ.
1) образовывать осадок при взаимодействии с раствором хлоридом натрия
2) разлагаться при нагревании
3) реагировать в растворе с медью
4) реагировать с уксусной кислотой
29. Осадок сначала выпадает, а затем исчезает при
1) добавлении раствора хлорида цинка к раствору гидроксида натрия
2) пропускании углекислого газа через известковую воду
3) сливании растворов сульфата натрия и хлорида бария
4) сливании растворов карбоната натрия и азотной кислоты
30. Карбонат натрия в растворе не реагирует с
1) СО2
2) КОН
3) Н3РО4
4) Са(ОН)2
31. С карбонатом натрия реагирует каждое из двух веществ
1) KOH и Mg(NO3)2
2) H2SO4 и Ca3(PO4)2
3) HCl и BaCl2
4) K2S и CaSO4
Ответы: 1-4, 2-2, 3-4, 4-1, 5-1, 6-2, 7-3, 8-2, 9-1, 10-4, 11-4, 12-3, 13-2, 14-1, 15-3, 16-4, 17-1, 18-3, 192, 20-3, 21-4, 22-3, 23-1, 24-3, 25-4, 26-1, 27-2, 28-4, 29-2. 30-2, 31-2.
А12. Взаимосвязь неорганических веществ.
Кочулева Людмила Рамановна, учитель химии МОБУ «Средняя общеобразовательная школа №73» г.
Оренбурга. Опорные конспекты и алгоритмы для подготовки к ЕГЭ.
Генетические связи - это связи между разными классами, основанные на их
взаимопревращениях.
Среди металлов можно выделить две разновидности рядов:
1.Генетический ряд, в котором в качестве основания выступает щёлочь.
Этот ряд можно представить с помощью следуюших превращений:
+ О2
+Н2О
+ кислота
Металл → основный оксид → щёлочь → соль.
K→ K2O→KOH→KCl
2. Генетический ряд, где в качестве основания выступает нерастворимое
основание:
+ О2
+ кислота +щёлочь
+t
+ Н2
Металл→ основный оксид→ соль → нерастворимое основание→ основный
оксид→ металл.
Cu→ CuO→CuCl2→ Cu(OH)2→ CuO→ Cu
Среди неметаллов также можно выделить две разновидности рядов:
1. Генетический ряд неметаллов, где в качестве звена ряда выступает растворимая
кислота. Цепочку превращений можно представить в следующем виде:
+ О2
+Н2О
+ щёлочь
Неметалл→ кислотный оксид→ растворимая кислота→ соль.
P→P2O5→H3PO4→Na3PO4.
2. Генетический ряд неметаллов, где в качестве звена ряда выступает
нерастворимая кислота :
+ О2
+ щёлочь + сильная кислота + tо
+ магний
Неметалл→ кислотный оксид → соль →
кислота → кислотный оксид→
неметалл.
Это генетический ряд кремния:
Si → SiO2→ Na2SiO3 → H2SiO3 → SiO2 → Si
1. В цепочке превращений
СО2 + Х = Na2СO3
Na2СО3 + СО2+Н2О = Y
веществами Х и Y соответственно являются
1) гидроксид натрия и гидрокарбонат натрия
2) оксид натрия и угольная кислота
3) гидрокарбонат натрия и угольная кислота
4) натрий и гидрокарбонат натрия
Кочулева Людмила Рамановна, учитель химии МОБУ «Средняя общеобразовательная школа №73» г.
Оренбурга. Опорные конспекты и алгоритмы для подготовки к ЕГЭ.
2. В цепочке превращений
Р4+О2(изб.)= Х
Х+NaOH(изб.)=Y
веществами Х и Y соответственно являются
1) оксид фосфора (III) и фосфат натрия
2) оксид фосфора (V) и фосфат натрия
3) оксид фосфора (III) и дигидрофосфат натрия
4) оксид фосфора (V) и гидрофосфат натрия
3. В цепочке превращений
NH3 + X = NH4Cl
NH4Cl + NaOH = Y
веществами Х и Y соответственно являются
1) хлор и аммиак
2) хлороводород и аммиак
3) хлороводород и хлорная кислота
4) соляная кислота и азот
4. В цепочке превращений
СаО + С = Х
Х + Н2О = Y
веществами Х и Y соответственно являются
1) карбид кальция и гидроксид кальция
2) карбонат кальция и гидроксид кальция
3) карбид кальция и оксид кальция
4) кальций и гидроксид кальция
Кочулева Людмила Рамановна, учитель химии МОБУ «Средняя общеобразовательная школа №73» г.
Оренбурга. Опорные конспекты и алгоритмы для подготовки к ЕГЭ.
5. В цепочке превращений
N2 + X = NH3
NH3 + О2 = Y
веществами Х и Y соответственно являются
1) вода и оксид азота (II)
2) водород и оксид азота (IV)
3) водород и азот
4) вода и оксид азота (IV)
6. В цепочке превращений
Fe -->X -->Y -->FeO
веществами Х и Y соответственно являются
1) хлорид железа (II) и гидроксид железа (II)
2) хлорид железа (II) и гидроксид железа (II)
3) оксид железа (II) и гидроксид железа (II)
4) хлорид железа (III) и оксид железа (III)
7. В цепочке превращений
FeO +X = Fe
Fe + HCl = Y
веществами Х и Y соответственно являются
1) угарный газ и хлорид железа (II)
2) уголь и хлорид железа (III)
3) водород и хлорид железа (III)
Кочулева Людмила Рамановна, учитель химии МОБУ «Средняя общеобразовательная школа №73» г.
Оренбурга. Опорные конспекты и алгоритмы для подготовки к ЕГЭ.
4) углекислый газ и хлорид железа (II)
8. В цепочке превращений
NH3 -->X -->Y -->HNO3
веществами Х и Y соответственно являются
1) оксид азота (IV) и оксид азота (II)
2) оксид азота (II) и оксид азота (IV)
3) оксид азота (II) и азот
4) азота и оксид азота (IV)
9. В цепочке превращений
Cu+X=CuCl2
CuCl2+NaOH=Y
веществами Х и Y соответственно являются
1) соляная кислота и гидроксид меди (II)
2) хлор и оксид меди (II)
3) хлорид калия и гидроксид меди (II)
4) хлор и гидроксид меди (II)
10. В цепочке превращений
Ba+H2O=X
X+CO2(изб.)=Y
веществами Х и Y соответственно являются
1) оксид бария и карбонат бария
2) гидроксид бария и карбонат бария
3) оксид бария и гидрокарбонат бария
Кочулева Людмила Рамановна, учитель химии МОБУ «Средняя общеобразовательная школа №73» г.
Оренбурга. Опорные конспекты и алгоритмы для подготовки к ЕГЭ.
4) гидроксид бария и гидрокарбонат бария
11. В цепочке превращений
H2S -->X -->Y -->H2SO4
веществами Х и Y соответственно являются
1) оксид серы (IV) и оксид серы (VI)
2) оксид серы (IV) и сульфит натрия
3) сера и оксид серы (VI)
4) сера и оксид серы (IV)
12. В цепочке превращений
NH4Cl -->X -->Y -->NO2
веществами Х и Y соответственно являются
1) аммиак и азот
2) аммиака и оксид азота (II)
3) азот и оксид азота (II)
4) азот и аммиак
13. В цепочке превращений
Fe+Cl2=X
X+NaOH=Y
веществами Х и Y соответственно являются
1) хлорид железа (II) и гидроксид железа (II)
2) хлорид железа (II) и FeO(OH)
3) хлорид железа (III) и гидроксид железа (II)
Кочулева Людмила Рамановна, учитель химии МОБУ «Средняя общеобразовательная школа №73» г.
Оренбурга. Опорные конспекты и алгоритмы для подготовки к ЕГЭ.
4) хлорид железа (III) и гидроксид железа (III)
14. В цепочке превращений
Fe2O3+X=Fe
Fe+Y=FeCl3
веществами Х и Y соответственно являются
1) оксид углерода (II) и хлор
2) алюминий и хлороводород
3) оксид углерода (IV) и хлор
4) сероводород и хлороводород
15. В цепочке превращений
N2 -->X -->Y -->NO2
веществами Х и Y соответственно являются
1) NH3 и N2
2) NH3 и NO
3) NO и HNO3
4) NO2 и HNO3
16. В цепочке превращений
Fe3O4+H2=X
X+Y=Fe(NO3)3
веществами Х и Y соответственно являются
1) Fe и разб. гор. HNO3
2) Fe2O3 и разб. HNO3
3) Fe(OH)2 и конц. HNO3
Кочулева Людмила Рамановна, учитель химии МОБУ «Средняя общеобразовательная школа №73» г.
Оренбурга. Опорные конспекты и алгоритмы для подготовки к ЕГЭ.
4) Fe(OH)2 и разб. HNO3
17. В цепочке превращений
Cu+X=CuCl2
CuCl2+Y=Cu(NO3)2
веществами Х и Y соответственно являются
1) HCl и HNO3
2) HCl и AgNO3
3) Cl2 и HNO3
4) Cl2 и AgNO3
18. В цепочке превращений
FeS+X=H2S
H2S+Y=S
веществами Х и Y соответственно являются
1) H2 и Na
2) H2O и O2
3) HCl и SO2
4) HCl и N2
19. В цепочке превращений
NaOH+SO2(изб.)=X
X+Y=NaCl
веществами Х и Y соответственно являются
1) сульфит натрия и соляная кислота
Кочулева Людмила Рамановна, учитель химии МОБУ «Средняя общеобразовательная школа №73» г.
Оренбурга. Опорные конспекты и алгоритмы для подготовки к ЕГЭ.
2) сульфит натрия и хлор
3) гидросульфит натрия и соляная кислота
4) сульфид натрия и хлор
20. В цепочке превращений
FeCl3 -->X -->Y -->Fe(OH)3
веществами Х и Y соответственно являются
1) сульфат железа (III) и оксид железа (III)
2) фосфат железа (III) и Fe3O4
3) нитрат железа (III) и оксид железа (III)
4) гидроксид железа (III) и сульфат железа (III)
21. В цепочке превращений
Fe2O3 -->X -->Y -->Fe2O3
веществами Х и Y соответственно являются
1) нитрат железа (III) и хлорид железа (III)
2) сульфат железа (III) и сульфид железа (II)
3) гидроксид железа (III) и хлорид железа (III)
4) хлорид железа (III) и гидроксид железа (III)
22. В цепочке превращений
FeSO4 -->X -->Y -->Fe
веществами Х и Y соответственно являются
1) хлорид железа (III) и фосфат железа (II)
2) оксид железа (II) и гидроксид железа (II)
3) гидроксид железа (II) и оксид железа (II)
Кочулева Людмила Рамановна, учитель химии МОБУ «Средняя общеобразовательная школа №73» г.
Оренбурга. Опорные конспекты и алгоритмы для подготовки к ЕГЭ.
4) фосфат железа (II) и гидроксид железа (II)
23. В цепочке превращений
Al(OH)3+A=X
X+Y -->Al(OH)3
веществами А и Y соответственно являются
1) сульфат калия и гидроксид калия
2) хлорид натрия и соляная кислота
3) сульфат натрия и серная кислота
4) азотная кислота и гидроксид натрия
24. Общая схема превращений
Э -->Э2О3 -->Э(ОН)3
соответствует генетическому ряду
1) натрий --> оксид натрия --> гидроксид натрия
2) алюминий --> оксид алюминия --> гидроксид алюминия
3) кальций --> оксид кальция --> гидроксид кальция
4) азот --> оксид азота (V) --> азотная кислота
25. В схеме превращений
FeCl2+NaOH=X
X+O2+H2O=Y
веществом Y является
1) FeO
2) Fe(OH)3
3) FeCl2
4) FeCl3
Кочулева Людмила Рамановна, учитель химии МОБУ «Средняя общеобразовательная школа №73» г.
Оренбурга. Опорные конспекты и алгоритмы для подготовки к ЕГЭ.
26. В схеме превращений
FeCl3 -->X -->Y -->Fe(OH)3
веществами X и Y могут быть соответственно
1) сульфат железа (III) и оксид железа (III)
2) фосфат железа (III) и Fe3O4
3) нитрат железа (III) и оксид железа (III)
4) гидроксид железа (III) и сульфат железа (III)
27. В схеме превращений
CaCO3 --> X
X + H2O = Y
Веществом Y является
1) оксид кальция
2) гидроксид кальция
3) гидрокарбонат кальция
4) гидрид кальция
28. В схеме превращений
Сu(OH)2 -->A
A+H2=B
В+О2=X
веществом X является
1) CuO
2) Cu
3) Cu(OH)2 4) CuCl2
29. В схеме превращений
Fe(OH)3 --> X
Кочулева Людмила Рамановна, учитель химии МОБУ «Средняя общеобразовательная школа №73» г.
Оренбурга. Опорные конспекты и алгоритмы для подготовки к ЕГЭ.
X + HCl = Y
веществом Y является
1) оксид железа (III)
2) оксид железа (II)
3) хлорид железа (III)
4) хлорид железа (II)
30. В схеме превращений
CaCO3 + HCl = X
X+Na2CO3=Y+NaCl
веществом Y является
1) карбонат кальция
2) хлорид кальция
3) оксид кальция
4) гидроксид кальция
31. В схеме превращений
S®X1
Ответы: 1-1, 2-2, 3-2, 4-1, 5-3, 6-1, 7-1, 8-2, 9-4, 10-4, 11-1, 12-2, 13-4, 14-1, 15-2, 16-1, 17-4, 18-3, 194, 20-1, 21-4, 22-3, 23-4, 24-2, 25-2, 26-4, 27-2, 28-1, 29-3, 30-1
А13. Теория строения органических соединений: гомология и изомерия.
Взаимное влияние атомов в молекулах. Типы связей в молекулах органических
веществ. Гибридизация атомных орбиталей углерода. Радикал. Функциональная
группа.
1)
Гомологи - это вещества одного класса, сходные по строению и свойствам и
отличающиеся на одну или несколько групп СН2.
Кочулева Людмила Рамановна, учитель химии МОБУ «Средняя общеобразовательная школа №73» г.
Оренбурга. Опорные конспекты и алгоритмы для подготовки к ЕГЭ.
Класс
Общая формула
Алканы
СnH2n+2
Циклоалканы
СnH2n
Алкены
СnH2n
Алкины
СnH2n-2
Алкадиены
СnH2n-2
Арены
СnH2n-6
Моногалогеналканы
СnH2n+1Г
Дигалогеналканы
СnH2nГ2
Одноатомные спирты
СnH2n+1ОН
Аледегиды
СnH2n+1СHO
Предельные одноосновные карбоновые
СnH2n+1СООН
кислоты
Простые эфиры
СnH2n+2О
Первичные амины
СnH2n+1NH2
Аминокислоты
(NH2)СnH2nCOOH
2)
Изомеры - это вещества, имеющие одинаковый состав, но разное строение и
разные свойства.
3)
Связи σ и π. Все одинарные связи - σ; кратные – одна σ, остальные - π.
4)
Характеристика углеводородов.
Признаки
Тип гибридиз.
Число гибридн.
орбиталей
атомов C
Связи
Алканы
sp3
4
Циклоалканы
sp3
4
Алкены
sp2
3
Алкадиены
sp2
3
Алкины
sp
2
Арены
sp2
3
σ
σ
σ, π
σ, π
σ, π
Форма молекул
тетраэдрическая
циклическая
плоская
плоская
линейная
σ,
π-элект.
система
плоская,
циклическая
Изомерия
структурная
1. угл. скелета
+
+
+
+
+
-
-
+
+
+
+
+
-
+
+
+
+
+
+
+
(в бок. цепи)
+
(в бок. цепи)
+
-
-
+
+
-
-
2. положения
кратных связей
3. межклассовая
4. положения
заместителей
Изомерия
геометр.
(пространствен.
цис-, трансизомерия)
5)
Радикал - группа атомов, содержащая углеводородный остаток в молекуле.
Название важнейших радикалов
Формула
Метил
- СH3
Этил
- С2H5
Кочулева Людмила Рамановна, учитель химии МОБУ «Средняя общеобразовательная школа №73» г.
Оренбурга. Опорные конспекты и алгоритмы для подготовки к ЕГЭ.
Пропил
Бутил
Фенил
- С3H7
- С4H9
- С6H5
Кочулева Людмила Рамановна, учитель химии МОБУ «Средняя общеобразовательная школа №73» г.
Оренбурга. Опорные конспекты и алгоритмы для подготовки к ЕГЭ.
6)
ФГ, изомерия кислородсодержащих и азотсодержащих ОС.
Класс
спирты
простые эфиры
форму
ла
- ОН
-О-
фенолы
кетоны
- ОН
>С=O
альдегиды
КК
–CHO
–
COOH
–COO-
сложные эфиры
ФГ
название
гидроксил
оксигруппа
(кислородный
мостик)
гидроксил
карбонильная
карбонильная
карбоксильная
Примеры
веществ
метанол
углеродного
скелета
+
СН3 - О - СН3
диметиловый эфир
+
С6Н5 - ОН
СН3 - С- СН3
О
С2Н5 - СНО
С3Н7 - СООН
фенол
ацетон, пропанон
+
пропаналь
бутановая кислота
+
+
метилпропионат или
метиловый эфир
пропановой кислоты
нитрометан
метиламин
+
глицин
(аминоуксусная
кислота)
+
СН3- ОН
сложноэфирная С2Н5 –СОО – СН3
нитросоединения
амины
–NO2
–NH2
нитро
аминогруппа
АМК
–NH2,
–
COOH
аминогруппа
карбоксильная
Название вещества
СН3–NO2
СН3–NH2
NH2- СН2- COOH
+
+
Изомерия
положения
межклассовая
ФГ
положения
изомерны простым
гидроксила
эфирам
изомерны спиртам
изомерны
альдегидам
изомерны кетонам
изомерны сложным
эфирам
изомерны КК
изомерны АМК
положения
аминогруппы
положения
аминогруппы
изомерны
нитросоединениям
Кочулева Людмила Рамановна, учитель химии МОБУ «Средняя общеобразовательная школа №73» г. Оренбурга. Опорные конспекты и алгоритмы для подготовки к
ЕГЭ.
А14.
Характерные
химические
свойства
углеводородов:
алканов,
циклоалканов, алкенов, диенов, алкинов, ароматических углеводородов (бензола и
толуола).
Химические свойства алканов
Тип реакции
I.
Реакции окисления.
1. Горение:
а) полное
Уравнение
CH4 + 2O2 → CO2 + 2H2O + Q
2С4Н10 + 13О2→ 8СО2
+10Н2О+Q
б) неполное.
2CH4 + 3O2 → 2CO + 4H2O
2. Каталитическое окисление
2CH4 + O2 → 2CH3OН
(различные катализаторы и t окисления)
CH4 + O2 → НCНO + H2O
а) метан – до метанола, метаналя или муравьиной 2CH4 + 3O2 → 2НCOOН+ 2H2O
кислоты
2C4H10 + 5O2 → 4СН3COOН+
б) гомологи метана окисляются с разрывом С-С цепи и 2H2O
образованием КК
II.
Замещение.
1.Галогенирование (на свету или при t).
CH4 + Cl2 → CH3Cl + HCl
При хлорировании или бромировании алкана с При достаточном количестве
вторичными или третичными атомами углерода легче хлора реакция продолжается
всего идет замещение водорода у третичного атома, дальше:
труднее у вторичного и еще труднее у первичного. CH3Cl + Cl2 → CH2Cl2 +HCl
Поэтому, например, при бромировании пропана CH2Cl2 + Cl2 →CHCl3 + HCl
основным продуктом реакции является 2-бромпропан.
CHCl3 + Cl2 → CCl4 + HCl
2.Нитрование (реакция Коновалова)
С2Н6 + НNО3→ С2Н5NО2+ H2O
III.
Термические превращения алканов
1.
Дегидрирование гомологов метана
Гомологи метана превращаются
в алкены:
С2Н6 → С2Н4 + Н2
2.
Изомеризация
СН3-СН2-СН2-СН3→
СН3-СН(при нагревании и в присутствии хлорида алюминия)
СН3
СН3
3.
Пиролиз (разложение на простые вещества):
1000о С
а) полное
СН4 → С + 2Н2
1500оС
б) неполное
2CH4 → С2Н2 + 3Н2
4.
Разрушение С - С цепи (крекинг) (400-500 оС)
Алкан
→ новый алкан +
алкен
С20Н42 → С10Н22 + С10Н20
о
IV.
Конверсия метана при 800 С и в присутствии СН4 + Н2О → СО + 3Н2
никелевого катализатора
V.
Ароматизация алканов (С ≥ 6) при нагревании и С6Н14→ С6Н6 + 4Н2
действии катализатора
VI.
При о.у. не обесцвечивают бромную воду и раствор перманганата калия
VII.
Реакции присоединения не характерны
Кочулева Людмила Рамановна, учитель химии МОБУ «Средняя общеобразовательная школа №73» г.
Оренбурга. Опорные конспекты и алгоритмы для подготовки к ЕГЭ.
Химические свойства циклоалканов
Тип реакции
Уравнение химической реакции
I.
Общие с алканами (для больших циклов С ≥ 5) протекают с сохранением цикла
1)
Замещение:
а) галогенирование
б) нитрование
2)
Дегидрирование
t,Pt
→ С6Н6 + 3Н2
циклогексан
3)
Горение
II.
бензол
С6Н12+ 9O2 → 6CO2 + 6 H2O+Q
Особые свойства - отличные от алканов (для малых циклов С = 3, 4),
протекают с разрывом цикла
1)
Реакции присоединения
а) галогенирование
б) гидрирование
в)
гидрогалогенирование
Тип реакции
I.
Окисление:
1)
Полное
(горение)
2)
Неполное
(разрыв только
связей)
3)
Жёсткое
окисление
Химические свойства алкенов
Уравнение химической реакции
С2Н4 + 2О2 → СО2 + 2Н2О + Q
Окисление этилена перманганатом калия в нейтральной среде
π- приводит к образованию двухатомного спирта – этиленгликоля.
Упрощённо:
CH2 = CH2 + НОН + [O] → CH2(OH) – CH2(OH)
3 CH2 = CH2 + 2KMnO4 + 4H2O → 3 CH2 – CH2 + 2MnO2 + 2KOH
|
|
OH OH
При окислении алкенов кипящим раствором KMnO4 в кислой
среде происходит полное разрушение двойной связи и
превращение атомов углерода, между которыми существует
Кочулева Людмила Рамановна, учитель химии МОБУ «Средняя общеобразовательная школа №73» г.
Оренбурга. Опорные конспекты и алгоритмы для подготовки к ЕГЭ.
Присоединение
1) Гидрирование
при о.у.
2)
Гидратация
II.
3)
Галогенирован
ие
4)
Гидрогалогени
рование
двойная связь, в атомы углерода карбоксильной группы с
образованием карбоновых кислот или СО2:
5CH3-CH=CH-CH2-CH3 + 8KMnO4 + 12H2SO4 →5CH3COOH +
5C2H5COOH + 8MnSO4 + 4K2SO4 + 17H2O
5CH3CH=CH2 + 10KMnO4 + 15H2SO4 →5CH3COOH + 5CO2 +
10MnSO4 + 5K2SO4 + 20H2O
Дихромат калия в сернокислотной среде окисляет алкены
аналогично.
Если атом углерода при двойной связи содержит 2 заместителя,
то при его окислении образуется кетон:
5(CH3)2-C=CH-CH3 + 6KMnO4 + 9H2SO4 →5CH3COOH +
5(CH3)2C=O + 6MnSO4 + 3K2SO4 + 9H2O
В щелочной среде:
3CH3-CH=CH-CH2-CH3 + 6KMnO4 + 10KOH →CH3COOK +
C2H5COOK + 6H2O + 6K2MnO4
4CH3CH=CH2 + 10KMnO4 + 13KOH →CH3COOK + K2CO3 +
8H2O+ 10K2MnO4
CH2 = CH2 + Н2 → CH3 – CH3
H2SO4
СН2 = СН2 + НОН → СН3 – СН2ОН
Присоединение воды к несимметричному алкену происходит по
правилу Марковникова: «Атом водорода присоединяется к
наиболее гидрированному атому углерода, а гидроксогруппа к
менее гидрированному атому углерода».
CH3-CH=CH2 + НОН → СН3 – СН– СН3
ОН
CH2 = CH2 + Br2 → CH2Br – CH2Br
CH2 = CH2 + HBr → CH3– CH2Br
Присоединение галогеноводорода к несимметричному алкену
происходит по правилу Марковникова: «Атом водорода
присоединяется к наиболее гидрированному атому углерода, а
галогены к менее гидрированному атому углерода».
СН3 – CH = CH2 + HBr → CH3 – CH – CH3
|
Br
III. Полимеризация - n CH2 = CH2 → (– CH2 – CH2 –)n - полиэтилен
процесс образования
полимера
из n CH2 = CН
→ (– CH2 – CH –)n - полипропилен
низкомолекулярных
|
|
веществ,
без
CH3
CH3
выделения побочных
продуктов
IV.Реакции
Наиболее легко замещается галогенами атом Н у первого атома С,
замещения:
считая от двойной связи:
галогенирование
CH3- CH = CH2 + Cl2 → Cl - CH2- CH = CH2 + HCl
Кочулева Людмила Рамановна, учитель химии МОБУ «Средняя общеобразовательная школа №73» г.
Оренбурга. Опорные конспекты и алгоритмы для подготовки к ЕГЭ.
Химические свойства алкадиенов
Тип реакции
Уравнение химической реакции
I.
Реакции присоединения
У алкадиенов реакции присоединения могут протекать по двум направлениям:
1) по месту разрыва одной двойной связи (1,2-присоединение).
2) с присоединением к концам молекулы и разрывом двух двойных связей (1,4присоединение). Преимущественное протекание реакции по тому или иному пути зависит от
конкретных условий.
1)
галогенирование
CH2 = CH - CH = CH2 + Br2 → CH2Br – CH = CH - CH2Br
1,4 –дибромбутен-2
2)
гидрирование
CH2 = CH - CH = CH2 + Н2 → CH3 – CH = CH - CH3
бутен-2
3)
гидрогалогениров CH2 = CH - CH = CH2 + НBr → CH2Br – CH = CH - CH3
ание
1 –бромбутен-2
II.
Реакция
полимеризации
CH2 = CH - CH = CH2 + НBr → CH3 – CHBr - CH = CH2
3 –бромбутен-1
Получают синтетические каучуки:
t, p, кат
nCH2 = CH - CH = CH2 → (- CH2 - C = CH - CH2- )n
бутадиеновый СК
t, p, кат
nCH2 = C - CH = CH2 → (- CH2 - C = CH - CH2- )n
СН3
СН3
изопреновый СК
Химические свойства алкинов
I.
Тип реакции
Уравнение химической реакции
Окисление: 2 C2H2 + 5О2 → 4СО2 + 2Н2О + Q
1)
Полное
(горение)
2)
Неполное
а) ацетилен
б) гомологи
Ацетилен может быть окислен перманганатом калия в нейтральной среде
до оксалата калия:
3CH≡CH +8KMnO4→3KOOC – COOK +8MnO2 + 2КОН + 2Н2О
В кислотной среде окисление идет до щавелевой кислоты или углекислого
газа:
5CH≡CH +8KMnO4 +12H2SO4 →5HOOC – COOH + 8MnSO4 + 4К2SO4
+12Н2О
В упрощённом виде:
KMnO4
HCCH + 4[О] НООС- СООН  НСООН + СО2
CH≡CH + 2KMnO4 +3H2SO4 → 2CO2 + 2MnSO4 + 4H2O + K2SO4
KMnO4 в кислой среде расщепляет тройную связь с образованием
Кочулева Людмила Рамановна, учитель химии МОБУ «Средняя общеобразовательная школа №73» г.
Оренбурга. Опорные конспекты и алгоритмы для подготовки к ЕГЭ.
карбоновых кислот
5CH3C CH + 8KMnO4 + 12H2SO4 → 5CH3COOH + 5CO2 + 8MnSO4 +
4K2SO4 + 12H2O
II. Присоедине
Ni, t
ние
(реакции HCCH + Н2 Н2С=СН2
могут протекать в
две стадии)
1)
Гидрирова
ние
2)
Гидратация Реакция Кучерова:
Н+, Hg2+
HCCH + Н2О  СН3-СНО
Гомологи ацетилена – в кетоны:
Н+, Hg2+
HCCСH3 + Н2О  СН3-С-СН3

О (ацетон)
3)
Галогениро HCCH + Br2 → HCBr = CBrH
ва-ние
HCBr=CBrH+ Br2→ HCBr2CBr2H
HCCH + НCl СН2 = СНCl
хлорэтен (винилхлорид)
По правилу Марковникова – несимметричниые алкины.
II.
Тримериза- Реакция Зелинского:
ция ацетилена
акт. С, 500 С
3НC  CH → С6H6 (бензол)
III.
Реакция
Водород при атоме углерода с тройной связью проявляет очень слабые
замещения
кислотные свойства и может замещаться на металл:
(кислотные
HCCH +Ag2O  Ag- C  C- Ag  +H2O
свойства)
* HCCH +2[Ag(NH3)2]OH  Ag- C  C- Ag + 4NH3 +2H2O
CH3CCH +Ag2O  CH3CCAg  +H2O
* CH3CCH + [Ag(NH3)2]OH  CH3CC- Ag + 2NH3 +2H2O
CH3CCСH3 + [Ag(NH3)2]OH  реакция не идёт
4)
Гидрогалог
ени-рование
Химические свойства аренов на примере бензола
Тип реакции
I. Окисление:
1)
Полное (горение)
Неполное окисление не характерно.
II. Реакции замещения - наиболее характерны,
бензольное кольцо не разрушается (сходство с
алканами):
1)
Галогенирование
2)
нитрование
Уравнение химической реакции
2C6H6 + 15О2 → 12СО2 + 6Н2О + Q
FeCl3
С6H6 + чист.Cl2 → С6H5Cl + HCl
хлорбензол
FeBr3
С6H6 + чист.Br 2 → С6H5Br + HBr
бромбензол
t, к.H2SO4
С6H6 + HNO3 → С6H5NO2 + H2O
Кочулева Людмила Рамановна, учитель химии МОБУ «Средняя общеобразовательная школа №73» г.
Оренбурга. Опорные конспекты и алгоритмы для подготовки к ЕГЭ.
3)
алкилирование-получение гомологов вещества
а) с хлоралканами
б) с алкенами
нитробензол
t, AlCl3
С6H6 + С2H5Cl → С6H5С2H5 + HCl
этилбензол
t, AlCl3
С6H6 + СН2=СH - СН3 →
С6H5СН(СН3)2
изопропилбензол
(кумол)
III.Реакции присоединения- менее характерны,
протекают с трудом (в жёстких условиях), т.к.
происходит разрушение
π-электронного облака (сходство с алканами):
1)
гидрирование
2)
присоединение Cl2 при освещении
t,Ni
С6Н6 + 3Н2 →
циклогексан
hυ
С6H6 + 3Cl2 → C6H6Cl6
гексахлорциклогексан
(гексахлоран)
Химические свойства гомологов бензола (на примере толуола)
Тип реакции
I.
Уравнение химической реакции
Реакции с участием бензольного кольца
1)
Реакции замещения –
протекают легче, чем у бензола,
бензольное кольцо не
разрушается:
а) галогенирование – идёт в ортои пара-положениях по отношению
к метильной группе
б) нитрование
Кочулева Людмила Рамановна, учитель химии МОБУ «Средняя общеобразовательная школа №73» г.
Оренбурга. Опорные конспекты и алгоритмы для подготовки к ЕГЭ.
в) алкилирование - получение
гомологов вещества
2)
Реакции присоединения гидрирование
метилциклогексан
II. Реакции с участием боковой цепи
1)
Замещение – атомы Н в
боковой цепи могут замещаться на
галоген, как у алканов
2)
Окисление:
а) полное (горение)
б) неполное окисление
C7H8 + 9О2 → 7СО2 + 4Н2О + Q
Любой гомолог бензола при нагревании окисляется
раствором KMnO4 до бензойной кислоты. Окислению
подвергается боковая цепь:
В упрощённом виде:
5С6H5СH3 + 6KMnO4 + 9H2SO4 → 5С6H5СOOH + 6MnSO4
+ 3K2SO4 + 14H2O
Кочулева Людмила Рамановна, учитель химии МОБУ «Средняя общеобразовательная школа №73» г.
Оренбурга. Опорные конспекты и алгоритмы для подготовки к ЕГЭ.
Гомологи бензола могут быть окислены раствором
перманганата калия в нейтральной среде до бензоата
калия:
C6H5CH3 +2KMnO4 = C6H5COOK + 2MnO2 + KOH + H2O
C6H5CH2CH3 + 4KMnO4 = C6H5COOK + K2CO3 + 2H2O +
4MnO2 + KOH
Если в молекуле несколько боковых цепей, то каждая из
них окисляется до карбоксильной группы, в результате
образуются многоосновные кислоты:
Непредельные:
3C6H5CH=CH2 + 2KMnO4+ 4H2O → 3C6H5CH(OH)CH2OH+
2MnO2+2KOH
А15. Характерные химические свойства предельных одноатомных и многоатомных
спиртов; фенола
Характерные химические свойства предельных одноатомных спиртов
Тип реакции
Уравнение
VIII.
1. Горение (полное окисление)
2.Неполное
окисление
(CuO,
растворы
KMnO4,
K2Cr2O7).
Первичные спирты
– до
альдегидов, вторичные – до
кетонов.
3. Ферментативное брожение
(окисление) жидкостей,
содержащих спирт (вино, пиво)
IX.
Реакции окисления
2СН3ОН + 3О2 → 2СО2 + 4Н2О
С2Н5ОН + 3О2 → 2СО2 + 3Н2О
t
C2H5OH + CuO → CH3COH + Cu+H2О
качественная реакция
K2Cr2O7 + 3C2H5OH + 4H2SO4 → 3CH3COH + Cr2(SO4)3
+ K2SO4 + 7H2O
3C2H5OH + K2Cr2O7 + 4H2SO4 = 3CH3CHO + K2SO4 +
Cr2(SO4)3 + 7H2O
С избытком окислителя (KMnO4, K2Cr2O7) в любой
среде первичные спирты окисляются до карбоновых
кислот или их солей, а вторичные – до кетонов.
Третичные спирты в этих условиях не окисляются, а
метиловый спирт окисляется до углекислого газа.
ферменты
C2H5OH + О2 → CH3CОOH + H2О
Реакции с разрывом связи О – Н
Кочулева Людмила Рамановна, учитель химии МОБУ «Средняя общеобразовательная школа №73» г.
Оренбурга. Опорные конспекты и алгоритмы для подготовки к ЕГЭ.
1.
Как
слабые
кислоты 2C2H5OH+2Na → 2C2H5ONa + H2
взаимодействуют со щелочными и
этилат натрия
щелочноземельными металлами
(со
щелочами
не
взаимодействуют)
2.
Реакция
этерификацииH2SО4 конц.,t
взаимодействие
спиртов
с CH3COOH + C2H5OH→ CH3COOC2H5+ H2О
органическими
кислотами
с
образованием сложных эфиров
X.
Реакция с разрывом связи С – О
1.
Гидрогалогенирование
к. H2SO4, t
C2H5OH + HC1→ C2H5C1+ H2О
2.
Дегидратация:
к. H2SO4, t >150°C
а) внутримолекулярная –
C2H5OH
→ C2H4 + H2О
образуются алкены
б) межмолекулярная - образуются
H2SО4, t <150°C
простые эфиры
2C2H5OH →
C2H5OC2H5 + H2О
диэтиловый эфир
XI.
Дегидрирование спиртов
1.
При пропускании паров
Cu
спирта
при
200-300оС
над C2H5OH → CH3COH + H2
металлическим катализатором (Cu,
Ag,
Pt)
первичные
спирты
превращааются в альдегиды, а
вторичные – в кетоны
XII.
Дегидратация и дегидрирование (реакция Лебедева)
Дегидратация и дегидрирование - 2CH3-CH2 - OH → CH2=CH-CH=CH2 + 2H2O + H2
образуются алкадиены
бутадиен-1,3
Характерные химические свойства предельных многоатомных спиртов
Сходство этиленгликоля и глицерина с одноатомными спиртами:
1. Так, они реагируют с активными металлами:
2. Многоатомные спирты в реакции с галогеноводородами обменивают одну или
несколько гидроксильных групп ОН на атомы галогена:
3.Глицерин взаимодействует с азотной кислотой с образованием сложных эфиров.
В зависимости от условий реакции (мольного соотношения реагентов, концентрации
Кочулева Людмила Рамановна, учитель химии МОБУ «Средняя общеобразовательная школа №73» г.
Оренбурга. Опорные конспекты и алгоритмы для подготовки к ЕГЭ.
катализатора – серной
тринитроглицериды:
кислоты
и
температуры)
получаются
моно-,
ди-
и
Отличия этиленгликоля и глицерина от одноатомных спиртов:
1.
Качественная реакция многоатомных спиртов, позволяющая отличить
соединения этого класса, – взаимодействие со свежеприготовленным гидроксидом
меди(II). В щелочной среде при достаточной концентрации гликоля голубой осадок
Cu(OH)2 растворяется с образованием раствора ярко-синего цвета – гликолята меди(II):
Окисление этиленгликоля
Двухатомный спирт, этиленгликоль HOCH2–CH2OH, при нагревании в кислой
среде с раствором KMnO4 или K2Cr2O7 легко окисляется до щавелевой кислоты, а в
нейтральной – до оксалата калия.
5СН2(ОН) – СН2(ОН) + 8КMnO4-+12H2SO4→5HOOC – COOH +8MnSO4 +4К2SO4
+22Н2О
3СН2(ОН) – СН2(ОН) + 8КMnO4→3KOOC – COOK +8MnO2 +2КОН +8Н2О
Химические свойства фенола
Кислотные свойства фенола(карболовой кислоты) выражены сильнее, чем у воды и
предельных спиртов, что связано с большей полярностью O—H связи и с большей
устойчивостью образующегося при ее разрыве фенолят-иона. Однако кислотные свойства
фенола выражены слабее, чем у карбоновых кислот и, тем более, у сильных
неорганических.
I.
1.
(горение)
II.
Тип реакции
Окисление
Полное
окисление
Уравнение
С6Н5ОН+7О2
→ 6СО2 + ЗН2О
Реакции с участием ФГ
Кочулева Людмила Рамановна, учитель химии МОБУ «Средняя общеобразовательная школа №73» г.
Оренбурга. Опорные конспекты и алгоритмы для подготовки к ЕГЭ.
1.
Реагируют
со
2C6H5ОH+2Na → 2C6H5ONa + H2
щелочными
и
щелочноземельными металлами,
образуя феноляты (сходство со
спиртами).
2.
Реагируют
с
C6H5OH+NaOH → C6H5ONa + H2О
растворами щелочей, образуя
феноляты (отличие от спиртов).
3.
Качественная
C6H5OH + FeCl3 → (C6H5O)3Fe + 3HCl
реакция с хлоридом железа(III).
фенолят железа
Образуется
комплексное
соединение фиолетового цвета фенолят железа.
III.
Замещение в бензольном кольце
1. Реакция с бромной водой
(качественная
реакция).
Галогенирование
идёт
при
обычных условиях.
+ 3Br2 →
+3HBr
Или С6Н5ОН+ ЗВг2 → С6Н2Вг3ОН + ЗНВr
Образуется 2,4,6-трибромфенол - осадок беложёлтого цвета.
2.
Нитрование
(при
комнатной
температуре),
применяется нитрующая смесь –
смесь конц. азотной и серной
кислот при нагревании.
+ 3HNO3(конц.) →
Образуется 2,4,6-тринитрофенол
кислота)- С6Н5(NО2)3OН.
+3H2O
(пикриновая
3.
Поликонденсация
фенола с формальдегидом (по этой
реакции происходит образование
фенолформальдегидной смолы).
А17. Основные способы получения углеводородов (в лаборатории). Основные
способы получения кислородсодержащих соединений (в лаборатории).
Получение алканов
Промышленные сособы:
1)
Выделяют из природных источников (природный и попутный газы, нефть,
каменный уголь).
2)
Гидрирование алкенов и непредельных углеводородов.
Лабораторные способы получения метана:
1)
Термокаталитическое восстановление оксидов углерода(t, Ni):
CO + 3H2 → CH4 + H2O
CO2 + 4H2 → CH4 + 2H2O
Кочулева Людмила Рамановна, учитель химии МОБУ «Средняя общеобразовательная школа №73» г.
Оренбурга. Опорные конспекты и алгоритмы для подготовки к ЕГЭ.
2)
Синтез из простых веществ: С + 2Н2→ СН4
3)
Гидролиз карбида алюминия: Al4C3 + 12H2O → 4Al(OH)3 + 3CH4
Лабораторные способы получения гомологов метана:
1)
Декарбоксилирование натриевых солей карбоновых кислот (реакция Дюма).
Образующийся алкан содержит на один атом углерода меньше, чем исходная соль.
CH3COONa + NaOH → CH4 + Na2CO3
2)
Синтез Вюрца (удвоение цепи); проводят с целью получения алканов с
более длинной углеродной цепью.
2CH3Cl + 2Na → C2H6 + 2NaCl
3)
Электролиз ацетата натрия:
электролиз
2 CH3COONa +2H2O → С2Н6 + 2СО2 + Н2 + 2 NaOH
Получение алкенов
В лаборатории:
1. Дегидрогалогенирование галогеноалканов производится спиртовым раствором
щёлочи:
CH3 – CH2Cl +
KOH(спирт.)
→ CH2 = CH2 + KCl + H2O
CH3 – CH – CH2 – CH3 + KOH (спирт)
→ CH3 – CH = CH – CH3 +
KI +
H2O
|
I
Правило А.М. Зайцева: «Водород отщепляется от менее гидрогенизированного
атома углерода».
2.Дегидратация спиртов протекает в присутствии концентрированной серной
кислоты или безводного оксида алюминия при нагревании (t > 150 оС) с образованием
алкенов.
CH3 – CH2 – CH2OH → CH3 – CH = CH2 + H2O
3.Дегалогенирование
дигалогенопроизводных
производят
с
помощью
мелкораздробленного цинка или магния:
CH3 – CH – CH2 + Zn → CH3 – CH = CH2 + ZnCl2
|
|
Cl
Cl
В промышленности:
1, Основной способ получения алкенов – крекинг алканов, приводящий к
образованию смеси низкомолекулярных алкенов и алканов, которую можно разделить
перегонкой.
С5Н12 → С2Н4 + С3Н8 (или С3Н6 + С2Н6) и др.
2 Дегидрирование алканов. (катализаторы: Pt; Ni; AI2O3 ;Cr2O3)
Ni, 450 – 5000C
СН3 – СН3
→ СН2 = СН2 + Н2
550 – 6500С
2CH4
→ CH2 = СН2 + 2Н2
3. Каталитическое гидрирование алкинов (катализаторы: Pt; Ni; Pd)
СН ≡ СН + Н2 → СН2 = СН2
Кочулева Людмила Рамановна, учитель химии МОБУ «Средняя общеобразовательная школа №73» г.
Оренбурга. Опорные конспекты и алгоритмы для подготовки к ЕГЭ.
Получение циклоалканов
1)
Действием активного металла на дигалогеналкан:
t, p, Ni
Br – CН2-CН2-CН2-Br + Mg →
+ Mg Br2
1,3-дибромпропан
2)
Гидрирование аренов ( t, p, Pt)
С6Н6 + 3 Н2 →
Получение алкинов
Ацетилен:
а) метановый способ:
1500С
2СН4  С2Н2 + 3Н2
б) гидролиз карбида кальция (лабораторный способ):
CaC2 + 2H2O  C2H2 + Ca(OH)2
2500С:
CaO + 3C  CaC2 + CO
Вследствие большой энергоемкости этот метод экономически менее выгоден.
Синтез гомологов ацетилена:
а) каталитическое дегидрирование алканов и алкенов:
СnH2n+2  CnH2n-2 + 2H2
СnH2n  CnH2n-2 + H2
б) дегидрогалогенирование дигалогеналканов спиртовым раствором щелочи
(щелочь и спирт берутся в избытке):
СnH2nГ2 + 2KOH(сп)  CnH2n-2 + 2KГ + 2H2O
Получение алкадиенов
1)
Дегидрированием алканов, содержащихся в природном газе и газах
нефтепереработки,
при
пропускании
их
над
нагретым
катализатором
t, Cr2O3,Al2O3
CH3–CH2–CH2–CH3 → CH2=CH–CH=CH2 + 2H2
t, Cr2O3,Al2O3
CH3–CH–CH2–CH3 → CH2= C–CH=CH2 + 2H2
CH3
CH3
2)
Дегидрированием и дегидратацией этилового спирта при пропускании
паров спирта над нагретыми катализаторами (метод акад. С.В.Лебедева):
t, ZnO, Al2O3
2CH3CH2OH → CH2= CH–CH = CH2 + 2H2O + H2
Получение аренов
Бензол
Тримеризация алкинов над активированным углем (Зелинский):
акт. С, 600 С
3НCCH  С6H6 (бензол)
2)
В лаборатории сплавлением солей бензойной кислоты со щелочами:
С6Н5 – СООNa + NaОН → С6Н6 + Na2СО3
1)
Кочулева Людмила Рамановна, учитель химии МОБУ «Средняя общеобразовательная школа №73» г.
Оренбурга. Опорные конспекты и алгоритмы для подготовки к ЕГЭ.
Бензол и гомологи
1)
При коксовании каменного угля образуется каменноугольная смола, из
которой выделяют бензол, толуол, ксилолы, нафталин и многие другие органические
соединения.
2)
Дегидроциклизация (дегидрирование и циклизация) алканов в присутствии
катализатора:
Cr2O3
CH3-CH2-CH2-CH2-CH2-CH3  C6H6 + 4H2
Из гексана получается бензол, а из гептана- толуол.
3)
Дегидрирование циклоалканов
→ С6Н6 + 3 Н2
4)
Получение гомологов - алкилирование бензола галогеналканами или
алкенами в присутствии безводного хлорида алюминия:
AlCl3
C6H6 + C2H5Cl  C6H5C2H5 + HCl
хлорэтан
этилбензол
Получение предельных одноатомных спиртов
Общие способы
1)
Гидратация алкенов (по правилу Марковникова):
t, H2SO4
СН3-СН=СН2 + Н-ОН→ СН3-СН-СН3
ОН (пропанол-2)
Гидролиз галогеналканов при действии водного раствора щёлочи:
t
C2H5I + NaОН (водн.) → C2H5-OН + NaI
3)
Восстановление (гидрирование) альдегидов и кетонов.
При гидрировании альдегтдов образуются первичные спирты:
t, Ni
СН3-СН2-СНО + Н2 → СН3-СН2- СН2-ОН
пропанол-1
При гидрировании кетонов образуются вторичные спирты:
t, Ni
СН3-С-СН3+ Н2→ СН3-СН-СН3
2)
О
ОН (пропанол-2)
Специфические способы получения
1)
Метанол – из синтез-газа:
t, р, кат
СО + 2Н2 → СН3ОН
2)
Этанол – спиртовым брожением глюкозы (ферментативное):
C6H12O6 → 2C2H5OH + 2CO2
Этиленгликоль
1)
В лаборатории - реакция Вагнера.
Кочулева Людмила Рамановна, учитель химии МОБУ «Средняя общеобразовательная школа №73» г.
Оренбурга. Опорные конспекты и алгоритмы для подготовки к ЕГЭ.
Окисление этилена перманганатом калия в нейтральной среде приводит к
образованию двухатомного спирта – этиленгликоля.
Упрощённо:
+ KMnO4, H2O
CH2 = CH2 + НОН + [O] → CH2 – CH2
|
|
OH OH
3 CH2 = CH2 + 2KMnO4 + 4H2O → 3 CH2 – CH2 + 2MnO2 + 2KOH
|
|
OH OH
2)
В промышленности – гидролизом 1,2 –дихлорэтана:
СН2Cl- СН2Cl + 2NaOH → СН2(ОН)-СН2ОН + 2NaCl
Глицерин
1)
Гидролиз жиров:
2)
Из пропена:
а) СН2 = СН-СН3 + Cl2 → СН2 = СН-СН2 Cl
3-хлорпропен-1
б) СН2 = СН-СН2 Cl + NaOH (водн.)→ СН2 = СН-СН2-ОН + NаCl
аллиловый спирт
в) СН2 = СН-СН2-ОН + Н2О2→ СН2 -СН- СН2
ОН ОН ОН
Получение фенолов
1)
Выделение из каменноугольной смолы.
2)
Гидролиз хлорбензола:
С6Н5-Cl + Н2О (пар) → С6Н5-ОН + НCl
3)
Окисление изопропилбензола (кумола) кислородом воздуха:
Получение простых эфиров
Межмолекулярная дегидратация этанола:
t, H2SO4
2C2H5ОH → C2H5-O-C2H5 +Н2О
1)
Кочулева Людмила Рамановна, учитель химии МОБУ «Средняя общеобразовательная школа №73» г.
Оренбурга. Опорные конспекты и алгоритмы для подготовки к ЕГЭ.
2)
Взаимодействием алкоголята металла с галогенпроизводными алканов:
C2H5I + C2H5ONa → C2H5-O-C2H5 + NaI
Получение альдегидов
Общий способ
1)
Окисление спиртов. Первичные спирты окисляются до альдегидов, а
вторичные – до кетонов:
t, Cu
2C2H5OH + O2 → 2CH3CHO + 2H2О
t, Cu
СН3-СН-СН3 + O2 → СН3-С-СН3
ОН (пропанол-2)
О
Специфические способы
1)
Формальдегид получают каталитическим окислением метана:
CH4 + O2 → НCНO + H2O
2)
Уксусный альдегид (ацетальдегид):
а) реакцией Кучерова
Н+, Hg2+
HCCH + Н2О  СН3-СНО
б) катилитическим окислением этилена
2СН2=СН2 + О2 → 2СН3-СНО
Получение карбоновых кислот
Общие способы
1)
Окисление альдегидов под действием различных окислителей:
t
R-CHO + Ag2O (амм.) → R-CОOH +2Ag↓
t
R- CHO + 2Cu(OH)2 →R-COOH + Cu2O↓ + 2H2O
2)
Каталитическое окисление - гомологи метана окисляются с разрывом С-С
цепи и образованием карбоновых кислот:
2C4H10 + 5O2 → 4СН3COOН+ 2H2O
Специфические способы
1)
Муравьиную кислоту получают нагреванием
под давлением порошкообразного гидроксида натрия и угарного газа с последующей
обработкой полученного формиата натрия сильной кислотой:
NaOH + CO → HCOONa
H2SO4 + 2HCOONa→ HCOOН + Na2SO4
2)
Уксусную кислоту:
а) Для пищевых целей получают ферментативным брожением (окислением)
жидкостей, содержащих спирт (вино, пиво):
ферменты
C2H5OH + О2 → CH3CОOH + H2О
б) В лаборатории из ацетатов:
2СН3COONa + H2SO4 → 2СН3COOН + Na2SO4
Получение сложных эфиров
Кочулева Людмила Рамановна, учитель химии МОБУ «Средняя общеобразовательная школа №73» г.
Оренбурга. Опорные конспекты и алгоритмы для подготовки к ЕГЭ.
1)
Реакция этерификации при нагревании кислоты и спирта в присутствии
серной кислоты или других минеральных кислот. Изотопными исследованиями показано,
что в реакции этерификации от молекулы спирта отделяется атом водорода, а от молекулы
кислоты - гидроксильная группа.
Эта реакция обратима и подчиняется правилу Ле-Шателье. Для увеличения выхода
сложных эфиров необходимо удалять из реакционной среды образующуюся воду.
CH3-CООН + НOCН2CH3 → CH3-CО-O- CН2CH3 + H2O
Получение мыла
1)
Щелочной гидролиз (омыление жиров происходит под действием щелочей
необратимо):
2)
Нейтрализация
карбоновых
кислот,
полученных
окислением высших парафинов нефти:
2 С32Н66 + 5О2→ 4 С15Н31СООН + 2Н2О
пальмитиновая кислота
С15Н31СООН + NaOH → С15Н31СООNa + Н2O
пальмитат натрия (твёрдое мыло)
С15Н31СООН + КOH → С15Н31СООК + Н2O
пальмитат калия (жидкое мыло)
каталитическим
Получение углеводов
1)
Глюкозу - гидролизом крахмала или целлюлозы:
(C6H10O5)n + nH2O
nC6H12O6
2)
Сахарозу - из сахарной свеклы и сахарного тростника.
Кочулева Людмила Рамановна, учитель химии МОБУ «Средняя общеобразовательная школа №73» г.
Оренбурга. Опорные конспекты и алгоритмы для подготовки к ЕГЭ.
А18. Взаимосвязь углеводородов и кислородосодержащих органических
соединений.
А19.Классификация химических реакций в неорганической и органической
химии
Химические реакции – это процессы, в результате которых из одних веществ
образуются другие, отличающиеся от них по составу и (или) строению.
Классификация реакций:
I.
По числу и составу реагирующих веществ и продуктов реакции:
1)
Реакции, идущие без изменения состава вещества:
Кочулева Людмила Рамановна, учитель химии МОБУ «Средняя общеобразовательная школа №73» г.
Оренбурга. Опорные конспекты и алгоритмы для подготовки к ЕГЭ.
В неорганической химии это реакции превращения одних аллотропных
модификаций в другие:
C (графит) → C (алмаз); P (белый) → P (красный).
В органической химии это реакции изомеризации – реакции, в результате которых
из молекул одного вещества образуются молекулы других веществ того же качественного
и количественного состава, т.е. с той же молекулярной формулой, но другим строением.
t, AlCl3
-СН2-СН2-СН3 → СН3-СН-СН3
СН3
н-бутан
2-метилпропан (изобутан)
2)
Реакции, идущие с изменением состава вещества:
а) Реакции соединения (в органической химии присоединения) – реакции, в ходе
которых из двух и более веществ образуется одно более сложное: S + O2→ SO2
В органической химии это реакции гидрирования, галогенирования,
гидрогалогенирования, гидратации, полимеризации.
t, H2SO4
СН2 = СН2 + НОН → СН3 – СН2ОН
б) Реакции разложения (в органической химии отщепления, элиминирования) –
реакции, в ходе которых из одного сложного вещества образуется несколько новых
веществ:
t, H2SO4
СН3 – СН2ОН → СН2 = СН2 + Н2О
2KNO3 →2KNO2 + O2
В органической химии примеры реакций отщепления - дегидрирование,
дегидратация, дегидрогалогенирование, крекинг.
в) Реакции замещения – реакции, в ходе которых атомы простого вещества
замещают атомы какого-нибудь элемента в сложном веществе (в органической химии –
реагентами и продуктами реакции часто являются два сложных вещества).
CH4 + Cl2 → CH3Cl +HCl ; 2Na+ 2H2O→ 2NaOH + H2
Примеры реакций замещения, не сопровождающихся изменением степеней
окисления атомов, крайне немногочисленны. Следует отметить реакцию оксида кремния с
солями кислородсодержащих кислот, которым отвечают газообразные или летучие
оксиды:
СаСО3+ SiO2 = СаSiO3 + СО2
Са3(РО4)2 + ЗSiO2 = ЗСаSiO3 + Р2О5
г) Реакции обмена – реакции, в ходе которых два сложных вещества обмениваются
своими составными частями:
NaOH
+
HCl
→
NaCl
+
H2O,
2CH3COOH + CaCO3 → (CH3COO)2Ca + CO2 + H2O
II.
По изменению степеней окисления химических элементов, образующих
вещества
1)
Реакции, идущие с изменением степеней окисления, или ОВР:
∙2| N+5 + 3e– → N+2 (процесс восстановления, элемент – окислитель),
∙3| Cu0 – 2e– → Cu+2 (процесс окисления, элемент – восстановитель),
8HNO3 + 3Cu → 3Cu(NO3)2 + 2NO + 4H2O.
В органической химии:
C2H4 + 2KMnO4 + 2H2O → CH2OH–CH2OH + 2MnO2 + 2KOH
2)
Реакции, идущие без изменения степеней окисления химических элементов:
Кочулева Людмила Рамановна, учитель химии МОБУ «Средняя общеобразовательная школа №73» г.
Оренбурга. Опорные конспекты и алгоритмы для подготовки к ЕГЭ.
Li2O
+
H2 O
→
2LiOH,
HCOOH + CH3OH → HCOOCH3 + H2O
III.
По тепловому эффекту
1)
Экзотермические реакции протекают с выделением энергии:
С
+
О2
→
СО2
+
Q,
СH4 + 2O2 → CO2 + 2H2O + Q
2)
Эндотермические реакции протекают с поглощением энергии:
СaCO3 → CaO + CO2 - Q
C12H26 → C6H14 + C6H12 - Q
IV.
По агрегатному состоянию реагирующих веществ
1)
Гетерогенные реакции – реакции, в ходе которых реагирующие вещества и
продукты реакции находятся в разных агрегатных состояниях:
Fe(тв)
+
CuSO4(р-р)
→
Cu(тв)
+
FeSO4(р-р),
CaC2(тв) + 2H2O(ж) → Ca(OH)2(р-р) + C2H2(г)
2)
Гомогенные реакции – реакции, в ходе которых реагирующие вещества и
продукты реакции находятся в одном агрегатном состоянии:
H2(г)
+
Cl2(г)
→
2HCl(г),
2C2H2(г) + 5O2(г) → 4CO2(г) + 2H2O(г)
V.
По участию катализатора
1)
Некаталитические реакции, идущие без участия катализатора:
2Н2 + О2 → 2Н2О, С2Н4 + 3О2 → 2СО2 + 2Н2О
2)
Каталитические реакции, идущие с участием катализаторов:
MnO2
2H2O2 → 2H2O + O2
VI.
По направлению
1)
Необратимые реакции протекают в данных условиях только в одном
направлении:
С2Н4 + 3О2 → 2СО2 + 2Н2О
2)
Обратимые реакции в данных условиях протекают одновременно в двух
противоположных направлениях: N2 + 3H2 ↔2NH3
VII.
По механизму протекания
1)
Радикальный механизм.
А : В → А· + ·В
Происходит гомолитический (равноценный) разрыв связи. При гемолитическом
разрыве пара электронов, образующая связь, делится таким образом, что каждая из
образующихся частиц получает по одному электрону. При этом образуются радикалы –
незаряженные частицы с неспаренными электрономи. Радикалы – очень
реакционноспособные частицы, реакции с их участием происходят в газовой фазе с
большой скоростью и часто со взрывом.
Радикальные реакции идут между образующимися в ходе реакции радикалами и
молекулами:
2H2O2 → 2H2O + O2
hυ
CH4 + Cl2 → CH3Cl +HCl
Примеры: реакции горения органических и неорганических веществ, синтез воды,
аммиака, реакции галогенирования и нитрования алканов, изомеризация и ароматизация
алканов, каталитическое окисление алканов, полимеризация алкенов, винилхлорида и др.
2)
Ионный механизм.
А : В → :А- + В+
Кочулева Людмила Рамановна, учитель химии МОБУ «Средняя общеобразовательная школа №73» г.
Оренбурга. Опорные конспекты и алгоритмы для подготовки к ЕГЭ.
Происходит гетеролитический (неравноценный) разрыв связи, при этом оба
электрона связи остаются с одной из ранее связанных частиц. Образуются заряженные
частиц (катионы и анионы).
Ионные реакции идут в растворах между уже имеющимися или образующимися в
ходе реакции ионами.
Например, в неорганической химии – это взаимодействие электролитов в растворе,
в органической химии – это реакции присоединения к алкенам, окисление и
дегидрирование спиртов, замещение спиртовой группы и другие реакции,
характеризующие свойства альдегидов и карбоновых кислот.
VIII.
По виду энергии, инициирующей реакцию:
1)
Фотохимические реакции происходят при воздействии квантов света.
Например, синтез хлороводорода, взаимодействие метана с хлором, получение озона в
природе, процессы фотосинтеза и др.
2)
Радиационные реакции инициируются излучениями больших энергий
(рентгеновскими лучами, γ-лучами).
3)
Электрохимические реакции инициирует электрический ток, например, при
электролизе.
4)
Термохимические реакции инициируются тепловой энергией. К ним
относятся все эндотермические реакции и множество экзотермических, для инициации
которых необходима теплота.
А20. Скорость реакции, ее зависимость от различных факторов
I.
Гетерогенными называются реакции, когда реагирующие вещества
находятся в разных фазах:
o
твердое вещество с другим твердым, жидкостью или газом,
o
две несмешивающиеся жидкости,
o
жидкость с газом.
Гомогенные реакции протекают между веществами в одной фазе:
o
между хорошо смешивающимися жидкостями,
o
газами,
o
веществами в растворах.
II.
Скорость химической реакции - изменение концентрации одного из веществ
в единицу времени:
n2
–
n1
Δn
Δс
υ
=
––––––––––
=
––––––––
=
––––––––
(t2 – t1) • V
Δt • V
Δt
где c = n/V – концентрация вещества,
Δ (читается «дельта») – общепринятое обозначение изменения величины.
III.
Скорость гетерогенных реакций (с участием твердых веществ) часто
выражают на единицу площади соприкасающихся поверхностей:
Δn
υ
=
––––––
Δt • S
IV.
Факторы, влияющие на скорость химических реакций.
1)
Природа реагирующих веществ (состав, строение). Разные вещества
реагируют с разной скоростью. Например, цинк бурно реагирует с соляной кислотой, а
железо довольно медленно. Натрий взаимодействует с водой быстрее лития.
Кочулева Людмила Рамановна, учитель химии МОБУ «Средняя общеобразовательная школа №73» г.
Оренбурга. Опорные конспекты и алгоритмы для подготовки к ЕГЭ.
2)
Концентрация веществ. Скорость реакции тем больше, чем выше
концентрация веществ. С сильно разбавленной кислотой цинк будет реагировать
значительно дольше, чем с концентрированной.
3)
Температура. Скорость реакции значительно повышается с повышением
температуры. Например, для горения топлива необходимо его поджечь, т.е. повысить
температуру. Для многих реакций повышение температуры на 10° C сопровождается
увеличением скорости в
2-4 раза (правило Вант-Гоффа).
4)
Поверхность соприкосновения твёрдых реагентов. Скорость гетерогенных
реакций увеличивается с увеличением поверхности реагирующих веществ. Для этого
твёрдые вещества измельчают. Мрамор в порошке быстрее взаимодействует с соляной
кислотой, чем кусочек мрамора той же массы. Порошки железа и серы при нагревании
быстрее вступят в реакцию, если железо будет в виде мелких опилок.
Агрегатные
Поверхность соприкосновения
Скорость реакции
состояния
реагентов
Жидк. + жидк. Максимальна, т.к. взаимодействуют ионы Очень большая
(растворы)
Тв. + жидк.
Реакция идёт на поверхности раздела Большая
реагентов
Твёрд. + твёрд.
Реакция идёт на поверхности
Малая
Газ + газ
Малая, т.к. большие промежутки между Малая
молекулами
5)
Наличие катализаторов или ингибиторов.
Катализаторы – вещества, ускоряющие химические реакции, но сами при этом не
расходующиеся. Пример – бурное разложение перекиси водорода при добавлении
катализатора – оксида марганца (IV): 2H2O2 = 2H2O + O2↑
Оксид марганца (IV) остается на дне, его можно использовать повторно.
Ингибиторы – вещества, замедляющие реакцию. Например, для продления срока
службы труб и батарей в систему водяного отопления добавляют ингибиторы коррозии. В
автомобилях ингибиторы коррозии добавляются в тормозную, охлаждающую жидкость.
6)
Давление (только для реакций между газами). Чем выше давление, тем
больше скорость реакции.
Массы веществ (количества) не влияют на скорость реакции.
А21. Обратимые и необратимые химические реакции. Химическое равновесие.
Смещение равновесия под действием различных факторов.
1)
Необратимыми называются химические реакции, протекающие только в
одном направлении, т.е. продукты этих реакций не взаимодействуют друг с другом с
образованием исходных веществ. Условия необратимости реакции – образование осадка,
газа или слабого электролита. Например:
BaCl2 + H2SO4 = BaSO4 + 2HCl
K2S + 2HCl = 2KCl + H2S
HCl + NaOH = NaCl + H2O.
Обратимыми называются реакции, протекающие одновременно в прямом и
обратном направлениях, например:
Кочулева Людмила Рамановна, учитель химии МОБУ «Средняя общеобразовательная школа №73» г.
Оренбурга. Опорные конспекты и алгоритмы для подготовки к ЕГЭ.
2)
Химическое равновесие – это состояние системы, при котором скорость
прямой реакции равна скорости обратной реакции.
3)
Принцип Ле Шателье: если на систему, находящуюся в равновесии,
производится внешнее воздействие (изменяется температура, давление или
концентрация), то равновесие смещается в том направлении, которое ослабляет это
воздействие.
o
Влияние температуры.
При увеличении температуры равновесие смещается в сторону эндотермической
реакции (-Q). При уменьшении температуры равновесие смещается в сторону
экзотермической реакции (+Q).
o
Влияние концентрации.
Увеличение концентрации исходного вещества приводит к смещению равновесия
вправо, в сторону реакции, в которой это вещество расходуется.
Увеличение концентрации продукта приводит к смещению равновесия влево, в
сторону реакции, в которой это вещество расходуется.
Уменьшение концентрации исходного вещества приводит к смещению равновесия
влево, в сторону реакции, в которой это вещество образуется.
Уменьшение концентрации продукта приводит к смещению равновесия вправо, в
сторону реакции, в которой это вещество образуется.
o
Влияние давления.
Для реакций, в которых принимают участие газообразные и жидкие вещества,
увеличение давления приводит к смещению равновесия в сторону реакции, ведущей к
уменьшению объёма. Уменьшение давления приводит к смещению равновесия в сторону
реакции, ведущей к увеличению объёма.
Если количество молей до и после реакции одинаково, то изменение давления
никак не влияет на смещение равновесия.
Например, в реакции увеличение давления смещает равновесие вправо.
Объёмы твёрдых веществ принимаются за 0, т.к. твёрдые вещества практически не
сжимаемы.
o
Катализатор не смещает химическое равновесие, а только ускоряет его
достижение.
А22. Электролитическая диссоциация электролитов в водных растворах.
Сильные и слабые электролиты.
ие
Электролиты и неэлектролиты.
Определен
Электролиты
–
вещества,
Неэлектролиты
–
которые проводят электрический ток в вещества,
которые
не
растворах. Электролиты в растворах проводят электрический ток
распадаются на заряженные частицы – в
растворах.
ионы, которые могут передвигаться к
электродам. Это и есть причина
электрического тока в растворах.
Кочулева Людмила Рамановна, учитель химии МОБУ «Средняя общеобразовательная школа №73» г.
Оренбурга. Опорные конспекты и алгоритмы для подготовки к ЕГЭ.
Тип
химической связи
Примеры
ионная,
сильнополярная
соли, кислоты, основания
ковалентная
неполярная и слабополярная
органические
вещества, газы,
оксиды
металлов и неметаллов
Классификация электролитов.
Электро
литы
Сильные
Средние
Слабые α < 3%
Степень диссоциации α
3% < α >
(на ионы распадается только
> 30%
30%
часть растворенного вещества)
(полный распад на
ионы)
Все соли, NaOH, KOH,
Mg(OH)
NH3·H2O, Al(OH)3, Fe(OH)3,
Ca(OH)2,
,
H
SO
,
HNO
,
H
CO
,
H
2
2
3
2
2
3
2SiO3, H2S, H2O, CH3COOH
Ba(OH)2, HCl, H2SO4,
H3PO4
HNO2
H+ + NO2−
HNO3
CH3COOH
H+ + CH3COO−
NaOH = Na+ + OH−
NH3 · H2O
NH4+ + OH−
Ba(OH)2 = Ba2+ + 2 OH−
K2SO4 = 2K+ + SO42−
Na2S = 2Na+ + S2−
2)
Ионы - заряженные частицы.
Классификация ионов:
o
По составу различают простые (Na+) и сложные (OH−) ионы.
o
По знаку заряда - на положительные ионы (катионы) и отрицательные
(анионы). Если через раствор или расплав электролита пропускать электрический ток, то
положительные ионы будут двигаться к отрицательному электроду – катоду.
Положительные ионы получили название катионы. Отрицательные ионы будут двигаться
к положительному электроду – аноду, и называются анионами.
4) Электролитическая диссоциация – процесс распада электролита на ионы при
растворении в воде или расплавлении.
3)
Электролитическая диссоциация кислот, щелочей и солей (средних).
o
Кислоты - это электролиты, которые при диссоциации образуются только
катионы водорода:
H2SO4 = 2H+ + SO42−.
Диссоциация многоосновной кислоты протекает главным образом по
первой ступени,
в меньшей степени по второй и лишь в незначительной степени
по третьей.
Н3РО4
Н+ + Н2РО-4 (первая ступень)
Н2РО-4
Н+ + НРO2-4 (вторая ступень)
НРО2-4
Н+ PОЗ-4 (третья ступень)
o
Основания - это электролиты, которые при диссоциации образуют катионы
металла и гидроксид-анионы:
KOH = K+ + OH−,
Ca(OH)2 = Ca2+ + 2OH−
Двух- и многокислотные основания диссоциируют ступенчато:
Ca(ОН)2
Са(ОН)+ + OH- (первая ступень)
Ca(OH)+
Ca2++OH- (вторая ступень)
1)
Кочулева Людмила Рамановна, учитель химии МОБУ «Средняя общеобразовательная школа №73» г.
Оренбурга. Опорные конспекты и алгоритмы для подготовки к ЕГЭ.
o
Соли - это электролиты, которые в водном растворе диссоциируют с
образованием катионов металла (а также катионов аммония (NH4+)) , и анионов
кислотного остатка.
CuSO4 = Cu2+ + SO42−
BaCl2 = Ba2+ + 2Cl−
(NH4)2SO4 = 2NH4+ +SO42−
А23. Реакции ионного обмена и условия их осуществления.
1)
Реакция ионного обмена — реакции между двумя сложными веществамиэлектролитами в растворах, в результате которых они обмениваются своими ионами.
2)
Реакцию обмена в растворе принято изображать тремя уравнениями:
молекулярным, полным ионным и сокращённым ионным. В ионном уравнении слабые
электролиты, газы и малорастворимые вещества изображают молекулярными формулами.
Na2CO3 + H2SO4 → Na2SO4 + CO2↑ +H2O
2Na+ + CO32- + 2H+ + SO42- → 2Na+ + SO42- + CO2↑ + H2O
CO32- + 2H+ → CO2↑ + H2O
3)
Правила написания уравнений реакций в ионном виде:
o
Записывают формулы веществ, вступивших в реакцию, ставят знак «равно»
и записывают формулы образовавшихся веществ. Расставляют коэффициенты.
o
Пользуясь таблицей растворимости, записывают в ионном виде формулы
веществ, обозначенных в таблице растворимости буквой «Р» (хорошо растворимые в
воде), исключение – гидроксид кальция, который, хотя и обозначен буквой «М», все же в
водном растворе хорошо диссоциирует на ионы.
o
Нужно помнить, что на ионы не разлагаются металлы, оксиды металлов и
неметаллов, вода, газообразные вещества, нерастворимые в воде соединения,
обозначенные в таблице растворимости буквой «Н». Формулы этих веществ записывают в
молекулярном виде. Получают полное ионное уравнение.
o
Сокращают одинаковые ионы до знака «равно» и после него в уравнении.
Получают сокращенное ионное уравнение.
4)
Правило Бертолле: реакции ионного обмена протекают до конца, если в
результате реакции образуется вода, газ или осадок.
1. Если в результате реакции выделяется малодиссоциирующее вещество –
вода.
а) Реакция щелочи с кислотой:
KOH + HCl = KCl + H2O
K+ + OH– + H+ + Cl– = K+ + Cl– + H2O
H+ + OH– = H2O
б) Реакция основного оксида с кислотой:
CaO + 2HNO3 = Ca(NO3)2 + H2O
CaO + 2H+ + 2NO3- = Ca2+ + 2NO3- + H2O
CaO + 2H+ = Ca2+ + H2O.
в) Реакция нерастворимого основания с кислотой:
3Mg(OH)2 + 2H3PO4 = Mg3(PO4)2 + 6H2O
3Mg(OH)2 + 6H+ + 2PO43- = Mg3(PO4)2 + 6H2O
В данном случае полное ионное уравнение совпадает с сокращенным ионным
уравнением.
г) Реакция амфотерного оксида с кислотой:
Al2O3 + 6HCl = 2AlCl3 + 3H2O
Кочулева Людмила Рамановна, учитель химии МОБУ «Средняя общеобразовательная школа №73» г.
Оренбурга. Опорные конспекты и алгоритмы для подготовки к ЕГЭ.
Al2O3 + 6H+ + 6Cl– = 2Al3+ + 6Cl– + 3H2O
Al2O3 + 6H+ = 2Al3+ + 3H2O
2. Если в результате реакции выделяется нерастворимое в воде вещество.
а) Реакция растворимой соли со щелочью:
CuCl2 + 2KOH = 2KCl + Cu(OH)2
Cu2+ + 2Cl– + 2K+ + 2OH– = 2K+ + 2Cl– + Cu(OH)2
Cu2+ + 2OH– = Cu(OH)2
б) Реакция двух растворимых солей, если в результате образуется хотя бы одна
нерастворимая соль:
Al2(SO4)3 + 3BaCl2 = 3BaSO4 + 2AlCl3
2Al3+ + 3SO42- + 3Ba2+ + 6Cl- = 3BaSO4 + 2Al3+ + 6ClBa2+ + SO42- = BaSO4
в) Реакция нерастворимого основания с кислотой:
Fe(OH)3 + H3PO4 = FePO4 + 3H2O
Fe(OH)3 + 3H++ PO43- = FePO4 + 3H2O
В данном случае полное ионное уравнение реакции совпадает с сокращенным. Эта
реакция протекает до конца, о чем свидетельствуют сразу два факта: образование
вещества, нерастворимого в воде, и выделение воды.
3. Если в результате реакции выделяется газообразное вещество.
а) Реакция растворимой соли (сульфида) с кислотой:
K2S + 2HCl = 2KCl + H2S .
2K+ + S2– + 2H+ + 2Cl– = 2K+ + 2Cl– + H2S .
2H+ + S2–= H2S .
Или реакция растворимой соли (карбоната) с кислотой:
Na2CO3 + 2HNO3 = 2NaNO3 + H2O + CO2
2Na+ +CO32- + 2H+ +NO3- = 2Na++ 2NO3- + H2O + CO2
2H++ CO32- = H2O + CO2
О протекании данной реакции до конца свидетельствуют два признака: выделение
воды и газа – оксида углерода(IV).
в) Реакция нерастворимой соли (карбоната) с кислотой:
3СaCO3 + 2H3РO4 = Са3(PO4)2 + 3H2O + 3CO2
3СaCO3 + 6H++ РO43- = Са3(PO4)2 + 3H2O + 3CO2
В данном случае полное ионное уравнение реакции совпадает с сокращенным
уравнением. Эта реакция протекает до конца, о чем свидетельствуют сразу три признака:
выделение газа, образование осадка и выделение воды.
А24. В4. Гидролиз солей. Среда водного раствора солей: кислая, нейтральная,
щелочная
Гидролиз - реакция обменного разложения веществ водой.
Гидролизу подвергаются многие вещества:
1.
Соли
Гидролиз соли – процесс обменного взаимодействия ионов соли с молекулами
воды. Причина – ЭД соли и воды на ионы и взаимодействие между ними. Вода
диссоциирует незначительно на ионы (1 молекула из 550000), причём в процессе
гидролиза один или оба этих иона могут связываться с ионами соли в осадок,
малодиссоциирующее или газообразное вещество.
Кочулева Людмила Рамановна, учитель химии МОБУ «Средняя общеобразовательная школа №73» г.
Оренбурга. Опорные конспекты и алгоритмы для подготовки к ЕГЭ.
На гидролиз влияют химическая природа соли, t.
Случаи гидролиза солей
кислота
основание
сильный электролит: щёлочи
слабый электролит: нерастворимые
(кроме NH4OH)
основания и NH4OH
сильная:
1. Соль образована сильным
3. Соль образована слабым основанием и
H2SO4,
основанием и сильной кислотой:
сильной кислотой:
HNO3,
гидролиз не идёт.
гидролиз по катиону.
HCl,
Среда нейтральная (рН=7).
Среда кислая (рН<7), образуется слабый
HI,
Цвет индикаторов не изменяется.
электролит – основная соль.
HClO4,
Примеры солей – NaCl, K2SO4, Красная окраска лакмуса.
HMnO4
LiNO3
Примеры солей -FeSO4 , Al(NO3)3, ZnCl2
ZnCl2→ Zп2+ +2ClH2O↔H++OHZп2+ +2Cl- + H2O= ZnOH++ H++2ClZnCl2 +H2O= ZnOHCl+ HCl
слабая:
2. Соль образована сильным
4. Соль образована слабым основанием и
HF,
основанием и слабой кислотой:
слабой кислотой.
H3PO4,
гидролиз по аниону.
Гидролиз необратимый - и по катиону, и
H2SiO3,
Среда щелочная (рН>7), образуется по аниону.
H2SiO3,
слабый электролит – кислая соль.
Обычно образуются 2 слабых электролита
H2CO3,
Малиновая окраска ф/ф.
- кислота и нерастворимое основание.
H2S,
Примеры солей - Na2CO3, K2S ,
Среда может быть различной.
CH3COO K2SiO3
Примеры солей - Fe2(CO3)3, Al2S3,
H
Na2CO3→2Na++CO32(NH4)2CO3,CH3COONH4
H2O↔H++OHAl2S3 +6 H2O=2Al(OH)3↓+ 3H2S↑
2Na++ CO32- +H2O =2Na++OH+HCO3Na2CO3 +H2O = NaHCO3 + NaOH
Таблица изменения окраски индикаторов в различных средах:
Индикатор
Кислая среда
Лакмус
Красный
Метилоранж
Розовый
Фенолфталеин
Бесцветный
Универсальный
Краснофиолетовый
Нейтральная
Фиолетовый
Оранжевый
Бесцветный
Жёлтый
Особые случаи гидролиза: некоторые соли образованные катионами слабых
оснований и анионами слабых кислот в водных растворах не существуют, т. к. в момент
образования они практически полностью разлагаются водой:
2AlCl3 +3Na2CO3 +3H2O=2Al(OH)3↓+6NaCl +3CO2↑
Полный гидролиз таких солей протекает, если продукты гидролиза менее
растворимы, чем сама соль. Из всех возможных продуктов гидролиза образуются
наименее растворимые. Так при гидролизе большинства таких солей образуются не
гидроксиды, а гидроксосоли:
2CuCl2 +2Na2CO3 +H2O=(CuOH)2 CO3 ↓+4NaCl +CO2↑
2.
Карбиды:
CaC2+ 2H2O = Ca(OH)2 + C2H2↑
3.
Нитриды
Mg3N2 + 6H2O = 3Mg(OH)2↓ + 2NH3↑
Кочулева Людмила Рамановна, учитель химии МОБУ «Средняя общеобразовательная школа №73» г.
Оренбурга. Опорные конспекты и алгоритмы для подготовки к ЕГЭ.
4.
Фосфиды
Ca3P2 + 6 H2O=3Ca(OH)2 +2PH3↑
5.
Галогениды
SiCl4 + 3 H2O = H2SiO3↓ + 4HCl↑
6.
Углеводы
C12H22O11+H2O = C6H12O6 (глюкоза) + C6H12O6 (фруктоза)
7.
Сложные эфиры, в т.ч. жиры
А25.Реакции окислительно-восстановительные. Коррозия металлов и способы
защиты от нее.
ОВР протекают с изменением степени окисления элементов. К ОВР относятся все
реакции замещения и те реакции соединения и разложения, в которых участвует хотя бы
одно простое вещество. Все реакции обмена протекают без изменения степени окисления.
Окисление - процесс отдачи электронов, приводящий к повышению степени
окисления. Вещества, атомы или ионы которых отдают электроны, называют восстановителями. Восстановитель, отдавая электроны, окисляется(отдал-окисляетсявосстановитель). Например: H20 - 2ē → 2H+
Восстановление - процесс присоединения электронов, приводящий к понижению
степени окисления. Вещества, атомы или ионы которых присоединяют электроны, называются окислителями. Окислитель, присоединяя электроны, восстанавливается (взялвосстанавливается -окислитель). Например: S0 + 2ē → S-2
Общее число электронов, отданных восстановителем, равно числу электронов,
принятых окислителем.
Пример:
P + 5HNO3 → 5NO2 + H3PO4 + H2O
1 P0 - 5 e- → P+5 окисляется, восстановитель
1 N+5 +5 e- → N0 восстанавливается, окислитель
Типичные восстановители:
Типичные окислители:
Металлы
Галогены
Водород
Перманганат калия(KMnO4)
Уголь С
манганат калия (K2MnO4)
Окись углерода (II) (CO)
оксид марганца (IV) (MnO2)
Сероводород (H2S)
Дихромат калия (K2Cr2O7)
Оксид серы (IV) (SO2)
хромат калия (K2CrO4)
Сернистая кислота H2SO3 и ее соли
Азотная кислота (HNO3)
Галогеноводородные кислоты и их соли
Серная кислота (H2SO4) конц.
Катионы металлов в низших степенях окисления
Оксид меди(II) (CuO)
(SnCl2, FeCl2, MnSO4, Cr2(SO4)3)
оксид свинца(IV) (PbO2)
Азотистая кислота HNO2
оксид серебра (Ag2O)
Аммиак NH3
пероксид водорода (H2O2)
Оксид азота(II) (NO)
Хлорид железа(III) (FeCl3)
Катод при электролизе.
Бертоллетова соль (KClO3)
Кочулева Людмила Рамановна, учитель химии МОБУ «Средняя общеобразовательная школа №73» г.
Оренбурга. Опорные конспекты и алгоритмы для подготовки к ЕГЭ.
Анод при электролизе.
Могут быть и окислителями, и восстановителями вещества, содержащие атомы
в промежуточных степенях окисления, способны как повышать, так и понижать степень
окисления. Являются восстановителями при действии более сильного чем они окислителя;
окислителями - при действии более активного, чем они, восстановителя. Это KNO2,
SO2, H2O2, Na2SO3 и др.
Самые известные полуреакции
Самые известные полуреакции
восстановления окислителей:
окисления восстановителей:
2Cl- - 2 e- → Cl2
Хромат КCrO4 и дихромат калия К2Cr2O7
выступают в качестве окислителей в кислой 2Br- - 2 e- → Br2
среде, восстанавливаясь до иона Сr+3:
2I- - 2 e- → I2
Cr2O72- + H+→ Cr3+
H2S→ S
CrO42- + H+→ Cr3+
Na2SO3 →Na2SO4
КМnО4 проявляет окислительные свойства за S → SO2
счет Мп+7 и восстанавливается: в кислой среде SO2 → SO4 2- до Мn+2, в нейтральной - до МnО2, в NH3 → N2
щелочной среде до манганат-иона - МnО22-:
NO2- + H2O → NO3+
2+
MnO4 + 5e + H → Mn (в кислой среде)
Cr3+- + OH- → CrO42MnO4- + 3e- + H2O→ MnO2 (в нейтральной
Mn2+ + H2O → MnO4среде)
Sn2+ → Sn4+
MnO4- + 3e- +OH- → MnO4 2- (в щелочной среде) Fe2+ → Fe3+
H2O2 → O2
-2
О2 + 4 e →2О
Cl2 + 2 e- → 2ClBr2 + 2 e- →2BrI2 + 2 e- →2IHClO → HCl
KClO3 → KCl
ClO4-→ ClIO3- → I2
H2SO4 → H2S, S, SO2
HNO3 → N2, NO, N2O, NO2, NH3, NH4NO3
NO3- →NO2- + H2O
NO2- + H+→ NO
PbO2 + H+→ Pb2+
MnO2 + H+→ Mn2+
Sn4+ → Sn2+
Fe3+ → Fe2+
Коррозия – это разрушение металлов и сплавов в результате воздействия на них
окружающей среды. По механизму протекания разрушений различают 2 типа коррозии:
химическую и электрохимическую.
Признаки
Химическая коррозия
Электрохимическая коррозия
сравнения
Определение Это
разрушение Это разрушение как одного, так и металлов в
металлов в результате контакте,
которое
сопровождается
взаимодействия их с возникновением электрического тока в воде
сухими
газами
или или среде другого электролита. Наряду с
жидкостями,
не химическими
процессами
(отдачей
проводящими
электронов) протекают и электрические –
Кочулева Людмила Рамановна, учитель химии МОБУ «Средняя общеобразовательная школа №73» г.
Оренбурга. Опорные конспекты и алгоритмы для подготовки к ЕГЭ.
электрический ток.
Агрессивные
реагенты
Примеры
перенос электронов от одного участка к
другому, т. е. от анода к катоду.
При контакте двух металлов появляется
электрический ток тем большей силы, чем
дальше стоят друг от друга металлы в ряду
напряжений. При этом поток электронов идет
от более активного металла к менее
активному; более активный металл в этом
случае разрушается.
Растворы электролитов
O2, пары H2O, CO2, SO2
,Cl2
3Fe + 2O2 = Fe3O4
Железо во влажном воздухе или воде:
4Fe + 3O2 + 6H2O = 4 Fe(OH)3
При контакте железа с цинком коррозии
подвергается цинк:
А (+) на цинке К (-) на железе
Zn0 - 2e- = Zn2+ 2H+ + 2e- = H2
При контакте алюминия с магнием коррозии
подвергается магний:
А(+) на магнии
К (-) на алюминии
0
2+
Mg - 2e = Mg
2H+ + 2e- = H2
Методы защиты от коррозии.
1)
Применение защитных покрытий
- Металлические изделия покрывают другим металлом, т.е. производят
никелирование, хромирование, цинкование, лужение и т.д.
- Металлические изделия покрывают лаками, красками, эмалями
2) Применение легированных сплавов стойких к коррозии.
3) Электрохимическая защита
- Применение заклёпок из активного металла (протектора)
- Прикрепление пластинок из более активного металла
- Нейтрализация тока, возникающего при коррозии постоянным током,
пропускаемом в обратном направлении.
4) Изменение состава среды (добавление ингибиторов).
5) Замена корродирующего металла на другие материалы керамику и пластмассу.
6) Шлифование поверхностей изделия.
А26. Правила работы в лаборатории. Лабораторная посуда и оборудование.
Правила безопасности при работе с едкими, горючими и токсичными веществами,
средствами бытовой химии. Научные методы исследования химических веществ и
превращений. Методы разделения смесей и очистки веществ. Качественные реакции
на неорганические вещества и ионы. Идентификация органических соединений.
Правила работы в лаборатории.
1)
Работать одному в лаборатории категорически запрещается, так как в
ситуации несчастного случая некому будет оказать помощь пострадавшему и
ликвидировать последствия аварии.
I.
Кочулева Людмила Рамановна, учитель химии МОБУ «Средняя общеобразовательная школа №73» г.
Оренбурга. Опорные конспекты и алгоритмы для подготовки к ЕГЭ.
2)
Во время работы в лаборатории необходимо соблюдать чистоту, тишину,
порядок и правила техники безопасности, так как поспешность и небрежность часто
приводят к несчастным случаям с тяжелыми последствиями.
3)
Каждый работающий должен знать, где находятся в лаборатории средства
противопожарной защиты и аптечка, содержащая все необходимое для оказания первой
помощи.
4)
Категорически запрещается в лаборатории курить, принимать пищу, пить
воду.
5)
Нельзя приступать к работе, пока учащиеся не усвоят всей техники ее
выполнения.
6)
Опыты нужно проводить только в чистой химической посуде. После
окончания эксперимента посуду сразу же следует мыть.
7)
В процессе работы необходимо соблюдать чистоту и аккуратность, следить,
чтобы вещества не попадали на кожу лица и рук, так как многие вещества вызывают
раздражение кожи и слизистых оболочек.
8)
Никакие вещества в лаборатории нельзя пробовать на вкус. Нюхать
вещества можно, лишь осторожно направляя на себя пары или газы легким движением
руки, а не наклоняясь к сосуду и не вдыхая полной грудью.
9)
На любой посуде, где хранятся реактивы, должны быть этикетки с
указанием названия веществ.
10)
Сосуды с веществами или растворами необходимо брать одной рукой за
горлышко, а другой снизу поддерживать за дно.
11)
Категорически запрещается затягивать ртом в пипетки органически
вещества и их растворы.
12)
Во время нагревания жидких и твердых веществ в пробирках и колбах
нельзя направлять их отверстия на себя и соседей. Нельзя также заглядывать сверху в
открыто нагреваемые сосуды во избежание возможного поражения при выбросе горячей
массы.
13)
После окончания работы необходимо выключить газ, воду, электроэнергию.
14)
Категорически запрещается выливать в раковины концентрированные
растворы кислот и щелочей, а также различные органические растворители, сильно
пахнущие и огнеопасные вещества. Все эти отходы нужно сливать в специальные бутыли.
15)
В каждой лаборатории обязательно должны быть защитные маски, очки.
16)
В каждом помещении лаборатории необходимо иметь средства
противопожарной защиты: ящик с просеянным песком и совком для него,
противопожарное одеяло (асбестовое или толстое войлочное), заряженные огнетушители.
II.
Правила безопасности при работе с едкими, горючими и токсичными
веществами, средствами бытовой химии.
1)
Для ускорения растворения твёрдых веществ в пробирке нельзя закрывать
её отверстие пальцем при встряхивании.
2)
Растворение щелочи следует производить в фарфоровой посуде путём
прибавления к воде небольших порций вещества, при непрерывном перемешивании.
3)
При определении запаха вещества нельзя наклоняться над ним, вдыхать
пары или выделяющийся газ. Нужно лёгким движением руки над горлом сосуда
направить пар или газ к носу и вдыхать осторожно.
4)
Пролитую кислоту или щёлочь следует засыпать чистым сухим песком и
перемешивать его до полного впитывания всей жидкости. Влажный песок убрать совком в
широкий стеклянный сосуд для последующей промывки и нейтрализации.
Кочулева Людмила Рамановна, учитель химии МОБУ «Средняя общеобразовательная школа №73» г.
Оренбурга. Опорные конспекты и алгоритмы для подготовки к ЕГЭ.
5)
Растворы из реактивных склянок необходимо наливать так, чтобы при
наклоне этикетка оказывалась сверху (этикетка - в ладонь). При попадании на кожу
растворов щелочей или кислот необходимо смыть их после стряхивания видимых капель
сильной струёй холодной воды, а затем обработать нейтрализующим раствором (2%
раствором уксусной кислоты или 2% раствором гидрокарбоната натрия) и ополоснуть
водой.
III.
Методы разделения смесей и очистки веществ. Чистые вещества и смеси
веществ.
Смесь — материал, состоящий из двух или нескольких веществ, хаотично
чередующихся друг с другом в пространстве.
Чистое вещество - физически и химически однородный материал, обладающий
определенным комплексом постоянных свойств. Содержание примесей в препаратах
особой чистоты измеряется миллионными и миллиардными долями процента.
Сравнительная характеристика смеси и чистого вещества
Признаки сравнения
Чистое вещество
Смесь
Состав
Постоянный
Непостоянный
Вещества
Одно и то же
Различные
Физические свойства
Постоянные
Непостоянные
С помощью химических
Разделение
Физическими методами
реакций
Смеси
Однородные (гомогенные)
Неоднородные (гетерогенные)
Однородными называют такие смеси, Неоднородными
называют
смеси,
в
частицы в которых нельзя обнаружить ни которых частицы можно обнаружить либо
визуально, ни с помощью оптических визуально, либо с помощью оптических
приборов, поскольку вещества находятся в приборов. Причём эти вещества находятся в
раздробленном состоянии на микроуровне
разных агрегатных состояниях (фазах)
Примеры смесей
Истинные растворы (поваренная соль + Суспензии (твёрдое + жидкость), например
вода, раствор спирта в воде)
вода + песок
Твёрдые
растворы,
сплавы, Эмульсии (жидкость + жидкость), например
например,латунь, бронза.
вода + жир
Газовые растворы (смеси любых количеств Аэрозоли (газ + жидкость), например туман
и любого числа газов)
Каждый компонент смеси сохраняет набор своих характеристик, поэтому разные вещества
можно выделить из смеси.
Способы разделения смесей
Однородные (гомогенные)
Неоднородные (гетерогенные)
Дистилляция (вода-поваренная соль)
Фильтрование (вода-песок)
Кристаллизация (вода-сахар)
Отстаивание (вода-мел)
Выпаривание (вода-поваренная соль)
Центрифугирование (вода-мел)
Перегонка (нефть)
Магнитная сепарация (железо-медь)
Отстаивание – это способ, основанный на различной плотности веществ.
Например, смесь растительного масла и воды можно разделить на масло и воду,
дав смеси просто отстояться.
Фильтрование – это способ, основанный на различной способности фильтра
пропускать вещества, из которых состоит смесь. Например, с помощь фильтра можно
отделить твердые примеси от жидкости.
Кочулева Людмила Рамановна, учитель химии МОБУ «Средняя общеобразовательная школа №73» г.
Оренбурга. Опорные конспекты и алгоритмы для подготовки к ЕГЭ.
Выпаривание – это выделение нелетучих твердых веществ из раствора в летучем
растворителе – в частности в воде. Например, чтобы выделить соль, растворенную в воде,
надо просто выпарить воду. Вода испарится, а соль останется.
IV.
Качественные реакции на ионы в растворе.
На катионы:
Катион
Реактив
Наблюдаемая реакция
Li+
пламя
Карминово-красное окрашивание
Na+
пламя
Желтое окрашивание
К+
пламя
Фиолетовое окрашивание
2+
Са
пламя
Кирпично-красное окрашивание
2+
Sr
пламя
Карминово-красное окрашивание
пламя
Желто-зеленое окрашивание
22+
SО4
Выпадение белого осадка, не растворимого в
Ва
кислотах: Ва2+ + SО42- → BaSО4
ОНВыпадение осадка синего цвета:
Сu2+
Сu2+ + 2ОН- → Сu(ОН)2
S2РЬ2+
Выпадение черного осадка: Pb2+ + S2- → PbS
Cl
Выпадение белого осадка; не растворимого в HNO3,
но растворимого в конц. NH3 • Н2О:
Аg+
Аg+ +Cl- →AgCl
1)
ОН1)
Выпадение светло-зелёного осадка:
2)
гексациано- Fe2++ 2ОН- → Fe (ОН)2
Fe2+
феррат (III) калия
2)
Выпадение синего осадка:
(красная кровяная
К++ Fe2+ + [Fe(CN)6]3-→ KFe[Fe(CN)6]4
соль), K3[Fe(CN)6]
1) ОН1)
Выпадение бурого осадка:
3+
2)
Fe + 3ОН- → Fe (ОН)3
гексацианоферрат
2)
Выпадение синего осадка:
+
(II) калия (желтая
К + Fe3+ + [Fe(CN)6]4- → KFe[Fe(CN)6]
Fe3+
кровяная соль)
3) Появление ярко-красного окрашивания за счет
K4[Fe(CN)6]
образования комплексных ионов Fe(SCN)2+,
3) роданид-ион
Fe(SCN)+2
SCN
щелочь
Выпадение осадка гидроксида алюминия при
(амфотерные
приливании первых порций щелочи и его
Al3+
свойства
растворение при дальнейшем приливании
гидроксида)
щелочь, нагрев
NH4+
Запах аммиака: NH4+ + ОН-→ NH3 + Н2О
Н+
индикаторы:
(кислая
лакмус, метиловый красное окрашивание
среда)
оранжевый
красное окрашивание
На анионы:
Анион
Реактив
Наблюдаемая реакция
SО42Ва2+
Выпадение белого осадка, нерастворимого в
кислотах: Ва2+ + SО42BaSО4
NО3
Добавить конц.
Образование голубого раствора, содержащего
H2SO4 и Си,
ионы Сu2+, выделение газа бурого цвета (NO2)
нагреть
Кочулева Людмила Рамановна, учитель химии МОБУ «Средняя общеобразовательная школа №73» г.
Оренбурга. Опорные конспекты и алгоритмы для подготовки к ЕГЭ.
РО43-
ионы Ag+
S2СО32-
ионы РЬ2+
ионы Са2+
CO2
известковая вода
Са(ОН)2
SO32-
ионы Н+
FCl-
ионы Са2+
ионы Аg+
Br-
ионы Аg+
I-
ионы Аg+
ОН(щелочная
среда)
индикаторы:
лакмус
фенолфталеин
Выпадение светло-желтого осадка в нейтральной
среде: ЗАg+ + РО43Аg3РО4
Выпадение черного осадка: Pb2+ + S2PbS
Выпадение белого осадка, растворимого в
кислотах: Са2+ + СО32- = СаСОз
Са(ОН)2 + СО2
СаСО3 + Н2О,
СаСО3 + СО2 + Н2О
Са(НСО3)2
Выпадение белого осадка и его растворение при
пропускании СО2
Появление характерного запаха SО2:
2Н+ + SO32Н2О + SО2
Выпадение белого осадка: Са2+ + 2FCaF2
Выпадение белого осадка, не растворимого в
HNО3, но растворимого в конц. NH3 • Н2О:
Аg+ +CIAgCl
AgCI + 2(NH3• Н2О)
[Ag(NH3)2]+ + CI- +2Н2О
Выпадение светло-желтого осадка, не
растворимого в HNО3:
Ag+ + Br- = AgBr осадок темнеет на свету
Выпадение желтого осадка, не растворимого в
HNO3 и NH3 конц.: Аg+ + IАgI осадок
темнеет на свету
синее окрашивание
малиновое окрашивание
Идентификация органических соединений
Соединение
Алканы
Реактив
Пламя
Наблюдаемая реакция
Обычно определяют путем исключения.
Низшие алканы горят голубоватым пламенем
Алкены С=С
1) Бромная
Обесцвечивание раствора.
вода
Обесцвечивание раствора, выпадение бурого осадка
2) р-р КМпО4
МnО2
3) Горение
Горят - слегка желтоватым пламенем (частицы
углерода).
Бензол
Горение
Обычно определяют путем исключения.
Горит коптящим пламенем.
Фенол
1) Бромная
Обесцвечивание, выпадение белого осадка
вода
трибромфенола
2) р-р Na2СО3
Выделение углекислого газа.
3) FeCI3
Фиолетовое окрашивание.
Спирты
1) Na
Выделение водорода.
2) Горение
Горят светлым голубоватым пламенем.
3) Прокаленная Восстановление красной окраски у прокаленной
Сu-проволока
горячей медной проволоки.
Многоатомные Сu(ОН)2 + NaOH Синее окрашивание — образование глицератов и др.
спирты
Кочулева Людмила Рамановна, учитель химии МОБУ «Средняя общеобразовательная школа №73» г.
Оренбурга. Опорные конспекты и алгоритмы для подготовки к ЕГЭ.
Амины
1) Лакмус
2) HHal
Анилин
1) Бромная
вода
Альдегиды
Карбоновые
кислоты
Крахмал
Белки
2) HHal
1) Ag2О
2) Сu(ОН)2
Лакмус
Раствор l2 в KI
или спиртовой
раствор иода
соломенножёлтого цвета
1) конц. HNO3
2) Сu(ОН)2
В водном растворе — синее окрашивание.
Образуют соли с галогеноводородами — после
выпаривания твердый осадок.
Обесцвечивание бромной воды, выпадение осадка
триброманилина.
После упаривания твердый осадок — соль
гидрогалогенида анилина.
Реакция серебряного зеркала
Выпадение красного осадка Cu2О
Красное окрашивание
Муравьиная — реакция серебряного зеркала
Олеиновая — обесцвечивание бромной воды
Синее окрашивание
Желтое окрашивание, при добавлении щелочного
раствора – оранжевое
Красно-фиолетовое окрашивание
А27. Понятие о металлургии: общие способы получения металлов. Общие
научные
принципы химического производства (на примере промышленного
получения аммиака, серной кислоты, метанола). Химическое загрязнение
окружающей среды и его последствия. Природные источники углеводородов, их
переработка. ВМС. Реакции полимеризации и поликонденсации. Полимеры.
Пластмассы, волокна, каучуки.
Металлургия – наука о промышленных способах получения металлов.
Способы получения металлов:
1. Пирометаллургия — это восстановление металлов из их руд при высоких
температурах с помощью восстановителей (неметаллических - кокс, оксид углерода (II),
водород; металлических - алюминий, магний, кальций и другие металлы).
Пирометаллургически получают: чугун, сталь, медь, свинец, никель, хром и другие
металлы.
TiO2 + 2Mg = Ti + 2MgO + Q
МоО3 + 3Н2 = Мо + 3Н2О
2. Гидрометаллургия — восстановление металлов из их солей в растворе.
Например, руда содержит оксид меди и ее растворяют в серной кислоте: CuO + H 2SO4 =
CuSO4 + H2O (медная руда не извлекается на поверхность), затем проводят реакцию
замещения CuSO4 + Fe = FeSO4 + Cu или электролиз.
3. Электрометаллургия — восстановление металлов в процессе электролиза
растворов или расплавов их соединений. Этим методом получают алюминий, щелочные
металлы, щелочноземельные металлы. При этом подвергают электролизу расплавы
оксидов, гидроксидов или хлоридов.
I.
Кочулева Людмила Рамановна, учитель химии МОБУ «Средняя общеобразовательная школа №73» г.
Оренбурга. Опорные конспекты и алгоритмы для подготовки к ЕГЭ.
II.
Общие научные
принципы химического производства (на примере
промышленного получения аммиака, серной кислоты, метанола).
Научные принципы организации химических производств
Общие принципы
Частные принципы
Создание оптимальных условий Противоток веществ, прямоток веществ, увеличесе
проведения химических реакций поверхности
соприкосновения
реагирующих
веществ, использование катализатора, повышение
давления, повышение концентрации реагирующих
веществ
Полное
и
комплексное Циркуляция, создание смежных производств (по
использование сырья
переработке отходов)
Использование
теплоты Теплообмен, утилизация теплоты реакции
химических реакций
Принцип непрерывности
Механизация и автоматизация производства
Защита окружающей среды и Автоматизация вредных производств, герметизация
человека
аппаратов, утилизация отходов, нейтрализация
выбросов в атмосферу
1)
2)
Контактный способ производства серной кислоты
Сырье: пирит FeS2, самородная сера, серосодержащие газы — отходы цветной
металлургии, воздух.
Вспомогательные материалы: серная кислота (98%), катализатор — оксид ванадия
(V).
Аппарат
Назначение
Реация
Печь для обжига Пирит
подвергают
обжигу 4FeS2 + 11 O2 = 2Fe2O3 + 8SO2 +
«в
кипящем кислородом воздуха
3310 кДж
слое»
Циклон
Тщательная очистка от крупных частиц пыли
Электрофильтр
Тщательная очистка от мелких частиц пыли
Сушильная
Осушение от водяных паров
башня
Теплообменник Очищенный обжиговый газ перед
поступлением
в
контактный аппарат нагревают
за
счет
теплоты
газов,
выходящих из контактного
аппарата.
Контактный
Обжиговый газ в присутствии 2SО2 + О2 = 2SО3 +197,8 кДж
аппарат
катализатора при 450 °С
окисляется до оксида серы (VI)
Поглотительная Оксид серы (VI) поглощают SО3 + Н2О = Н2SО4
башня
концентрированной
серной
кислотой. Образуется олеум, из
которого можно приготовить
серную
кислоту
любой
концентрации.
Кочулева Людмила Рамановна, учитель химии МОБУ «Средняя общеобразовательная школа №73» г.
Оренбурга. Опорные конспекты и алгоритмы для подготовки к ЕГЭ.
Особенности технологического процесса:
производство непрерывное, обжиг колчедана ведут в кипящем слое, продувая в
печь воздух, нагретый отходящим обжиговым газом. Тщательно очищенный обжиговый
газ перед поступлением в контактный аппарат нагревают за счет теплоты газов,
выходящих из контактного аппарата. В поглотительных башнях оксид серы (VI)
поглощают серной кислотой противотоком.
Основной продукт: олеум.
Синтез аммиака
Сырье: азотоводородная смесь.
Вспомогательный материал: катализатор (пористое железо).
Основной химический процесс:
Предварительно получают азотоводородную смесь. Водород
парокислородной конверсией метана (из природного газа):
СН4 + Н2О(г) → СО + ЗН2 - Q
2СН4 + О2 = 2СО + 4Н2 + Q
СО + Н2О(г) = СО2 + Н2 + Q
Азот получают ректификацией жидкого воздуха.
Аппарат
Турбоком
прессор
Колонна
синтеза
Холодиль
получают
Назначение
Реация
Сжатие смеси до необходимого
6
давления 25·10 Па
Газы реагируют при 450—500 °С в
N2 + 3H2
присутствии катализатора под давлением с 2NH3 + 92 кДж
образованием 10—20% аммиака
Сжижение аммиака
-
ник
Сепаратор
Отделение
аммиака
непрореагировавших азота и водорода
от
-
Кочулева Людмила Рамановна, учитель химии МОБУ «Средняя общеобразовательная школа №73» г.
Оренбурга. Опорные конспекты и алгоритмы для подготовки к ЕГЭ.
↔
Особенности технологического процесса: направление движения азотоводородной
смеси в колонне синтеза выбирают таким образом, чтобы максимально использовать
теплоту реакции и предохранить наружные стенки аппарата от чрезмерного нагревания.
Образующийся аммиак (10—20%) отделяют сжижением, возвращая непрореагировавшую
азотоводородную смесь в колонну синтеза. Процесс непрерывный, циркуляционный.
Основной продукт: аммиак.
Применение: производство азотных удобрений, взрывчатых
веществ,
пластических масс и др.
Производство метанола (метилового спирта)
До промышленного освоения каталитического способа получения метанол
получали при сухой перегонке дерева (отсюда его название «древесный спирт»). В данное
время этот способ имеет второстепенное значение.
Современный способ:
Сырье: синтез-газ — смесь оксида углерода (II) с водородом (1:2).
Вспомогательные материалы: катализаторы (ZnO и CuO).
Основной химический процесс: синтез-газ при температуре 250 °С и давлении 7
МПа превращается каталитически в метанол: СО + 2Н2 → СНзОН + Q
Особенности технологического процесса: при прохождении газовой смеси через
слой катализатора образуется 10—15% метанола,
который конденсируют, а
непрореагировавшую смесь смешивают со свежей порцией синтез - газа и после
нагревания снова направляют в слой катализатора (циркуляция). Общий выход - 85%.
Основной продукт: метанол.
Условия проведения синтеза метанола и аммиака при среднем давлении сходны, а
сырье (природный газ) общее для обоих процессов. Поэтому чаще всего производства
метанола и аммиака объединяют (азотно-туковые заводы).
Кочулева Людмила Рамановна, учитель химии МОБУ «Средняя общеобразовательная школа №73» г.
Оренбурга. Опорные конспекты и алгоритмы для подготовки к ЕГЭ.
Производство азотной кислоты
Сырье: аммиак, воздух.
Вспомогательные материалы: катализаторы (платинородиевые
сетки), вода,
концентрированная серная кислота.
Аппарат
Назначение
Реация
Смеситель
Смешение воздуха и аммиака
Контактный
Аммиак в смеси с воздухом 4NH3 + 5O2 = 4NO + 6H2O(r) +
аппарат
окисляется при температуре 800 °С в Q
присутствии катализатора до оксида
азота (II)
Окислительная Окисление оксида азота (II) в оксид 2NO + О2 = 2NО2 + Q
башня
азота (IV) происходит при обычной
температуре
Поглотительная Оксид азота (IV) при взаимодействии 4NО2 + 2H2О + О2 ↔ 4HNO3 +
башня
с кислородом и водой превращается в Q
азотную кислоту
или
3NО2 + H2О ↔ 2HNO3 +NO +
Q
Особенности технологического процесса:
производство непрерывное,
воздушноаммиачная смесь поступает в контактный аппарат, где происходит окисление
аммиака. Необходимая температура поддерживается за счет
выделяемой теплоты.
Газовую смесь, содержащую оксид азота (II), охлаждают в топке котла-утилизатора.
Полученную смесь, содержащую оксид азота (IV), направляют в поглотительную башню,
где по принципу
противотока происходит смешивание воды и газовой смеси с
образованием азотной кислоты (концентрация не менее 60%). Более концентрированную
азотную кислоту получают, добавляя концентрированную серную кислоту в качестве
водоотнимающего средства.
Основной продукт: азотная кислота.
III.
Природные источники углеводородов, их переработка.
Коксование угля
Сырье: коксующиеся угли.
Вспомогательные материалы: воздух, горючий газ.
Основной химический процесс: нагревание угля без доступа воздуха до 900—1050
°С приводит к его термическому разложению с образованием летучих продуктов
(каменноугольная смола, аммиачная
вода и коксовый газ) и твердого остатка — кокса.
Кочулева Людмила Рамановна, учитель химии МОБУ «Средняя общеобразовательная школа №73» г.
Оренбурга. Опорные конспекты и алгоритмы для подготовки к ЕГЭ.
Особенности технологического процесса: в коксовую печь, состоящую из камер,
загружают уголь и в каналах отопительных простенков зажигают газ. Коксование угля —
периодический процесс.
Основные продукты: кокс — 96—98% углерода; коксовый газ —60% водорода,
25% метана, 7% оксида углерода (II) и др.
Побочные продукты: каменноугольная смола (бензол, толуол), аммиак (из
коксового газа) и др.
Переработка нефти методом ректификации
Сырье: нефть.
Особенности технологического процесса: предварительно
очищенную нефть
подвергают атмосферной (или вакуумной) перегонке на фракции с определенными
интервалами температур кипения в ректификационных колоннах непрерывного действия.
Основные продукты: легкий и тяжелый бензин, керосин, газойль, смазочные масла,
мазут, гудрон.
Переработка нефти каталитическим крекингом
Сырье: высококипящие нефтяные фракции (керосин, газойль и др.)
Вспомогательные материалы: катализаторы (модифицированные алюмосиликаты).
Основной химический процесс: при температуре 500—600 °С и давлении 5·105 Па
молекулы углеводородов расщепляются на более мелкие молекулы, каталитический
крекинг сопровождается реакциями ароматизации, изомеризации, алкилирования.
Особенности технологического процесса: нагретое в
теплообменнике сырье
поступает в реактор, смешиваясь с катализатором. Пары продуктов крекинга отделяются
от катализатора и поступают в
ректификационную колонну непрерывного действия.
Продукты: смесь низко кипящих углеводородов (топливо, сырье для нефтехимии).
Производство ацетилена
Сырье: природный газ.
Основной химический процесс: метан пропускают через вольтову дугу между
металлическими электродами (электрокрекинг) при 1500 °С:
2СН4 → С2Н2 + ЗН2
Особенности технологического процесса: природный газ нагревают пламенем
электрической дуги в специальных печах, пребывание газа в зоне высоких температур
должно быть кратковременным, после чего газ быстро охлаждают (закалка).
Основной продукт: ацетилен.
Побочные продукты: водород и метан, которые применяют для получения
азотоводородной смеси в синтезе аммиака.
IV.
ВМС. Полимеры.
Полимеры – это высокомолекулярные соединения.
Кочулева Людмила Рамановна, учитель химии МОБУ «Средняя общеобразовательная школа №73» г.
Оренбурга. Опорные конспекты и алгоритмы для подготовки к ЕГЭ.
Мономеры – это низкомолекулярные вещества, из которых получают полимеры.
Степенью полимеризации (поликонденсации) называют среднее число
структурных звеньев в молекуле полимера.
Повторяющийся участок структуры молекулы полимера называют структурным
звеном.
Природные органические ВМС – целлюлоза, белки, крахмал, натуральный каучук;
неорганические – графит, силикаты.
Искусственные ВМС получают из природных ВМС, используя химические методы,
которые
не изменяют главную цепь.
Синтетические ВМС получают при помощи реакций полимеризации и
поликонденсации низкомолекулярных веществ.
Искусственные органические полимеры – ацетил-целлюлоза, нитроцеллюлоза,
резина;
синтетические
органические
полимеры
–
полиэтилен,
полистирол,
поливинилхлорид, капрон, лавсан, каучуки.
V.
Реакции полимеризации и поликонденсации.
план
опре
деление
ый
нак
нтов
еры
VI.
полимеризация
Процесс соединения
молекул
в более крупные
поликонденсация
Процесс образования ВМВ из
низкомолекулярных с
отщеплением
побочного продукта (чаще всего
воды)
главн
Двойная связь, каждая
Наличие в молекулах не менее 2молекула
х ФГ, каждая
приз
мономера
молекула мономера соединяется
соединяется
не менее чем
реаге
не менее чем с двумя
с двумя молекулами - слева и
молекулами - слева и справа
справа
прим
Получение ПП, ПЭ,
Получение ФФС, образование
тефлона
полисахаридов, РНК и ДНК,
белков в природе
Пластмассы, волокна, каучуки.
Пластмассы - это конструкционные материалы, содержащие полимер и способные
при нагревании приобретать заданную форму и сохранять её после охлаждения.
Термоплатичные пластмассы – полиэтилен, полипропилен, полистирол, ПВХ,
полиметилметакрилат, полиамид. Термореактивные пластмассы – полиуретан, силиконы,
ФФС.
Волокна – это полимеры линейного строения, которые пригодны для изготовления
текстильных материалов.
Природные волокна – шерсть, шёлк, хлопок, лён.
Искусственные волокна – вискозное, ацетатное – получают из природных
полимеров или продуктьов их переработки, главным образом из целлюлозы и её эфиров.
Кочулева Людмила Рамановна, учитель химии МОБУ «Средняя общеобразовательная школа №73» г.
Оренбурга. Опорные конспекты и алгоритмы для подготовки к ЕГЭ.
Синтетические волокна – капрон, лавсан, найлон, энант – получают из
синтетических полимеров.
В1. Классификация неорганических веществ. Номенклатура неорганических
веществ (см. А7)
В3. Электролиз расплавов и растворов (солей, щелочей, кислот)
1)
Электролиз - это окислительно-восстановительный процесс, который
протекает на электродах при
пропускании постоянного электрического тока через
раствор или расплав электролита. Прибор, в котором проводят электролиз, называют
электролизёром.
2)
Сущность электролиза состоит в осуществлении за счёт электрической
энергии химических реакций.
3)
Катод К(-), к нему перемещаются катионы, катод отдаёт электроны
катионам, катионы разряжаются, идёт процесс восстановления.
4)
Анод А(+), к нему перемещаются анионы, анионы отдают электроны
аноду и разряжаются, идёт процесс окисления.
5)
Электролиз растворов отличается от электролиза расплавов тем, что
процессы протекают сложнее из-за непосредственного участия воды.
6)
Катодные процессы:
Катионы
катионы
неактивных
Ме (Cu, Hg,
Ag, Pt, Au)
катионы активных Ме
от Li до Al (вкл.)
(Li, Rb, K, Ba, Sr, Ca, Na, Mg,
Al)
катионы
малоактивных Ме
(между Мn и Н)
Н+
(раствор
кислоты)
процесс
Полностью
восстанавли
в.
до Ме
Катионы Ме не
восстанавливаются, остаются
в растворе.
Восстанавливаются ионы Н+
из молекул Н2О:
2Н2О +2ė → Н2 +2ОН-
Восстанавливаются
ионы этих Ме и
частично Н+ из
молекул Н2О:
Меn+ + nė → Me 0
2Н2О +2ė → Н2 +2ОН-
Восстанавли
ваются
катионы Н+
до Н2.
среда
продукт
металл
среда щелочная
щелочь и водород
металл и водород
водород
7) Анодные процессы (для нерастворимого А – уголь, платина, графит, иридий):
Анионы
процесс
анионы
бескислородных
кислот
(Cl-, Br -, I-, S2-,
CN-) (кроме F- )
Анионы
окисляются до
простых
веществ.
Например:
анионы
кислородсодержащих
кислот и фторидов
(SO42-, NO3- , SO32- ,
CO32-, PO43-, F-)
Анионы не окисляются,
идёт процесс окисления
OHиз молекул Н2О:
2 Н2О - 4 ė → О2 + 4Н+
OH(растворы
щелочей)
4ОН - - 4 ė
→ О2 +
2Н2О
анионы в растворах
солей карбоновых
кислот
2 R - СОО- - 2 ė →
R - R + 2СО2
Кочулева Людмила Рамановна, учитель химии МОБУ «Средняя общеобразовательная школа №73» г.
Оренбурга. Опорные конспекты и алгоритмы для подготовки к ЕГЭ.
среда
продукт
2Cl- -2ė → Cl2
простые
вещества
среда кислая
кислород и кислота
Примеры:
1) LiCl(р-в) → Li+ + СlK(-)
Li+
Li+ +1ė → Li
A(+)
Cl2Cl- -2ė → Cl2
эл-з
2LiCl → 2Li + Cl2
2)NaCl (р-р) → Na+ + ClK(-)
H2O
2Н2О +2ė →Н2 +
Na+
2ОНA(+)
ClH2O
2Сl--2ė → Сl2
эл-з
2NaCl + 2 H2O → H2 + Cl2 +2NaOH .
3)CuCl2 (р-р) → Cu2+ + 2ClK(-)
Cu2+
Cu2+ +2ė → Cu
H2O
A(+)
Cl2Cl- - 2ė → Cl2
H2O
эл-з
CuCl2→ Cu + Cl2
4)FeSO4 (р-р) → Fe2++SO42K(-)
Fe2+
Fe 2+ +2ė → Fe
H2O
2Н2О +2ė → Н2 +
A(+)
SO42- 2ОНH2O
2 Н2О- 4 ė → О2 +
4Н+
эл-з
FeSO4 +2Н2О → Fe+ Н2 + О2 +Н2SO4.
кислород и
вода
Алкан и СО2
2
1
1
1
1
1
1
1
5)Hg (NO3)2 (р-р) → Hg 2+ + 2NO3K(-)
Hg 2+ Hg 2+ + 2ė → Hg
2
Н2О
A(+)
NO3Н2О
2 Н2О- 4 ė → О2 +
1
4Н+
эл-з
2Hg (NO3)2 +2Н2О → 2Hg + О2 + 4 HNO3
эл-з
Кочулева Людмила Рамановна, учитель химии МОБУ «Средняя общеобразовательная школа №73» г.
Оренбурга. Опорные конспекты и алгоритмы для подготовки к ЕГЭ.
6) 4NaOH → 4Na + O2 + 2H2O
эл-з
7) Co (NO3)2 +2 H2O → Co + H2 +O2 + 2HNO3
эл-з
8) Na2SO4 + 2H2O → 2H2 + O2 + Na2SO4
эл-з
2H2O → 2H2 + O2
А24. В4. Гидролиз солей. Среда водного раствора солей: кислая, нейтральная,
щелочная (см. А24)
В6. Ионный (правило В.В. Марковникова) и радикальный механизмы
реакций в органической химии.
IX.
По механизму протекания
3)
Радикальный механизм.
А : В → А· + ·В
Происходит гомолитический (равноценный) разрыв связи. При гемолитическом
разрыве пара электронов, образующая связь, делится таким образом, что каждая из
образующихся частиц получает по одному электрону. При этом образуются радикалы –
незаряженные частицы с неспаренными электрономи. Радикалы – очень
реакционноспособные частицы, реакции с их участием происходят в газовой фазе с
большой скоростью и часто со взрывом.
Радикальные реакции идут между образующимися в ходе реакции радикалами и
молекулами:
2H2O2 → 2H2O + O2
hυ
CH4 + Cl2 → CH3Cl +HCl
По такому механизму протекает большинство реакций горения органических и
неорганических веществ: синтез воды, аммиака, полимеризация этилена, винилхлорида и
др.
4)
Ионный механизм.
А : В → :А- + В+
Происходит гетеролитический (неравноценный) разрыв связи, при этом оба
электрона связи остаются с одной из ранее связанных частиц. Образуются заряженные
частиц (катионы и анионы).
Ионные реакции идут в растворах между уже имеющимися или образующимися в
ходе реакции ионами.
Например, в неорганической химии – это взаимодействие электролитов в растворе,
в органической химии – это реакции присоединения к алкенам, окисление и
дегидрирование спиртов, замещение спиртовой группы и другие реакции,
характеризующие свойства альдегидов и карбоновых кислот.
Взаимное влияние химических групп на свойства молекул
Эффекты заместителей
Особо большое влияние на химические свойства веществ оказывают заместители в
органических соединениях.
Кочулева Людмила Рамановна, учитель химии МОБУ «Средняя общеобразовательная школа №73» г.
Оренбурга. Опорные конспекты и алгоритмы для подготовки к ЕГЭ.
1.Заместители – атомы или группы атомов, которые замещают в молекуле
углеводорода один или несколько атомов водорода.
2. Заместитель всегда влияет на связанный с ним атом углерода, иногда это
влияние распространяется и на другие атомы углеродного скелета.
3. Заместители действуют, притягивая или отталкивая электроны, образующие
химическую связь.
4. Эффект заместителя может быть положительным, если заместитель отдал свои
электроны (заместитель приобрел заряд +), или отрицательным, если он оттянул на себя
электроны (заместитель приобрел заряд -) .
5. Различают два типа эффектов заместителей.
Индуктивный эффект
1.Индуктивный эффект – это передача электронного влияния заместителя по
цепи σ - связей.
2. Индуктивный эффект может передаваться на небольшие расстояния.
3. Индуктивный эффект затухает через 3 – 4 связи.
4. Индуктивный эффект атома водорода считается равным нулю.
5. Заместитель, притягивающий электроны (заряжающийся отрицательно), имеет
отрицательный индуктивный эффект, а отталкивающий электроны (заряжающийся
положительно) – положительный индуктивный эффект.
6.Большинство заместителей обладают отрицательным индуктивным эффектом.
а) Это группы, электроотрицательный атом, например, -F, -CI, -Br, = O, -CF3, CHCI2, -COOH и т.д.
б) Радикалы с кратными связями: -СН = СН2, -C≡CH.
7. К заместителям с положительным индуктивным эффектом относятся все
металлы, атомы O-, S- в солеобразных веществах, алкильные радикалы: -CH3,-C2H5, и др.,
особенно если они разветвлены, например, -C(CH3)3.
8. Индуктивный эффект изображается стрелкой, совпадающей по написанию с
положением черточки, изображающей химическую связь, и показывающей на
направление смещения электронной плотности.
Мезомерный эффект
1.Мезомерный эффект проявляется только в так называемых сопряженных
системах связей.
2. Сопряженными связями называют две двойные связи, разделенные одной
одинарной связью: С = С - С = С.
В этом случае наступает сопряжение этих связей, когда π-электроны сопряженных
связей образуют общее электронное облако, и электроны делокализованы, т.е. равномерно
распределены по всей сопряженной системе связей.
3. При мезомерном эффекте передача влияния заместителя происходит по
сопряженной системе π-связей.
4. Все заместители, имеющие кратные связи, обладают отрицательным
мезомерным эффектом.
а) Например, - СНО, - СООН, - СN, -SO2OH, -NO2.
б) Эти заместители заряжаются отрицательно, оттягивая на себя электроны.
5. Все заместители с неподеленными парами электронов на внешней орбитали
имеют положительный мезомерный эффект.
а) Например, -I, -Br, -Cl, -F, -OH, -NH2, -O-.
б) Неподеленная пара электронов этих заместителей делокализуется с электронами
сопряженных связей, в результате заместитель приобретает положительный заряд.
Кочулева Людмила Рамановна, учитель химии МОБУ «Средняя общеобразовательная школа №73» г.
Оренбурга. Опорные конспекты и алгоритмы для подготовки к ЕГЭ.
6. В отличие от индуктивного эффекта мезомерный эффект передается в молекуле
на гораздо большее расстояние.
7. Мезомерный и индуктивный эффекты поляризуют молекулу совместно, при этом
необходимо учитывать, что
а) Положительный мезомерный эффект больше, чем отрицательный индуктивный
эффект у следующих заместителей: аминогруппы, спиртовой группы, простого эфира.
б) У галогенов отрицательный индуктивный эффект превышает положительный
мезомерный.
Примеры заданий
Пример: как изменяется сила галогенуксусных кислот в зависимости от галогена?
Решение:
1)атом галогена может находиться на втором положении:
СН2Нal-COOH, где Hal – атом галогена.
2) Наиболее электроотрицательный среди галогенов атом фтора, менее
электроотрицательный – атом иода.
3) Галоген будет притягивать электроны σ-связи, поляризуя ее, уменьшая тем
самым заряд на ОН группе:
Hal←CH2←C = O
↑
O←H
4)Чем меньше заряд кислорода на ОН группе, тем легче оторвать от него протон,
т.е. тем лучше будет идти диссоциация, следовательно, сила кислоты будет больше. Таким
образом, фторуксусная кислота – самая сильная, а иодуксусная – самая слабая среди
галогенуксусных кислот.
Пример: как изменяется сила хлормасляной кислоты в зависимости от положения
атома хлора в ее молекуле?
Решение:
так как при увеличении пространственного удаления заместителя от
функциональной группы, влияние заместителя ослабевает, 4-хлормасляная кислота будет
менее сильной, чем 2-хлормасляная кислота.
Cl←CH2←CH2←CH2←C = O
↑
O←H
CH3→CH2→CH←C = O
↓
↑
Cl
O←H
Пример: как изменяется сила хлоруксусной кислоты в зависимости от количества
атомов хлора в ней?
Решение: так как индуктивный эффект заместителей может складываться,
суммарный индуктивный эффект у трихлоруксусной кислоты будет больше, чем в
монохлоруксусной кислоте. Следовательно, трихлоруксусная кислота более сильная, чем
хлоруксусная кислота.
Пример: как влияет длина и разветвленность углеродной цепи на силу карбоновых
кислот?
Решение: алкильные радикалы обладают положительным индуктивным эффектом,
следовательно, они будут отдавать электроны на атом кислорода группы ОН и повышать
Кочулева Людмила Рамановна, учитель химии МОБУ «Средняя общеобразовательная школа №73» г.
Оренбурга. Опорные конспекты и алгоритмы для подготовки к ЕГЭ.
отрицательный заряд этого атома, вследствие чего связь между атомом водорода и
кислорода в группе ОН станет прочнее, сила кислоты уменьшится.
Пример: как изменяется сила кислоты спиртов в зависимости от разветвления
углеродной цепи?
Решение:
1.спирты являются слабыми кислотами, например, могут реагировать с щелочными
металлами, при этом выделяется водород.
2.Т.к. алкильные радикалы обладают положительным индуктивным эффектом, они
повышают отрицательный заряд на атоме кислорода, в результате связь кислорода с
водородом усиливается, и сила кислоты падает.
3. Следовательно, спирты с разветвленными заместителями являются более
слабыми кислотами.
Пример: расположите в ряд галогениды этана по легкости образования спиртов в
реакции: CH3-CH2-Hal + NaOH → CH3-CH2-OH + NaHal.
Решение:
1.
с увеличением порядкового номера галогена его атомный радиус возрастает,
следовательно, увеличивается длина связи С-Наl.
2.
Это ведет к уменьшению энергии связи.
3.
Поэтому при переходе от фтора к иоду будет увеличиваться легкость
образования спиртов из галогенида этана.
Пример: какой атом углерода в алканах легче окислить: первичный, третичный
или вторичный?
Решение:
1.окисление алкана идет тем легче, чем легче он способен отдать электрон. Обычно
алканы окисляются в очень жестких условиях.
2.Индуктивный эффект приводит к увеличению электронной плотности у
третичного атома углерода в большей степени:
CH3
CH3
↓
↓
CH3→CH2→CH3
< CH3→CH2
< CH3→CH-σ
↑
CH3
3. Поэтому третичные атомы углерода будут легче отдавать электроны и
следовательно легче окисляться.
4. Первичный атом окисляется обычно сначала до спирта и затем быстро до
соответствующей кислоты, вторичный - до спирта и быстро до кетона, в то время как
третичный атом окисляется только до спирта.
В8. Характерные химические
соединений: аминов и аминокислот.
свойства
азотсодержащих
органических
Нитросоединения
Нитросоединениями называются органические вещества, в молекулах которых
содержится нитрогруппа — NO2, связанная с углеводородным радикалом.
Их можно рассматривать как производные углеводородов, получающиеся путем
замещения атома водорода на нитрогруппу. По числу нитрогрупп различают моно-, ди- и
полинитросоединения.
Кочулева Людмила Рамановна, учитель химии МОБУ «Средняя общеобразовательная школа №73» г.
Оренбурга. Опорные конспекты и алгоритмы для подготовки к ЕГЭ.
Названия нитросоединемий производят от названий исходных углеводородов с
добавлением приставки нитро-:
Общая формула этих соединений R—NO2.
При восстановлении нитросоединений образуются амины.
Химические свойства аминов
По химическим свойствам амины сходны с аммиаком благодаря наличию
неподелённой электронной пары у атома азота, которая способна взаимодействовать с
вакантной орбиталью Н+ (по донорно-акцепторному механизму), потому амины являются
основаниями.
Амины-органические основания (более сильные по сравнению с аммиаком).
Реакции с участием аминов протекают по ионному механизму.
Химические свойства обусловлены наличием аминогруппы, и поэтому сходны с
химическими свойствами аммиака:
1)
Взаимодействие с кислотами (реакция нейтрализации) –сходство с
аммиаком:
CH3-NH2 + HCl → (CH3-NH3)+Clметиламин
хлорид метиламмония
R-NH2 + H2SO4 → [R-NH3]HSO4
2)
Взаимодействие с водой (сходство с аммиаком):
CH3-NH2 + H2O → (CH3-NH3) + OH метиламин
гидроксид метиламмония
3)
Горение на воздухе (отличие от аммиака):
4CH3-NH2 + 9O2 → 2N2 + 4CO2 + 10H2O
4)
Алкилирование аминов -реакции аминов с галогенуглеводородами.
Этим способом получают из первичных аминов вторичные и третичные, а из
вторичных – третичные. Таким образом в молекулу амина вводится новая алкильная
группа и поэтому данные реакции называются реакциями алкилирования аминов. Процесс
идет через образование солей алкиламмония, которые при нагревании превращаются
амины.
CH NH + CH Cl  [(CH ) NH ]+Cl–  (CH ) NH + HCl
3
2
метиламин
3
3 2
2
хлорид диметиламмония
3 2
диметиламин
Анилин
Анилин – первичный ароматический амин:
С6H5NH2 или
Анилин C6H5NH2 — важнейший из ароматических аминов. Он представляет собой
бесцветную маслянистую жидкость, мало растворимую в воде.
Анилин – очень слабое основание, что объясняется влиянием бензольного ядра на
аминогруппу.
Алифатические амины – более сильные основания, чем аммиак. Ароматические
Кочулева Людмила Рамановна, учитель химии МОБУ «Средняя общеобразовательная школа №73» г.
Оренбурга. Опорные конспекты и алгоритмы для подготовки к ЕГЭ.
амины – более слабые основания, чем аммиак.
Таким образом, основные свойства увеличиваются в ряду:
С6H5NH2 < NH3 < СН3NH2 < (СН3)2 NH.
Химические свойства анилина
I.
Реакции с участием аминогруппы
1)
Его водный раствор не изменяет окраску индикаторов. Со слабыми
кислотами и водой анилин почти не взаимодействует.
2)
Взаимодействие с кислотами (сильными – серной, соляной):
C6H5-NH2 + HCl → (C6H5-NH3)+Clанилин
хлорид фениламмония
II.
Реакции с участием бензольного кольца
В свою очередь аминогруппа оказывает влияние на бензольное кольцо, поэтому
анилин легко (при о.у.) взаимодействует с бромной водой с образованием белого осадка
2,4,6-триброманилина. Это качественная ракция на анилин:
III.
Окисление на воздухе.
На воздухе анилин буреет вследствие окисления.
IV.
Качественная реакция.
Фиолетовое окрашивание с хлорной известью.
KMnO4
С6H5NH2 + 3[O]  С6H5NO2 + H2O
анилин
нитробензол
Аминокислоты
Аминокислоты — это органические бифункциональные соединения, в состав
которых входят карбоксильная группа — СООН и аминогруппа —NH2. В зависимости от
взаимного расположения обеих функциональных групп различают ά-, β
-, γаминокислоты и т. д.:
Греческая буква при атоме углерода обозначает его удаленность от карбоксильной
группы.
В природе встречаются только ά – аминокислоты. В состав белков входят 20
основных аминокислот: глицин, аланин, цистеин, серин, фенилаланин, тирозин,
глутаминовая кислота, лизин и др.
Кочулева Людмила Рамановна, учитель химии МОБУ «Средняя общеобразовательная школа №73» г.
Оренбурга. Опорные конспекты и алгоритмы для подготовки к ЕГЭ.
Физические свойства.
Аминокислоты представляют собой твердые кристаллические вещества, хорошо
растворимые в воде и мало растворимые в органических растворителях. Многие
аминокислоты имеют сладкий вкус. Они плавятся при высоких температурах и обычно
при этом разлагаются. В парообразное состояние переходить не могут.
Водные растворы аминокислот имеют нейтральную, щелочную или кислую среду в
зависимости от количества функциональных групп. Так, глутаминовая кислота образует
кислый раствор (две группы —СООН, одна —NH2), лизин — щелочной (одна группа —
СООН, две —NH2).
Получение.
1. Гидролиз белковых веществ обычно дает сложные смеси аминокислот.
2. Замещение галогена на аминогруппу в соответствующих галогенокислотах.
H2O
Сl – CH2– COOH + 2NH3 → H2N – CH2 – COOН + NH4Cl
глицин
Химические свойства.
Аминокислоты — это органические амфотерные соединения. Они содержат в
составе молекулы две функциональные группы противоположного характера:
аминогруппу с основными свойствами и карбоксильную группу с кислотными
свойствами. Аминокислоты реагируют как с кислотами, так и с основаниями:
1. Взаимодействие с кислотами.
NH2-CH2-COOH + HCl → Cl[NH3-CH2-COOH]
2. Взаимодействие со щелочами.
NH2-CH2-COOH + NaOH → NH2-CH2-COONa + H2O
3. Взаимодействие со спиртами с образованием сложных эфиров.
4. Образование дипептидов (далее полипептидов).
5. Качественные реакции на аминокислоты:
o Все аминокислоты окисляются нингидрином с образованием продуктов,
окрашенных в сине-фиолетовый цвет.
o При нагревании ароматических аминокислот с концентрированной азотной
кислотой происходит нитрование бензольного кольца и образуются соединения,
окрашенные в желтый цвет. Эта реакция называется ксантопротеиновой (от греч. ксантос
— желтый).
В9. Вычисление массы растворенного вещества,
определенной массе раствора с известной массовой долей.
содержащегося
1 тип - вычисление массовой доли растворённого вещества
Пример 1. Определите массовую долю нитрата калия в растворе, полученном
растворением 50 г нитрата калия в 200 г воды.
Дано:
Решение:
m(KNO3) = 50 г
w (в-ва) = m(в-ва)/ m(р-ра) ·100%
m(Н2О) = 200 г
w (KNO3) =50 г/ 250 г·100% = 20%
w (KNO3) -?
2 тип - вычисление массы растворённого вещества
w перевести в доли от единицы: например 50% = 0,5; 3% = 0,03 и т.д.
Кочулева Людмила Рамановна, учитель химии МОБУ «Средняя общеобразовательная школа №73» г.
Оренбурга. Опорные конспекты и алгоритмы для подготовки к ЕГЭ.
в
Пример 2. Вычислите массу гидроксида калия в растворе объёмом 600 мл и
плотностью 1,082 г/мл, если массовая доля гидроксида калия составляет 10%.
Дано:
Решение:
V(р-ра) = 600 мл
m (в-ва)= w (в-ва) · m(р-ра) /100%
w (КОН) = 10%
m(р-ра) = p · V(р-ра) =1,082 г/мл · 600 мл = 649,2 г
= 0,1
p = 1,082 г/мл
m (КОН) =649,2 г · 0,1 = 64,92 г
m (КОН)-?
3 тип - смешивание растворов с разными концентрациями одного вещества
w перевести в доли от единицы.
Пример 3. Смешали 300 г раствора с массовой долей хлорида натрия 20% и 500 г
раствора с массовой долей 40%. Вычислите массовую долю хлорида натрия в полученном
растворе.
Дано:
Решение:
m1 = 300 г
m1 · w1 + m2 · w2 = m3 · w3
w1 = 20% = 0,2
300 г · 0,2 + 500 г · 0,4 = 800 г · w3
m2 = 500 г
60 г + 200 г = 800 г · w3
w2 = 40% = 0,4
260 г = 800 г · w3
w3 -?
w3 =260 г / 800 г = 0,325 = 32,5%
4 тип - разбавление водой
w перевести в доли от единицы.
w2 = 0, т.к. в воде не содержится вещество, находящееся в первом растворе
Пример 4. Какую массу воды надо добавить к раствору гидроксида натрия массой
150 г с массовой долей 10%, чтобы получить раствор с массовой долей 2%?
Дано:
Решение:
m1 = 150 г
m1 · w1 + m2 · w2 = m3 · w3
w1 = 10% = 0,1
150 г · 0,1 + m2 · 0 = (150 г + m2) · 0,02
w2 = 0
15 г + 0 = 3 г + 0,02 m2
w3= 2% = 0,02
0,02 m2 = 12 г
m2 -?
m2 = 12 г / 0,02 = 600 г
5 тип - концентрирование (добавление твёрдого вещества)
w перевести в доли от единицы.
w2 = 100% = 1, т.к. добавляемое вещество чистое.
Пример 5. Какую массу хлорида бария надо добавить к раствору хлорида бария
массой 150 г с массовой долей 10%, чтобы получить раствор с массовой долей 25%?
Дано:
Решение:
m1 = 150 г
m1 · w1 + m2 · w2 = m3 · w3
w1 = 10% = 0,1
150 г · 0,1 + m2 · 1 = (150 г + m2) · 0,25
w2 = 100% = 1
15 г + 1m2 = 37,5 г + 0,25 m2
w3= 25% = 0,25
0,75 m2 = 22,5 г
m2 -?
m2 = 22,5 г / 0,75 = 30 г
6 тип - упаривание раствора (частичное выпаривание)
w перевести в доли от единицы.
Пример 6. Вычислите массовую долю хлорида натрия в растворе, если из 200 г 30%
раствора испарилось 50 г воды.
Дано:
Решение:
Кочулева Людмила Рамановна, учитель химии МОБУ «Средняя общеобразовательная школа №73» г.
Оренбурга. Опорные конспекты и алгоритмы для подготовки к ЕГЭ.
m1 = 200 г
w1 = 30% = 0,3
m2 = 200 г- 50 г
= 150 г
m1 · w1 = m2 · w2
200 г · 0,3 = 150 г · w2
60 г = 150 г · w2
w2 = 60 г/150 г
w2 = 0,4 = 40%
w2-?
В10. Расчеты: массы вещества или объема газов по известному количеству
вещества, массе или объему одного из участвующих в реакции веществ.
По
условию
По
уравнению
m
n
V
n
M
n
n
Vm
=22,4
л/моль
m= n · M
V=n·V
Объёмные отношения газов: по уравнению записать только n.
Кочулева Людмила Рамановна, учитель химии МОБУ «Средняя общеобразовательная школа №73» г.
Оренбурга. Опорные конспекты и алгоритмы для подготовки к ЕГЭ.
Download